You are on page 1of 229

Economic for Managers

Some important instructions and note:


- These notes are only for your knowledge and for your study purpose.
- These notes are prepared by the good subject expert team which will help to clear
your concept for the Exam but there is no guarantee or surety that it will come in
you Exams.
- Its requested you to please study your self and not dependent on these notes.
- Very important thing is that you have not suppose to blaim us for not matching
your exam questions.
- We are already alert you for this concern by marketing team.

Note : This study materials only for study reference purpose

Section-A
____________________________________________________
1. Explain the reason why demand curves slopes download?
Ans- The demand curve slopes downward because of the assumption that utility is generated by
a variety of consumption activities.
Downward sloping demand curve means a rational consumer will demand more of a commodity
when its price falls. Some of the reasons for. the phenomenon would be:
Income Effect: When price of a commodity falls, consumer's real income rises that is he can now
purchase more of the commodity with the same income.
Substitution Effect : When price of a commodity falls, it become cheaper than its substitute good
so the users of substitute good would shift their consumption to the given good and therefore the
quantity demanded of the commodity will rise.
Law of Diminishing Marginal
Utility: It states that the satisfaction derived from a commodity will diminish from every
successive unit. So the consumer would be willing to pay less and less for each successive unit
.
2. Define monopoly. What is the basic difference between monopoly and monopolistic
competition?
Ans- .Meaning Monopoly refers to a market structure where a single seller produces/sells
product to large number of buyers.

Number of players One Two to Ten or even more.


Product differentiation ExtreME Slight
Degree of control over price Considerable but very regulated. Some
Competition Does not exist. Stiff competition exist between firms.
Demand curve Steep Flat
Barriers to entry and exit Many No
Difference between firm and industry No Yes

3. What are the functions of business manager? How does economics help business
managers in performing their functions ?
Ans- Planning: This step involves mapping out exactly how to achieve a particular goal. Say, for
example, that the organization's goal is to improve company sales. The manager first needs to
decide which steps are necessary to accomplish that goal. These steps may include increasing
advertising, inventory, and sales staff. These necessary steps are developed into a plan. When the
plan is in place, the manager can follow it to accomplish the goal of improving company sales.
Organizing: After a plan is in place, a manager needs to organize her team and materials
according to her plan. Assigning work and granting authority are two important elements of
organizing.
Staffing: After a manager discerns his area's needs, he may decide to beef up his staffing by
recruiting, selecting, training, and developing employees. A manager in a large organization
often works with the company's human resources department to accomplish this goal.
Leading: A manager needs to do more than just plan, organize, and staff her team to achieve a
goal. She must also lead. Leading involves motivating, communicating, guiding, and
encouraging. It requires the manager to coach, assist, and problem solve with employees.
Controlling: After the other elements are in place, a manager's job is not finished. He needs to
continuously check results against goals and take any corrective actions necessary to make sure
that his area's plans remain on track.
All managers at all levels of every organization perform these functions, but the amount of time
a manager spends on each one depends on both the level of management and the specific
organization.

4. How monetary policy helps in controlling inflation in the economy?


Ans Increased interest rates will help reduce the growth of aggregate demand in the economy.
The slower growth will then lead to lower inflation. Higher interest rates reduce consumer
spending because: Increased interest rates increase the cost of borrowing, discouraging
consumers from borrowing and spending.

5. What Are The Features Of Perfect Competition?


Ans.
Main Features of Perfect Competition
The following are the characteristics or main features of perfect competition:-

1. Many Sellers
In this market, there are many sellers who form total of market supply. Individually, seller is a
firm and collectively, it is an industry. In perfect competition, price of commodity is decided by
market forces of demand and supply. i.e. by buyers and sellers collectively. Here, no individual
seller is in a position to change the price by controlling supply. Because individual seller's
individual supply is a very small part of total supply. So, if that seller alone raises the price, his
product will become costlier than other and automatically, he will be out of market. Hence, that
seller has to accept the price which is decided by market forces of demand and supply. This
ensures single price in the market and in this way, seller becomes price taker and not price
maker.

2. Many Buyers
Individual buyer cannot control the price by changing or controlling the demand. Because
individual buyer's individual demand is a very small part of total demand or market demand.
Every buyer has to accept the price decided by market forces of demand and supply. In this way,
all buyers are price takers and not price makers. This also ensures existence of single price in
market.

3. Homogenous Product
In this case, all sellers produce homogeneous i.e. perfectly identical products. All products are
perfectly same in terms of size, shape, taste, colour, ingredients, quality, trade marks etc. This
ensures the existence of single price in the market.

4. Zero Advertisement Cost


Since all products are identical in features like quality, taste, design etc., there is no scope for
product differentiation. So advertisement cost is nil.

5. Free Entry and Exit


There are no restrictions on entry and exit of firms. This feature ensures existence of normal
profit in perfect competition. When profit is more, new firms enter the market and this leads to
competition. Entry of new firms competing with each other results into increase in supply and
fall in price. So, this reduces profit from abnormal to normal level.
When profit is low (below normal level), some firms may exit the market. This leads to fall in
supply. So remaining firms raise their prices and their profits go up. So again this ensures normal
level of profit.

6. Perfect Knowledge
On the front of both, buyers and sellers, perfect knowledge regarding market and pricing
conditions is expected. So, no buyer will pay price higher than market price and no seller will
charge lower price than market price.

7. Perfect Mobility of Factors


This feature is essential to keep supply at par with demand. If all factors are easily mobile
(moveable) from one line of production to another, then it becomes easy to adjust supply as per
demand.
Whenever demand is more additional factors should be moved into industry to increase supply
and vice versa. In this way, with the help of stable demand and supply, we can maintain single
price in the Market.

8. No Government Intervention
Since market has been controlled by the forces of demand and supply, there is no government
intervention in the form of taxes, subsidies, licensing policy, control over the supply of raw
materials, etc.

9. No Transport Cost
It is assumed that buyers and sellers are close to market, so there is no transport cost. This
ensures existence of single price in market.

6. What Is An Indifference Curve? Discuss The Properties Of Indifference Curves.


Ans.

An indifference curve is a graph showing different bundles of goods between which a consumer
is indifferent. That is, at each point on the curve, the consumer has no preference for one bundle
over another. One can equivalently refer to each point on the indifference curve as rendering the
same level of utility (satisfaction) for the consumer. In other words an indifference curve is the
locus of various points showing different combinations of two goods providing equal utility to
the consumer. Utility is then a device to represent preferences rather than something from which
preferences come. The main use of indifference curves is in the representation of potentially
observable demand patterns for individual consumers over commodity bundles.

Properties of Indifference Curves


The concept of an indifference curve is predicated on the idea that a given consumer has rational
preferences in regard to the purchase of groupings of goods, with a series of key properties that
define the process of mapping these curves:
1. Indifference curves only reside in the non-negative quadrant of a two-dimensional
graphical illustration (or the upper right). This assumes that negative quantities are
meaningless - one can't consume a negative amount of a good.

2. Indifference curves are always negatively sloped. This is based on the assumption that a
consumer is always better off consuming more of a good, so as quantity consumed of one
good increases, total satisfaction would increase if not offset by a decrease in the quantity
consumed of another good. This also assumes that the marginal rate of substitution is
always positive.

3. All curves projected on the indifference map must also be transitive to ensure that if A
is preferred to B and B is preferred to C, C is not also preferred to A. This is manifested
in indifference curves that never intersect. To demonstrate this see , which demonstrates
that each of the three curves are transitive.

4. Nearly all indifference lines will be convex, or curving inwards at the center (towards
the bottom left). This demonstrates that increasingly high quantities of one good over
another have a cost in respect to their overall utility per unit (diminishing returns). It is
technically possible for indifference curves to be perfectly straight as well, which would
imply that the two goods are identical (perfect substitutes).
Combining these various properties, one can highlight a number of critical implications of
consumer purchasing behavior and the concept of utility. Consumers naturally desire a bundle of
goods that is varied (hence the convex curves for most comparisons) in order to maximize their
utility. Similarly, all indifference curves will naturally identify diminishing rates of substitution
as the quantity increases for a certain good compared to another, and can create demand
projections of prospective supply.

7. How fiscal policy helps in controlling inflation in the economy?


Ans.
Inflation can be reduced by policies that slow down the growth of AD and/or boost the rate of
growth of aggregate supply (AS)
Fiscal policy:
1. Controlling aggregate demand is important if inflation is to be controlled. If the
government believes that AD is too high, it may choose to ‘tighten fiscal policy’ by
reducing its own spending on public and merit goods or welfare payments
2. It can choose to raise direct taxes, leading to a reduction in real disposable income
3. The consequence may be that demand and output are lower which has a negative
effect on jobs and real economic growth in the short-term
Monetary policy:
1. A ‘tightening of monetary policy’ involves the central bank introducing a period of
higher interest rates to reduce consumer and investment spending
2. Higher interest rates may cause the exchange rate to appreciate in value bringing
about a fall in the cost of imported goods and services and also a fall in demand for
exports (X)
Supply side economic policies:
1. Supply side policies seek to increase productivity, competition and innovation – all
of which can maintain lower prices. These are ways of controlling inflation in the
medium term
i. ​A reduction in company taxes to encourage greater investment
ii. ​A reduction in taxes which increases risk-taking and incentives to work
– a cut in income taxes can be considered both a fiscal and a supply-side
policy
iii. ​Policies to open a market to more competition to increase supply and
lower prices
Rising productivity will cause an outward shift of aggregate supply
Direct controls - a government might choose to introduce direct controls on some prices and
wages
1. Public sector pay awards – the annual increase in government sector pay might be
tightly controlled or even froze (this means a real wage decrease).
2. The prices of some utilities such as water bills are subject to regulatory control – if
the price capping regime changes, this can have a short-term effect on the rate of
inflation
Evaluation points – how best can inflation be controlled?
· ​The most appropriate way to control inflation in the short term is for the government
and the central bank to keep control of aggregate demand to a level consistent with
our productive capacity
· ​AD is probably better controlled through the use of monetary policy rather than an
over-reliance on using fiscal policy as an instrument of demand-management
· ​Controlling demand to limit inflation is likely to be ineffective in the short run if the
main causes are due to external shocks such as high world food and energy prices
· ​The UK is an open economy in which inflation is strongly affected by events in the
rest of the world
· ​In the long run, it is the growth of a country’s supply-side productive potential that
gives an economy the flexibility to grow without suffering from acceleration in cost
and price inflation.

8. Define Oligopoly. What is the basic difference between oligopoly and monopolistic?
Ans.
Oligopoly is a market structure in which a small number of firms has the large majority of
market share. An oligopoly is similar to a monopoly, except that rather than one firm, two or
more firms dominate the market. There is no precise upper limit to the number of firms in an
oligopoly, but the number must be low enough that the actions of one firm significantly impact
and influence the others.

Difference Between Oligopoly and Monopolistic Competition


Oligopoly is a market structure containing a small number of relatively large firms, with
significant barriers to entry of other firms. Monopolistic competition is a market structure
containing a large number of relatively small firms, with relative freedom of entry and exit.
While it might seem as though the difference between oligopoly and monopolistic competition is
clear cut, such is not always the case.

A comparison between these two market structures is offers a little insight into each.
Many or Few: The primary difference between oligopoly and monopolistic competition is the
relative size and the market control of each firm based on the number of competitors in the
market. However, there is no clear-cut dividing line between these two market structures. It is
not possible to say that some number like 50 firms is the dividing line, such that 50 firms make it
an oligopoly and 51 make it monopolistic competition.

While one industry containing only 3 firms is clearly oligopoly and another industry with 30,000
firms is undoubtedly monopolistic competition, the structure of an industry with 30 firms or 300
firms is not as obvious.

Such industries might have characteristics of both oligopoly and monopolistic competition. As
the number of firms decreases, the firms tend to behave more like oligopoly and less like
monopolistic competition.

Dominance by a Few: In some cases status depends more on the dominance of a few firms rather
than the total number of firms in the industry. An industry with 3,000 relatively equal firms is
most assuredly monopolistic competition. However, an industry with 3,000 firms that is
dominated by 3 relatively large firms is most likely oligopoly. For example, the petroleum
extraction industry has thousands of firms, but a handful of the largest firms dominate the
market, making it an oligopoly.

Geographic Area: In other cases, the geographic size of the market is the prime determinant of
market structure. A particular industry might be monopolistic competition in a large city, but
oligopoly in a smaller town. Retail sales offer an example. Larger cities usually have hundreds,
even thousands, of shopping alternatives, including discount supercenters, mom-and-pop stores,
shopping malls, and nationwide chains. Such a market is monopolistic competition. Smaller
towns, however, tend to have fewer stores, perhaps a single super center or shopping mall and a
handful of stores located in a small downtown area. Such a market is oligopoly.

Barriers to Entry: A key difference between oligopoly and monopolistic competition is barriers
to entry. Oligopoly barriers are high. Monopolistic competition barriers are low. However, entry
barriers are a matter of degree. The need for government authorization is one entry barrier that
can create oligopoly, especially if entry is limited to only a few firms. However, it can also create
monopolistic competition if a larger number are allowed entry. Other barriers, such as start-up
cost and resource ownership, also limit entry to different degrees, leading to either oligopoly or
monopolistic competition. Moreover, these entry barriers can change over time, transforming
oligopoly and monopolistic competition or vice versa.

9. “Managerial economics is an integration o economic theory, decision science and


business management.” Comment.

Ans​. Managerial economics is a science that deals with the application of various economic
theories, principles, concepts and techniques to business management in order to solve business
and management problems. It deals with the practical application of economic theory and
methodology to decision-making problems faced by private, public and non-profit making
organizations.

The same idea has been expressed by Spencer and Siegelman in the following words.
“Managerial Economics is the integration of economic theory with business practice for the
purpose of facilitating decision making and forward planning by the management” .According to
Mcnair and Meriam, “Managerial economics is the use of economic modes of thought to analyze
business situation”. Brigham and Pappas define managerial economics as,” the application of
economic theory and methodology to business administration practice”. Joel dean is of the
opinion that use of economic analysis in formulating business and management policies is
known as managerial economics.

Managerial economics is a highly specialized and new branch of economics developed in recent
years. It highlights on practical application of principles and concepts of economics in to
business decision making process in order to find out optimal solutions to managerial problems.
It fills up the gap between abstract economic theory and managerial practice. It lies mid-way
between economic theory and business practice and serves as a connecting link between the two.
10. “Economics is a science of choice when faced with unlimited ends and scarce resources
having alternative uses.” Comment.

Ans. Economics is the study of how economics agents or societies choose to use scare
productive resources that have alternative that have alternative uses to satisfy wants which are
unlimited and of varying degree of importance. The main concern of economics is economic
problem: its identification, description, explanation and solution The source if any economic
problem is scarcity. Scarcity of resources forces economic agents to choose among alternative.
Therefore, economic problem can be said to be a problem of choice and valuation of alternatives.
The problem of choice arises because limited resources with alternatives uses are to be utilized to
satisfy unlimited wants, which are of varying degrees of importance.

11. Explain the utility analysis for understanding consumer behaviour and demand.

Ans. UTILITY ANALYSIS:

A subset of consumer demand theory that analysis consumer behaviour and market demand
using total utility and marginal utility. The key principle of utility analysis is the law of
diminishing marginal utility, which offers an explanation for the law of demand and the negative
slope of the demand curve.

Utility analysis, a subset of consumer demand theory, provides insight into an understanding of
market demand and forms a cornerstone of modern microeconomics. In particular, this analysis
investigates consumer behaviour, especially market purchases, is based on the satisfaction of
wants and needs (that is, utility) generated from the consumption of a good.

Utility analysis is primarily taught in introductory courses. A more sophisticated version of


consumer demand theory relies on the analysis of indifference curves and is more commonly
found at the intermediate course level and above.

12. What is the law of Diminishing Marginal Utility? Explain law with empirical example.

Ans. A law of economics stating that as a person increases consumption of a product - while
keeping consumption of other products constant - there is a decline in the marginal utility that
person derives from consuming each additional unit of that product. ​When you have that first
bite of chocolate cake it seems like you just can't get enough of it! However, as you eat your
second piece and third piece you may start to realize you have had enough cake. For each
additional unit of a good (in this case for each additional piece of cake) the added satisfaction
you receive from consuming the good decreases. This is known as diminishing marginal utility.

Example: ​Buffets are a prime example of restaurants depending on diminishing marginal utility.
For one price, you can eat all the food you want. The reason that buffets work economically is
that people reach a point where the utility (pleasure) gained from an additional plate of food isn't
worth the cost of eating that additional plate.

13. Explain the concept of Production Function with the help of a two-input and one-output
case.
Ans. Concept of Production Function: ​As seen earlier, production involves a transformation of
inputs into output. The technical relationship between inputs and output which gives maximum
output is called production function. Production function gives different combinations inputs that
produce maximum level of output. A production function is written as Q = f (I1....In) where, Q is
output, f is a functional relationship and I1 to In are quantities of different inputs. To keep the
things as simple as possible, at this stage, we will define production function as follows

Q = f (L,K)

Where

Q is output

L is labour used in process of production

K is capital used in the process of production

That means, firm’s output depends upon the labour employed and units of capital services used
up in the production. Now, suppose a firm requires to increase its output, it cannot change the
quantities of labour and capital at the same speed. Generally labour units can be employed at a
short notice but it takes more time to install machinery or equipment i.e. capital. In the short run,
one of the inputs may remain fixed say capital. Other inputs that may remain fixed in the short
run may be supply of skilled labour, land plot, etc. But some inputs like unskilled labour units
can be easily changed even in the short run.

So we can further define production function using the short-run and long-run period.

Short run production function  ⇒  Q  =  f  (L,K)  where  L  is  variable  and  K  is 
fixed factor of production.

Long run production function  ⇒  Q  =  f  (L,K)  where  both  L  and  K  are 


variable factors of production.

14. What is the law of diminishing return as applied to any production system?

Ans. In economics, diminishing returns (also called diminishing marginal returns) is the decrease
in the marginal output of a production process as the amount of a single factor of production is
increased, while the amounts of all other factors of production stay constant. The law of
diminishing returns states that in all productive processes, adding more of one factor of
production, while holding all others constant ("ceteris paribus"), will at some point yield lower
per-unit returns . The law of diminishing returns does not imply that adding more of a factor will
decrease the total production, a condition known as negative returns, though in fact this is
common.
For example, the use of fertilizer improves crop production on farms and in gardens; but at some
point, adding more and more fertilizer improves the yield less per unit of fertilizer, and excessive
quantities can even reduce the yield. A common sort of example is adding more workers to a job,
such as assembling a car on a factory floor. At some point, adding more workers causes
problems such as workers getting in each other's way or frequently finding themselves waiting
for access to a part. In all of these processes, producing one more unit of output will eventually
cost increasingly more, due to inputs being used less and less effectively.

This increase in the marginal cost of output as production increases can be graphed as the
marginal cost curve, with quantity of output on the x axis and marginal cost on the y axis. For
many firms, the marginal cost curve will initially be downward sloping, representing added
efficiency as production increases. If the law of diminishing returns holds, however, the marginal
cost curve will eventually slope upward and continue to rise, representing the higher and higher
marginal costs associated with additional output.

Source: Boundless. “The Law of Diminishing Returns.” Boundless Economics. Boundless, 14


Nov. 2014. Retrieved 03 Dec. 2014 from
https://www.boundless.com/economics/textbooks/boundless-economics-textbook/production-9/t
he-production-function-63/the-law-of-diminishing-returns-238-12336/

15. Write down the differences between pure competition and perfect competition.

Ans. Pure competition is said to exist in a market where (a) there is a large number of buyers and
sellers (b) products are homogeneous and (c) there is freedom of entry and exit of buyers and
sellers. The implication of these conditions taken as a whole is that no individual seller is in a
position to influence the price in the market. In perfect competition, all the three features of pure
competition exist. Besides these, perfect competition has more features. These are (d) perfect
knowledge of the buyers and sellers regarding the market conditions (e) perfect mobility of
factors of production (f) absence of transport cost and (g) uniform price.

Thus, perfect competition is not only pure but also free from other imperfection. It is a broader
concept than pure competition. The essential feature of pure competition is the absence of any
monopoly element.

In the words of Chamberlain, pure competition means “Competition unalloyed with monopoly
elements” whereas perfect competition involves “perfection in many other respects than the
absence of monopoly.” It is possible to come across pure competition in real life but not perfect
competition.

The American economists attach great importance to pure competition whereas the English
economists emphasize perfect competition. The difference between the two is one of degree and
not of kind.

The demand curve facing a firm under pure and perfect competition is a horizontal straight line.
It is due to their characteristics. Under perfect competition, there is large number of buyers and
sellers. The products are homogeneous.
There is freedom of entry and exist of buyers and sellers. Factors of production are freely mobile.
The transport cost is nil. There is no place for advertisement. Single price prevails in the market.
Buyers and sellers are price takers. Sellers are output adjusters. Each seller and each buyer faces
a price that is determined by the market forces, which are beyond his control.

1. Explain why the demand curve of a firm in Oligopolistic market structure is


indeterminate?
Ans-:
The Structure Of The Market Structure Of Oligopoly And The Difficulty In Predicting Output
And Profits

Market structure of oligopoly

Oligopoly is a market structure where there are a few firms producing all or most of the market
supply of a particular good or service and whose decisions about the industry's output can affect
competitors. Examples of oligopolistic structures are supermarket, banking industry and
pharmaceutical industry.

The characteristics of the oligopoly are:

● Small number of large firms dominate the industry


● High degree of interdependence: the behaviour of firms are affected by what they
believe other rivalry firms might do
● High barriers to entry that restrict new firms to enter the industry e.g. control of
technology
● Price stability within the markets
● Goods are highly differentiated or standardized
● Non ​price competitive e.g. free deliveries and installation, extended warranties
● Restricted information.

Oligopolies do not compete on prices. Price wars tend to lead to lower profits, leaving a little
change to market shares. However, Oligopolies firms tend to charge reasonably premium prices
but they compete through advertising and other promotional means. Existing companies are safe
from new companies entering the market because barriers to entry to the market are high. For
example, if products are heavily promoted and producers have a number of existing successful
brands, it will be very costly and difficult for new firms to establish their own new brand in an
oligopoly market.

Because there are few firms in an oligopoly industry, each firms output is a large share of the
market. As a result, each firm's pricing and output decisions have a substantial effect on the
profitability of other firms. In addition, when making decisions relating to price or output, each
firm has to take into consideration the likely reaction of rival firms. Because of this
interdependence, oligopoly firms engage in strategic behaviour. Strategic behaviour means when
the best outcome of a firm is determined by the actions of other firms.
Oligopolists are drawn in two different directions, either to compete with each other or to collude
with each other. If they collude, they end up acting as monopoly and thereby maximising the
industry's profits. However they are often tempted to compete with each other inorder to gain a
bigger share of the profit of the industry.

In oligopoly, there is always tension between co-operation and self-interest. The group of
oligopoly is better off cooperating and acting like the monopoly. However, because the
oligopolist cares about their own personal profit, there is an incentive for them to act on their
won. This will therefore limit the ability of the group to act as a monopoly.

Oligopolists maximize their total profits by forming a cartel and acting like a monopolist.
However, if they make decisions about production levels individually, there would be a greater
quantity and a lower price compared to operating as a monopoly.

For these reasons, Oligopoly prices and output are indeterminate; they may be anything within
the range and are unpredictable.

2. Explain Baumol’s Sales Revenue Maximization model?


Ans-:​Sales maximization model is an alternative model for profit maximization. This model is developed
by Prof. Boumol, an American economist. This alternative goal has assumed greater significance in the
context of the growth of Oligopolistic firms.
Baumol’s sales revenue maximization model highlights that the primary objective of a firm is to
maximize its sales rather than profit maximization. It states that the goal of the firm is maximization of
sales revenue subject to a minimum profit constraint. The minimum profit constraint is determined by the
expectations of the share holders. This is because no company can displease the share holders.
“Though businessmen are interested in the scale of their operations partly because they see some
connection between scale and profits, I think management’s concern with the level of sales goes
considerably further. In my dealings with them I have been struck with the importance the oligopolistic
enterprises attach to the value of their sales. A small reversal in an upward sales trend that can quite
reasonably be dismissed as a random movement sometimes leads to a major review of the concern’s
selling and production methods, its product lines, and even its internal organizational structure.” –
Baumol
Assumptions:
The theory is based on the following assumptions:
1. There is a single period time horizon of the firm.
2. The firm aims at maximising its total sales revenue in the long run subject to a profit constraint.
3. The firm’s minimum profit constraint is set competitively in terms of the current market value of its
shares.
4. The firm is oligopolistic whose cost cures are U-shaped and the demand curve is downward sloping. Its
total cost and revenue curves are also of the conventional type.
3. Differentiate between returns to factor and returns to scale?
Ans-:
Returns to a factor and returns to scale are two important laws of production. Both laws explain the
relation between inputs and output. Both laws have three stages of increasing, decreasing and constant
returns. Even then, there are fundamental differences between the two laws.
Returns to a factor relate to the short period production function when one factor is varied keeping the
other factor fixed in order to have more output, the marginal returns of the Variable factor diminish. On
the other hand, returns to scale relate to the long period production function when a firm changes its scale
of production by changing one or more of its factors.
5 Major Differences between Returns to Scale and Returns to a factor Proportions are listed below:
Returns to a factor:
1. Only one factor varies while all the rest are fixed.
2. The factor-proportion varies as more and more of the units of the variable factor are employed to
increase output.
4. Returns to a factor or to variable proportions end up in negative returns.
3. It is a short-run phenomenon.
5. Returns to variable proportions are caused by indivisibility of certain fixed factors, specialisation of
certain variable factors, or sub-optimal factor proportions.
Returns to scale:
1. All or at least two factors vary.
2. Factor proportion called scale does not vary. Factors are increased in same proportion to increase
output.
3. It is a long-run phenomenon.
4. Returns to scale end up in decreasing return
5. Returns to scale can be attributed to economies and diseconomies of scale caused by technical and/or
managerial indivisibilities, exhaustibility of natural and managerial resources, or depreciability of certain
factors.

4. Discuss the policy recommendation to curb the inflationary pressures in the country?
Ans-:
Inflation is generally controlled by the Central Bank and/or the government. The main policy
used is monetary policy – set by Central Banks. However, in theory, there are a variety of tools
to control inflation including:

1. Monetary policy – Setting interest rates. Higher interest rates reduce demand, leading to
lower economic growth and lower inflation
2. Control of money supply – Monetarists argue there is a close link between the money
supply and inflation, therefore controlling money supply can control inflation.
3. Supply-side policies – policies to increase competitiveness and efficiency of the
economy, putting downward pressure on long-term costs.
4. Fiscal policy – a higher rate of income tax could reduce spending and inflationary
pressures.
5. Wage controls. Trying to control wages could, in theory, help to reduce inflationary
pressures. However, apart from the 1970s, it has been rarely used.

Monetary Policy
In a period of rapid economic growth, demand in the economy could be growing faster than its
capacity to meet it. This leads to inflationary pressures as firms respond to shortages by putting
up the price. We can term this demand-pull inflation. Therefore, reducing the growth of
aggregate demand (AD) should reduce inflationary pressures.

The Central bank could increase interest rates. Higher rates make borrowing more expensive and
saving more attractive. This should lead to lower growth in consumer spending and investment.
See more on higher interest rates
A higher interest rate should also lead to a higher exchange rate, which helps to reduce
inflationary pressure by

• Making imports cheaper.


• Reducing demand for exports and
• Increasing incentive for exporters to cut costs.

Interest rates were increased in the late 1980s / 1990 to try and control the rise in inflation

Inflation target
As part of monetary policy, many countries have an inflation target (e.g. UK inflation target of
2%). The argument is that if people believe the inflation target is credible, then it will help to
lower inflation expectations. If inflation expectations are low, it becomes easier to control
inflation.

Countries have also made Central Bank independent in setting monetary policy. The argument is
that an independent Central Bank will be free from political pressures to set low-interest rates
before an election.

Fiscal Policy
The government can increase taxes (such as income tax and VAT) and cut spending. This
improves the budget situation and helps to reduce demand in the economy.

Both these policies reduce inflation by reducing the growth of Aggregate Demand. If economic
growth is rapid, reducing the growth of AD can reduce inflationary pressures without causing a
recession.

If a country had high inflation and negative growth, then reducing aggregate demand would be
more unpalatable as reducing inflation would lead to lower output and higher unemployment.
They could still reduce inflation, but, it would be much more damaging to the economy.

Other Policies to Reduce Inflation


Wage Control
If inflation is caused by wage inflation (e.g. powerful unions bargaining for higher real wages),
then limiting wage growth can help to moderate inflation. Lower wage growth helps to reduce
cost-push inflation and helps to moderate demand-pull inflation.
However, as the UK discovered in the 1970s, it can be difficult to control inflation through
incomes policies, especially if the unions are powerful.

Monetarism
Monetarism seeks to control inflation by controlling the money supply. Monetarists believe there
is a strong link between the money supply and inflation. If you can control the growth of the
money supply, then you should be able to bring inflation under control. Monetarists would stress
policies such as:

• Higher interest rates (tightening monetary policy)


• Reducing budget deficit (deflationary fiscal policy)
• Control of money being created by the government

However, in practice, the link between money supply and inflation is less strong.

Supply Side Policies


Often inflation is caused by persistent uncompetitiveness and rising costs. Supply-side policies
may enable the economy to become more competitive and help to moderate inflationary
pressures. For example, more flexible labour markets may help reduce inflationary pressure.

However, supply-side policies can take a long time, and cannot deal with inflation caused by
rising demand.

Ways To Reduce Hyperinflation – change currency


In a period of hyperinflation, conventional policies may be unsuitable. Expectations of future
inflation may be hard to change. When people have lost confidence in a currency, it may be
necessary to introduce a new currency or use another like the dollar (e.g. Zimbabwe
hyperinflation).

Ways to Reduce Cost-Push Inflation


Cost-push inflation (e.g. rising oil prices can lead to inflation and lower growth. This is the worst
of both worlds and is more difficult to control without leading to lower growth.

Q1-: Explain the characteristics features of perfect competition?


Ans.

A Perfect Competition market is that type of market in which the number of buyers and sellers is
very large, all are engaged in buying and selling a homogeneous product without any artificial
restrictions and possessing perfect knowledge of the market at a time.

Characteristics of Perfect Competition:


The following characteristics are essential for the existence of Perfect Competition:
1. Large Number of Buyers and Sellers:
The first condition is that the number of buyers and sellers must be so large that none of them
individually is in a position to influence the price and output of the industry as a whole. In the
market the position of a purchaser or a seller is just like a drop of water in an ocean.

2. Homogeneity of the Product:


Each firm should produce and sell a homogeneous product so that no buyer has any preference
for the product of any individual seller over others. If goods will be homogeneous then price will
also be uniform everywhere.

3. Free Entry and Exit of Firms:


The firm should be free to enter or leave the firm. If there is hope of profit the firm will enter in
business and if there is profitability of loss, the firm will leave the business.

4. Perfect Knowledge of the Market:


Buyers and sellers must possess complete knowledge about the prices at which goods are being
bought and sold and of the prices at which others are prepared to buy and sell. This will help in
having uniformity in prices.

5. Perfect Mobility of the Factors of Production and Goods:


There should be perfect mobility of goods and factors between industries. Goods should be free
to move to those places where they can fetch the highest price.

6. Absence of Price Control:


There should be complete openness in buying and selling of goods. Here prices are liable to
change freely in response to demand and supply conditions.

7. Perfect Competition among Buyers and Sellers:


In this purchasers and sellers have got complete freedom for bargaining, no restrictions in
charging more or demanding less, competition feeling must be present there.

8. Absence of Transport Cost:


There must be absence of transport cost. In having less or negligible transport cost will help
complete market in maintaining uniformity in price.

9. One Price of the Commodity:


There is always one price of the commodity available in the market.

10. Independent Relationship between Buyers and Sellers:


There should not be any attachment between sellers and purchasers in the market. Here, the
seller should not show prick and choose method in accepting the price of the commodity. If we
will see from the close we will find that in real life “Perfect Competition is a pure myth.”

​Q3. What is National Income? Explain various of National Income? 300 WORD ONLY

Introduction:
National income is an uncertain term which is used interchangeably with national dividend,
national output and national expenditure. On this basis, national income has been defined in a
number of ways. In common parlance, national income means the total value of goods and
services produced annually in a country.

In other words, the total amount of income accruing to a country from economic activities in a
year’s time is known as national income. It includes payments made to all resources in the form
of wages, interest, rent and profits.

Though the definition advanced by Marshall is simple and comprehensive, yet it suffers from a
number of limitations. First, in the present day world, so varied and numerous are the goods and
services produced that it is very difficult to have a correct estimation of them.

Consequently, the national income cannot be calculated correctly. Second, there always exists
the fear of the mistake of double counting, and hence the national income cannot be correctly
estimated. Double counting means that a particular commodity or service like raw material or
labour, etc. might get included in the national income twice or more than twice.

For example, a peasant sells wheat worth Rs.2000 to a flour mill which sells wheat flour to the
wholesaler and the wholesaler sells it to the retailer who, in turn, sells it to the customers. If each
time, this wheat or its flour is taken into consideration, it will work out to Rs.8000, whereas, in
actuality, there is only an increase of Rs.2000 in the national income.

Third, it is again not possible to have a correct estimation of national income because many of
the commodities produced are not marketed and the producer either keeps the produce for
self-consumption or exchanges it for other commodities. It generally happens in an agriculture-
oriented country like India. Thus the volume of national income is underestimated.
A.C. Pigou has in his definition of national income included that income which can be measured
in terms of money. In the words of Pigou, “National income is that part of objective income of
the community, including of course income derived from abroad which can be measured in
money.”

This definition is better than the Marshallian definition. It has proved to be more practical also.
While calculating the national income now-a- days, estimates are prepared in accordance with
the two criteria laid down in this definition.
First, avoiding double counting, the goods and services which can be measured in money are
included in national income. Second, income received on account of investment in foreign
countries is included in national income.

1. Discuss the fundamental nature of Management Economies with respect to the three
choice problems of the economy.

ANSWER- A close relationship between management and economics has led to the development
of managerial economics. Management is the guidance, leadership and control of the efforts of a
group of people towards some common objective. While this description does inform about the
purpose or function of management, it tells us little about the nature of the management process.
Koontz and O’Donell define management as the creation and maintenance of an internal
environment in an enterprise where individuals, working together in groups, can perform
efficiently and effectively towards the attainment of group goals. Thus, management is:

a) Coordination

b) An activity or an ongoing process

c) A purposive process

d) An art of getting things done by other people.

On the other hand, economics as stated above is engaged in analyzing and providing answers to
manifestations of the most fundamental problem of scarcity. Scarcity of resources results from
two fundamental facts of life:

a) Human wants are virtually unlimited and insatiable, and

b) Economic resources to satisfy these human demands are limited.


Thus, we cannot have everything we want; we must make choices broadly in regard to the
following:

What to produce?

Societies have to decide the best combination of goods and services to meet their needs. For
example, how many resources should be allocated to consumer goods, and many resources to
capital goods, or how many resources should go to schools, and how many to defence, and so on.

How to produce?

Societies also have to decide the best combination of factors to create the desired output of goods
and services. For example, precisely how much land, labour, and capital should be used produce
consumer goods such as computers and motor cars.

For whom to produce?

Finally, all societies need to decide who will get the output from the country’s economic activity,
and how much they will get. For example, who will get the computers and cars that have been
produced? This is often called the problem of distribution

These three choice problems have become the three central issues of an economy. Economics
has developed several concepts and analytical tools to deal with the question of allocation of
scarce resources among competing ends. The non-trivial problem that needs to be addressed is
how an economy through its various institutions solves or answers the three crucial questions
posed above. There are three ways by which this can be achieved. One, entirely by the market
mechanism, two entirely by the government or finally and more reasonably by a combination of
the first two approaches. Realistically all economies employ the last option, but the relative roles
of the market and government vary across countries.

According to the central deduction of economic theory, under certain conditions, markets
allocate resources efficiently. ‘Efficiency’ has a special meaning in this context. The theory says
that markets will produce an outcome such that, given the economy’s scarce resources, it is
impossible to make anybody better-off without making somebody else worse-off.

2. The demand function of a product is given as Q = 500-5P. Find out the point price
elasticity demand when

a) P = Rs. 15 and Q = 200

a) P = Rs. 50 and Q = 200

What inferences do you draw from the results when the price of a commodity increases from Rs.
15 to Rs. 50, the quantity demanded remaining constant?

ANSWERE-
Price elasticity of demand =%change in quantity / %change in price

Q=500-5P

IF P=RS 15 AND Q=200

Q=200-15P

Hence the change in quantity =Q=500-Q=200

Therefore Q=300

Q=300/100

Q=3%

Change in price=P=5-P=15

P=-10

P=10/100

P=-0.1

P= 1%
Price elasticity of demand=3%/ 1%

=3%

b) If P=50 AND Q=200

Q=500-5P

IF P=RS 50 AND Q=200

Q=200-50P

Hence the change in quantity =Q=500-Q=200

Therefore Q=300

Q=300/100

Q=3%

Change in price=P=5-P=50

P=-45

P=45/100

P=-0.45
P= 45%

Price elasticity of demand=3%/ 45%

=0.066%

3. Distinguish between accounting costs and Economics costs. Explain giving suitable
examples.

Ans.

Economic cost is a more comprehensive idea that accounting costs. Accounting costs only
include what economists call "explicit costs." These are the amounts that a firm actually pays
out to other people in the process of producing their product. So, if you open a business selling
cosmetics from your home, the accounting costs would include things like the price of the
cosmetics, the money you spend on advertising, if any, and the amount that it costs you to go
around selling your product.

Economic costs include "implicit costs," which are the same as opportunity costs. In the
example mentioned above, the economic costs of starting this business would also include the
value of whatever else you could have been doing with your time. Economic costs would also
include, then, the wages that you could have been getting if you had gone to work instead of
opening this business.

The Economic cost is the monetary value of all resources employed in the course of business. It
also refers to the opportunity cost of the inputs used in the enterprise. For example, a business
that operates from a building it owns forfeits the rent that it would have otherwise received if it
rented out the building. In this case, the opportunity cost is the rent forfeited due to business
functions. Economic costs also focus predominantly on implicit costs. Implicit costs are costs
that the business incurs from the use of owned resources. A monetary value for the resources can
be determined, but the resources are not paid for in monetary terms.

Accounting costs, on the other hand, are based on explicit costs incurred by the business. Explicit
costs are costs incurred in normal market transactions. For instance, wages paid to workers are
an explicit cost. Explicit costs are incurred in the purchase of productive resources.

Accounting costs account only for the explicit costs incurred in conducting a business and not
the implicit costs. The explicit costs include the direct costs to the company, such as employee
wages, utility bills (water, electricity, etc.), raw material cost, premises cost, transportation and
storage costs, etc. Since these are expenses for which bills or receipts are available, such costs
can be objectively verified. In fact, accountants only account for accounting costs in the financial
statement of the company. Since these expenses are already incurred, accounting costs are
backward looking.

Economic costs, on the other hand, account for both explicit and implicit costs. Implicit costs is
the opportunity cost in terms of revenue lost by forgoing the next best alternative, say renting out
premises instead of conducting the business there. Implicit costs do not appear on the financial
statements and are not objectively verifiable, since there can be a number of alternative to any
given course of action. Implicit costs are forward looking, since they include the what if (say, we
rented out the premises for next year instead of using it to conduct the business) scenario.
The accounting cost reveals the expenses with production, while the economic costs may be
evaluated as the total of accounting costs and opportunity costs.

The opportunity cost is associated with every decision or action and it represents the value of a
resource that is given in exchange of something else.

The account books do not comprise the opportunity costs but they are emphasized when
computing the expenditure required in acquiring an asset.

Hence, the difference between the economic cost and accounting cost is that the economic cost
comprises the opportunity cost, while the accounting cost does not.

Although the accounting costs are very important to company owners, the economists are mainly
focused on economic costs rather than accounting costs.

The internal and external company reports do include the accounting costs, while the economic
costs are only revealed in internal accounting reports.

4. Explain the functional forms of cost function giving illustration.


ANSWERE-

The crux of the dual approach is then to estimate a manifestation of behavior that economist
know something about.

Thus, instead of estimating production functions that are purely physical forms that economist
have little expertise in developing, we could estimate the cost function that represents cost
minimizing behavior.

We then would be able to determine whether the properties of these cost functions are consistent
with our hypotheses about technology.

However, it is often the direct implications of the cost minimizing behavior that we are interested
in:

How will farmers react to changes in agricultural prices through commodity programs?

What is the impact of a change in input prices (say in an increase in fuel prices) on agricultural
output?

Thus, the dual cost function results are usually sufficient for most question facing agricultural
economists.
Given that we are interested in estimating the cost function directly, the next question involves
how to specify the cost function?

One approach to the estimation of cost functions would then be to hypothesize a primal
production function and derive the theoretically consistent specification for the cost function
based on this primal.

However, this approach would appear too restrictive.

Thus, economists have typically turned to flexible functional forms that allow for a wide variety
of technologies.

A basic approach to the specification of a cost function is to assume that an unspecified function
exists, and then derive a closed form approximation of the function.

One typical approach from optimization theory involves the Taylor series expansion:

General Form of Univariate Taylor Expansion

The real problem is that we don’t know the value of x*. As a result, we approximate this term
with a residual:

Given this approach, we can conjecture the relative size of the approximation error based on the
relative size of the third derivative of the cost function.

Extending this result to vector space, we have:


Given that the cost function is a function of input prices w and output levels y, we could then
stack the two into a single vector and derive the flexible functional form:

This form is typically referred to as the quadratic cost function. It is a second-order Taylor series
expansion to an unknown cost function.

Following the general concept (and ignoring for the moment the error of approximation),
Shephard’s lemma can be applied to this cost specification:

Flexible Cost System

Why have I imposed symmetry?

Why am I only estimating three demand curves?

One generalization of the Taylor series approach involves a transformation of variables.


Specifically, if we assume:

The cost function can be expressed as:

This formulation complicates the Shephard’s lemma results slightly:

Note that letting l go to zero implies


Fourier Expansion

In a univariate sense, any function can be approximated by a series of sine and cosine curves:

Where the lI are different periodicities.

Extending this representation to a multivariate formulation:

Where li is a constant for periodicity and ka is referred to as an Elementary Multi-Index:

K=1

K=2

This representation minimizes the Sobolev Norm, which says that it does a better job
approximating the derivatives of the function. In fact, it represents up to the kth derivative of the
function.

Note that if the cost function is specified as a multivariate Fourier expansion, the system of
demand equations can be defined by Shephard’s lemma.
Estimation of Cost Systems

Regardless of the function form, cost functions are typically estimated as systems of equations
using Seemingly Unrelated Regression, Iterated Seemingly Unrelated Regression, or Maximum
Likelihood.

I prefer the use of Maximum Likelihood based on a concentrated likelihood function.

Q4. what are the prerequisites of capital budgeting ? how can we determine the optimum level of
investment under uncertainty?
Ans.
Some of the important prerequisites of capital budgeting is as follows:

(a) Determining Capital Expenditure:

Refers to the fact that capital plays an important role in selecting any investment project. An
organization needs to define its total capital expenditure for purchasing various fixed assets. In
capital budgeting, only long-term capital expenditure is taken into account, which is not
adjustable in long term, whereas short-term capital expenditures, such as inventories and
receivables, are not considered.

However, the long-term capital expenditures of an organization vary with projects of different
time durations. Usually, capital expenditures having a duration of one year are taken into account
for capital budgeting.

An ideal long-term capital expenditure involves the following:

i. New capital equipment

ii. Long-term assets

iii. Expansion or diversification of assets

iv. Replacement expenditure

v. Advertisement expenditure

vi. Research and development expenditure


(b) Determining the Planning Period:

Refers to one of the most important aspects of capital budgeting. The duration of any capital
expenditure include a high degree of risks. Therefore, an organization needs to clearly define the
planning period of capital expenditure.

A well-defined period of capital expenditure is necessary for the following purposes:

i. Performing various operative functions, such as planning, executing, and controlling

ii. Merging new plans with old ones for the growth and development of the organization

iii. Assessing the size of plant and economies of scale

iv. Acquiring funds on time and managing finance of the organization

(c) Selecting Decision Rules:

Refers to the condition required for accepting or rejecting a project. Decision rules are generally
selected on the basis of the goals of an organization, such as profit maximization and
minimization of cost. Therefore, before deciding criteria for decision rule, it is required to define
the objectives of investment and then selecting criteria for the evaluation of project.

The following methods are used for evaluating projects:

i. Payback period

ii. Net present value

iii. Internal rate of return

price uncertainty on the investment decision of a risk-neutral competitive firm which faces
convex costs of adjustment.' This issue has been analyzed by Richard Hartman (1972) and by
Robert Pindyck (1982), but they reached dramatically different results. Hart- man showed that
with a linearly homogeneous production function, increased output price uncertainty leads the
competitive firm to increase its investment. However, Pindyck found increased output price
uncertainty leads to increased investment only if the marginal adjustment cost function is
convex; but, if the marginal adjustment cost function is concave, then increased uncertainty will
reduce the rate of investment. Pindyck argues that his results differ from Hartman's results
because of a different stochastic specification of the price of output. In Hartman's discrete time
model, price is random in each period including the current period, whereas in Pindyck's
continuous-time model, the current price is known but the future evolution of prices is stochastic.
In this paper, I demonstrate that Hartman's results continue to hold using Pindyck's stochastic
specification and that Pindyck's analysis applies to a so called "target" rate of investment, which
in general is not optimal.
Q2 explain keynessian theory of national income determination model with help of suitable
diagram?
Ans: ​According to Keynes, there can be different sources of national income, such as
government, foreign trade, individuals, businesses and trusts.
For determining national income, Keynes had divided the different sources of income into four
sectors namely’ household sector, business sector, government sector, and foreign sector.

The two-sector model of economy involves households and businesses only, while three-sector
model represents households businesses, and government. On the other hand, the four-sector
model contains households, businesses, government, and foreign sector.Let us discuss these three
types of models of income determination given by Keynes.

Determination of National Income in Two-Sector Economy:

The determination of level of national income in the two-sector economy is based on an


assumption that two-sector economy is an economy where there is no intervention of the
government and foreign trade.

Apart from this, an economy can be a two-sector economy if it satisfies the following
assumptions:

a. Comprises only two sectors, namely, households and businesses. The households are the
owners of factors of production and provide factor services to businesses to earn their livelihood
in the form of wages, rents, interest, and profits. In addition the households are the consumers of
final goods and services produced by businesses. On the other hand, businesses purchase factor
services from households to produce goods and services and sell it to households.

b. Does not have government interference. If government is there, it does not have any role to
play in the economic activity of a country. For example, in the two-sector economy, the
government is not involved in activities, such as taxation, expenditure, and consumption.

c. Comprises a closed economy in which the foreign trade does not exist. In other words, import
and export services are absent in such an economy.

d. Contains no profit that is undistributed or savings by the organization. In other words, the
profit earned by an organization is completely distributed in the form of dividends among
shareholders.

e. Keeps the prices of goods and services, supply of factors of production, and production
technique constant throughout the life cycle of organization.

Keynes believed that there are two major factors that determine the national income of a country.
These two factors are Aggregate Supply (AS) and Aggregate Demand (AD) of goods and
services.
In addition, he believed that the equilibrium level of national income can be estimated when
AD=AS. Before representing the relationship between AS and AD on a graph, let us understand
these two concepts in detail.

5. "It is believed that a firm under a perfect competition is a price-taker and not a
price-maker." Explain giving examples.

ANSWERE-

Perfect competition describes a market structure where competition is at its greatest possible
level. To make it more clear, a market which exhibits the following characteristics in its structure
is said to show perfect competition:

1. Large number of buyers and sellers

2. Homogenous product is produced by every firm

3. Free entry and exit of firms

4. Zero advertising cost

5. Consumers have perfect knowledge about the market and are well aware of any changes in the
market. Consumers indulge in rational decision making.
6. All the factors of production, viz. labour, capital, etc, have perfect mobility in the market and
are not hindered by any market factors or market forces.

7. No government intervention

8. No transportation costs

9. Each firm earns normal profits and no firms can earn super-normal profits.

10. Every firm is a price taker. It takes the price as decided by the forces of demand and supply.
No firm can influence the price of the product.

In perfect market conditions (also called perfect competition) a firm is a price taker because
other firms can enter the market easily and produce a product that is indistinguishable from every
other firm’s product. This makes it impossible for any firm to set its own prices.

A price taker is a firm that cannot have any say in setting its own prices. A price taker simply
has to accept the market price. This is in contrast to a price maker, which can have an influence
over the price at which it sells its goods.

In perfect competition, there are two main reasons why a firm cannot get away with setting its
prices above the market price. First, there is no difference between its product and that of every
other firm in the market. Therefore, no one will pay extra for a firm’s product the way that they
might pay extra for something like Nike shoes. Second, if a firm were to succeed in setting a
higher price, more firms would enter the market, attracted by the higher profits that were
available. This would increase supply and drive down the price of the firm’s product.

In perfect competition, firms sell homogeneous products and it is easy for a firm to enter the
market. These two factors make it impossible for firms to set their prices above the market price.
This makes them into price takers.

A Perfectly Competitive Market is one in which the number of sellers is large and all of them are
producing homogeneous goods, and there is no price competition.

A price is set by the industry and each firm acts as a price taker, this happens because all firms
are producing homogeneous goods due to which they cannot set different prices, because if a
firm sets different prices the consumer will shift towards another firm.

(i) A firm under perfect competition is contributing such a small fragment to the market supply
that total supply schedule remains unaffected by any change in individual firm's supply.

(ii) All firms are selling homogeneous product. Accordingly, even partial control over price
is not possible.

(iii) If any firm tries to fix- its own price, it won't succeed. Higher price would drive the
buyers to a large number of other sellers. Lower price would bring so many buyers to a firm that
it cannot cope with the demand.
6. What are the various factors which may influence the demand for intermediate goods like
cables? Explain the most appropriate method of forecasting the demand for such an item.

ANSWER-

Demand for a commodity increases or decreases due to a number of factors.

The various factors affecting demand are discussed below:

1. Price of the Given Commodity:

It is the most important factor affecting demand for the given commodity. Generally, there exists
an inverse relationship between price and quantity demanded. It means, as price increases,
quantity demanded falls due to decrease in the satisfaction level of consumers.

Micro Economics

For example, If price of given commodity (say, tea) increases, its quantity demanded will fall as
satisfaction derived from tea will fall due to rise in its price.

Demand (D) is a function of price (P) and can be expressed as: D = f (P). The inverse
relationship between price and demand, known as ‘Law of Demand’, is discussed in Section 3.7.

The following determinants are termed as ‘other factors’ or factors other than price’.
2. Price of Related Goods:

Demand for the given commodity is also affected by change in prices of the related goods.
Related goods are of two types:

(i) Substitute Goods:

Substitute goods are those goods which can be used in place of one another for satisfaction of a
particular want, like tea and coffee. An increase in the price of substitute leads to an increase in
the demand for given commodity and vice-versa. For example, if price of a substitute good (say,
coffee) increases, then demand for given commodity (say, tea) will rise as tea will become
relatively cheaper in comparison to coffee. So, demand for a given commodity is directly
affected by change in price of substitute goods.

(ii) Complementary Goods:

Complementary goods are those goods which are used together to satisfy a particular want, like
tea and sugar. An increase in the price of complementary good leads to a decrease in the demand
for given commodity and vice-versa. For example, if price of a complementary good (say, sugar)
increases, then demand for given commodity (say, tea) will fall as it will be relatively costlier to
use both the goods together. So, demand for a given commodity is inversely affected by change
in price of complementary goods.

Examples of Substitute and Complementary Goods:


Substitute Goods

1. Tea and Coffee 2. Coke and Pepsi 3. Pen and Pencil

4. CD and DVD 5. Ink pen and Ball Pen 6. Rice and Wheat

Complementary Goods:

1. Tea and Sugar 2. Pen and Ink 3. Car and Petrol

4. Bread and Butter 5. Pen and Refill 6. Brick and Cement

For detailed discussion on substitute goods and complementary goods, refer Section 3.11.

3. Income of the Consumer:


Demand for a commodity is also affected by income of the consumer. However, the effect of
change in income on demand depends on the nature of the commodity under consideration.

a) If the given commodity is a normal good, then an increase in income leads to rise in its
demand, while a decrease in income reduces the demand.

b) If the given commodity is an inferior good, then an increase in income reduces the
demand, while a decrease in income leads to rise in demand.

Example:

Suppose, income of a consumer increases. As a result, the consumer reduces consumption of


toned milk and increases consumption of full cream milk. In this case, ‘Toned Milk’ is an
inferior good for the consumer and ‘Full Cream Milk’ is a normal good. For detailed discussion
on normal goods and inferior goods, refer Section 3.12.

4. Tastes and Preferences:

Tastes and preferences of the consumer directly influence the demand for a commodity. They
include changes in fashion, customs, habits, etc. If a commodity is in fashion or is preferred by
the consumers, then demand for such a commodity rises. On the other hand, demand for a
commodity falls, if the consumers have no taste for that commodity.
5. Expectation of Change in the Price in Future:

If the price of a certain commodity is expected to increase in near future, then people will buy
more of that commodity than what they normally buy. There exists a direct relationship between
expectation of change in the prices in future and change in demand in the current period. For
example, if the price of petrol is expected to rise in future, its present demand will increase.

7. State the assumption underlying the economists' theory of firm. Develop a critique of the
theory and suggest the need for alternative models.

Ans.

The basic problem of economics can be summarized in one sentence: How to best satisfy
unlimited wants with unlimited resources.

We can break this problem into two parts:

Preferences -- What we like and what we dislike.

Resources -- We all have limited resources. Even Warren Buffett and Bill Gates have limited
resources. They have the same 24 hours in a day that we do and neither is going to live forever.

MACROECONOMICS
All of economics, including both microeconomics and macroeconomics, come back to this basic
problem on how to use limited resources to satisfy our preferences and unlimited wants.

RATIONAL MAXIMIZING BEHAVIOR

In order to simply model how humans attempt to do this, we need a basic behavioral assumption.
The assumption is that people attempt to maximize outcomes (that is, to do as well as possible
for themselves) as defined by their preferences, given their resource constraints.

Economists refer to people who do this as exhibiting 'rational maximizing behavior'. Note that in
more complex economic models that this assumption can be weakened but at a cost of added
complexity.

This 'rational maximizing behavior' assumption does not necessarily mean that people make ex
ante perfect decisions. People may be limited by the amount of information they have (e.g. 'it
seemed like a good idea at the time!').

As well, 'rational maximizing behavior' says nothing about the quality or nature of people's
preferences (But I enjoy hitting myself on the head with a hammer!')

TRADEOFFS -- YOU GET WHAT YOU GIVE

The struggle between preferences and constraints means that economists must, at their core, deal
with the problem of tradeoffs.
In order to get something, we must use up some of our resources.

Examples: You give up $20 to obtain the new best seller from Amazon.com. I give up 3 hours of
time to watch the Blue Jays game on T.V. (‘that’s three hours of my life I will never get back!')

Anything obtained has a cost. Economists have a saying for this -- "There is no such thing as a
free lunch!"

Critiques of the Rational Model

Critics of rational choice theory—or the rational model of decision-making—claim that this
model makes unrealistic and over-simplified assumptions. Their objections to the rational model
include:

People rarely have full (or perfect) information. For example, the information might not be
available, the person might not be able to access it, or it might take too much time or too many
resources to acquire. More complex models rely on probability in order to describe outcomes
rather than the assumption that a person will always know all outcomes.

Individual rationality is limited by their ability to conduct analysis and think through competing
alternatives. The more complex a decision, the greater the limits are to making completely
rational choices.
Rather than always seeking to optimize benefits while minimizing costs, people are often willing
to choose an acceptable option rather than the optimal one. This is especially true when it is
difficult to precisely measure and assess factors among the selection criteria.

Alternative Theories of Decision-Making

Prospect Theory

Alternative theories of how people make decisions include Amos Tversky's and Daniel
Kahneman's prospect theory. Prospect theory reflects the empirical finding that, contrary to
rational choice theory, people fear losses more than they value gains, so they weigh the
probabilities of negative outcomes more heavily than their actual potential cost. For instance,
Tversky's and Kahneman's studies suggest that people would rather accept a deal that offers a
50% probability of gaining $2 over one that has a 50% probability of losing $1.

Bounded Rationality

Other researchers in the field of behavioral economics have also tried to explain why human
behavior often goes against pure economic rationality. The theory of bounded rationality holds
that an individual's rationality is limited by the information they have, the cognitive limitations
of their minds, and the finite amount of time they have to make a decision. This theory was
proposed by Herbert A. Simon as a more holistic way of understanding decision-making.
Bounded rationality shares the view that decision-making is a fully rational process; however, it
adds the condition that people act on the basis of limited information. Because decision-makers
lack the ability and resources to arrive at the optimal solution, they instead apply their rationality
to a set of choices that have already been narrowed down by the absence of complete information
and resources.
8. 'Price leadership is an alternative cooperative method used to avoid tough competition'.

ANSWERE-

Price leadership is an observation made of oligopolic business behavior in which one company,
usually the dominant competitor among several leads the way in determining prices, the others
soon following. When business conditions permit, the price leader will raise prices with the
expectation that the others will follow. The practice of price leadership prevails in many
industries: automobiles, breakfast cereals, beer, steel and bank loans are among the many goods
and services that are usually priced in this manner.

On the surface, it looks as though the effect of price leadership is the same as the effect of the
fixing of prices by a cartel or a trust. But there is a fundamental difference. The trust or cartel
assigns production quotas to its members in order to keep production down. Competition does
not exist in any form. Oligopolies that follow a price leader do not engage in price competition,
but they still contest for market share with a variety of forms of non-price competition. Pepsi and
Coke each spend billions on TV ads designed to entice the consumer to switch cola brands, but
those expensive ads never mention price. An example. Company A has two principal
competitors and a number of smaller niche players. The niche players serve focused geographic
areas or offer specialty products or services to unique segments of the customer base. Company
A is a premium priced producer and is a price leader. Each time it raises or lowers prices, its
competitors also make a price move. If Company B has a current price which is 90% of
Company A, then when Company A lowers its price Company B will adjust proportionally. Thus
Company B will still have a price 90% of Company A. The reverse occurs when Company A
takes a price increase.

PRICE LEADERSHIP- AVOIDING TOUGH PRICE COMPETITION

Price Leadership: Changing a price is always a dangerous practice for an oligopoly. If the firm
lowers the price, its competitors are also likely to lower theirs, then all will suffer from lower
profits. On the other hand, raising prices may lead to a loss of market share unless competitors
also raise their prices. In many industries, one firm (usually the largest) is accepted by the others
as the price leader. The price leader will be the first to adjust prices to new conditions (higher
labor costs, lower raw materials costs, etc.) and the others will fall into line. Of course this
arrangement is entirely informal and unwritten - since any actual agreement to follow such a
practice would violate the antitrust laws.

Price Setting: But on what basis does the price leader set prices? The interplay of supply and
demand forces so beloved by neoclassical economists loses its cogency as a theory of prices
when

1) There is too little competition to force prices to equal marginal costs and marginal benefits
through a Darwinian struggle;

2) Costs per unit fall with increased output as the firm is able to spread its fixed costs over a
larger number of units; and

3) Demand is itself subject to manipulation by the firm through advertising.

Institutionalists and Post-Keynesians claim that the structure of oligopoly leads to a form of
administered pricing. The price leader will use its average cost as a basis for price setting. Prices
will be set at a level which achieves a target level of operating profits. This target level will be
sufficient to enable the firm to self-finance expansion without the need to issue new stock or to
borrow amounts of money that might threaten the firm's independence. Rather than constantly
move the price up and down in order to sell as many units as the firm is capable of producing at a
profit, the firm will normally adjust its output in order to maintain its target price. When this
form of administered pricing is combined with the normal tendency of firms to expand, one
result is that firms will usually have more production capacity than they use. When demand
increases, they will increase output rather than prices. Enforcement of Price Leadership In the
economic as in the political arena, leadership will occasionally be challenged. Sometimes the
mere threat of lowering prices suffices.

In 1989, Miller and Coors tried to expand their market shares by discounting their premium
beers. Anheuser-Busch, with 41 percent of the U.S. beer market, simply issued a press release:

We cannot permit a further slowing in our volume trend... [the company will take] appropriate
competitive pricing actions to support our long-term market share growth strategy. Actually the
Anheuser statement is simply a warning to competitors that if they do not stop discounting they
will face a costly battle which they will certainly lose. But sometimes actual price reductions are
necessary. In the late 1960s, Chrysler tired to break away from the pricing pattern then set by
General Motors. But GM was a larger and more efficient producer and was not about to abandon
its price-leader role. So GM lowered prices below the cost of production in those lines in which
Chrysler was competitive. Thus Chrysler was left with a lower margin per vehicle sold but was
still unable to expand its market share. When GM raised their prices again in 1971 Chrysler
quickly followed. Chrysler also filed suit charging GM (and Ford) with predatory pricing, but
Ford and GM were acquitted due to lack of evidence. Predatory Pricing A large or diverse firm
that can stand temporary losses can cut its prices below the cost of production until it runs
competitors out of business or establishes its price leadership. Then it can raise prices again. This
is illegal. But it is very hard to prove, since normal competitive pressures can lead to prices set
temporarily below the cost of production.

SOME PRICE LEADERSHIP MODELS

Barometric price leadership: The barometric price leadership is not a dominant firm. One firm in
a group of firms of more or less comparable size comes to pay the role of price leader, not
because of its dominant position in the market but because other firms regard its actions as a
suitable barometer of changing market conditions. They are willing to follow its policies in the
belief that competitive disturbances are minimized by so doing. But the actual powers of a
barometric price leader are greatly restricted by its realization that other firms will follow him
only with reasonable limits.

Two simplified models of price leadership:

Suppose there are only firms in the industry and they have assigned half of the market.
Furthermore, suppose that they are producing homogeneous products, but one firm has lower
costs that the other. The price and output policies of the two firms differ because of the different
cost of the two firms. This can be illustrated with the following figure:

DD presents the market demand curve and dd represents the firms demand curve. The firm 1
fixes the price at op 1 and its level of output is OM1 whereas the firm 2 fixes its price at op 2 and
produces OM2 amount of output. The firm 2 has comparatively advantageous position than the
firm 1 because of its lower cost. Hence, the firm 2 becomes the price leader.
Section-B
____________________________________________________

CASE STUDY
Question 1: What are the different methods of measuring national income? Which methods
have been followed in India?

Answer:
National Income is the money value of all goods and services produced in a country during a
year. National income shows the economic position of a nation. The basic objective of an
economy is to achieve economic progress. This is achieved by coordinating natural resources,
human resources, capital, technology etc. National income will help to assess and compare the
progress achieved by a country over a period of time.

We can calculate national income by following any one of these methods. Considering the nature
and requirements of the economy one or more methods are used for calculating national income.
The three methods used for calculating national income are:

A Production method
B Income method
C Expenditure method

A. Production method

This method is based on the total production of a country during a year. First of all production
units are classified into primary, secondary and tertiary sectors. Then we identify the various
units that come under these sectors. We estimate the goods and services produced in each of
these sectors. The sum total of products produced in these three sectors is the total output of the
nation. The next step is to find out the value of these products in terms of money. The money
sent by Indian citizens working abroad is also added to this. Now we get the gross national
income.

GNI = Money value of total goods and services + Income from abroad.

This method helps us to find out contributions of various sectors to national income.

B. Income Method
Factors of production together produce output and income. The income received by the factors of
production during a year can be obtained by adding rent to land, wages to labour, interest to
capital and profit to organisations. This will be equal to the income of the nation. In other words,
total income is equal to the reward given to various factors of production. By adding the money
sent by the Indian citizens from abroad to the income of the various factors of production, we get
the gross national income.

GNI = Rent + Wage + Interest + Profit + Income from abroad.

This method will help us to know the contributions made by different agents like Landlords,
labourers, capitalists and organizers to national income.

C. Expenditure Method

National income can also be calculated by adding up the expenditure incurred for goods and
services. Government as well as private individuals spend money for consumption and
production purposes. The sum total of expenditure incurred in a country during a year will be
equal to national income.

GNI = Individual Expenditure + Government Expenditure.

This method will help us to identify the expenditure incurred by different agents.

Any one of the above methods can be used for calculating national income.

Production method = Income method = Expenditure method.

Today national income in India is calculated and published by the Central Statistical
Organization. All the three methods are used for calculating national income in India.

Question 2: What do you understand by the investment multiplier? In what way does it
defend the policy of public works on the part of the state during business depression?

Answer:
Investment multiplier is simply the multiplier effect of an injection of investment into an
economy. In general, a multiplier shows how a sum injected into an economy travels and
generates more output.

For example if you buy $100 worth of chips. Say the stall owner saves $10 and spends $90 on
burgers. Then the burger stall owner saves 10% that is $9 and consumes the rest ($81) on cheese,
and so on... Each $ received 10% is saved (marginal propensity to save- MPS) and 90%
consumed (marginal propensity to consume - MPC). This eventually results in 100/(1-.9)=$1000
worth of expenditure in the economy. The multiplier=1/(1-MPC).
The investment multiplier is simply the same idea applied to an increase in investment as
opposed to consumption above. Using consumption just makes it easier to understand;
investment multiplier is just the same.

Question 3: Discuss the various phases of business cycle:


(i) Are cyclical fluctuations necessary for economic growth?

Answer:
The business cycle or economic cycle refers to the ups and downs seen somewhat simultaneously
in most parts of an economy. The cycle involves shifts over time between periods of relatively
rapid growth of output (recovery and prosperity), alternating with periods of relative stagnation
or decline (contraction or recession). These fluctuations are often measured using the real gross
domestic product. To call those alternances "cycles" is rather misleading, as they don't tend to
repeat at fairly regular time intervals. Most observers find that their lengths (from peak to peak,
or from trough to trough) vary, so that cycles are not mechanical in their regularity.

Business cycles are a type of fluctuation found in the aggregate economic activity of nations that
organize their work mainly in business enterprises: a cycle consists of expansions occurring at
about the same time in many economic activities, followed by similarly general recessions,
contractions and revivals which merge in the expansion phase of the next cycle; this sequence of
changes is recurrent but not periodic; in duration business cycles vary from more than one year
to ten or twelve years; they are not divisible into shorter cycles of similar character with
amplitudes approximately their own. (Burns and Mitchell)

Actually there are six phases of business cycles counting the peak points (when booms turns into
burst and the troughs).Business cycles are the recurring rises and falls in overall economic
activity as reflected in production, employment, profits, prices, wages, and other macroeconomic
series. Business cycles are recurring, but non-periodic, and one cycle must be more than a year,
otherwise, it would be considered a seasonal cycle. Business cycles reflect the inability of the
marketplace to accommodate smoothly such factors as new technologies, changing needs for
occupational skills, shifting markets for new and substitute products, and risks in business
investments. Business cycles can also reflect shortages and high prices created by external
shocks such as war, cutbacks in oil production by the Organization of Petroleum Exporting
Countries (OPEC), bad harvests, and natural disasters.

Six phases of the business cycle are illustrated: Peak, Recession, Contraction, Trough, Recovery,
and Expansion. One complete cycle can be measured from Peak-to-Peak or Trough-to-Trough.
Beginning with the peak, the six phases of the business cycle unfold as follows. The Peak is the
high point of continuous expansion just before the downturn in economic activity. This is
followed by a Recession, the immediate downturn in economic activity after the peak in the
business cycle, and represents the downward region of the cycle from the peak to the trough. If
overall activity falls below the lowest level (i.e., trough) of the previous recession (lower
horizontal dotted line on the graph), then this more severe decline may be referred to as
Contraction.

The Trough is the lowest point of the recession phase of the business cycle just before economic
activity turns upward. Once economic activity turns upward, the economy is then in Recovery.
This phase of the business cycle immediately follows the trough, and is characterized by the
continuous expansion of economic activity. The economy is in Expansion when overall activity
in the recovery phase exceeds the peak of the previous business cycle (upper horizontal dotted
line).

Sometimes during the upward phase of the business cycle, the expanding economy may not be
increasing production fast enough to absorb those entering the labor market, and may even result
in some of those already employed being laid off. That is, overall production and unemployment
rates are both rising. This situation is known as a growth recession. If the opposite occurs, that is,
if the economy expands beyond the long- run sustainable growth rate for a significant period of
time, then this period may be characterized as a boom. Booms are usually followed by a
recession. There may be obvious signs that the economy is in a recession, such as slack business
activity and a rising unemployment rate. We may be affected personally, by having our
workweeks reduced or even by losing our jobs. But there is also a set of observable
measurements of a recession. These measurements provide the criteria for clearly defining dates
for the peak and the trough.

(ii) Suggest appropriate fiscal and monetary policies for depression.


Fiscal policy focuses on controlling the government spending in order to accelerate or retard
economic growth. During depression, an expansionary fiscal policy should be used which aims
at increasing government spending to directly increase investment, employment and thus
domestic demand. Expansionary fiscal policy is either financed through borrowing or tax
increase.

Monetary policy aims at controlling the liquidity in the market to spur up or slow down the
economy. It is implemented primarily through interest rate control (though there are other ways
as well). Usually the central bank is responsible for fixing the interest rate. During depression, an
expansionary monetary policy is implemented (easy money) which aims to bring down interest
rate or make money cheaper. It is then expected that with cheaper money, the private sector will
increase production, which will increase employment and then demand.

CASE STUDY
In early 1991, there was a sharp increase in the price of newsprint, the paper used by the
newspapers. Since newsprint is the largest expense for India newspapers (after salaries)
publishers were concerned about the price hike. Suppose that the demand for newsprint can be
represented as follows:

Qi = 17-3 – 0-0092 p +0-0067


Where Q. equals the quantity demanded (in kilograms per capital), P is the price of newsprint (in
Rs. Per metric ton) and I is the income per capita (in Rs.),

Question

Q1. If there are 1 million people in the market, and if per capita income equals Rs. 10,000 what
is the demand curve for newsprint?

Ans.

Reduction in grammage, i.e. thickness of the paper: As I have mentioned earlier, there was no
choice for newspapers but to purchase newsprint. It was licensed to raj and indigenous mills they
used to produce newsprint with more thickness i. e. 52 to 53 grammage. It was not even and used
to vary every six inches, lengthwise as well as widthwise. In 1982 to 86 all the newspapers
shifted themselves form letterpress to offset technology. Offset technology helped them for
better printing, photo compose and mainly it became possible for them to use newsprint with less
grammage or thickness. As newsprint became import free item, Newspapers started importing
fine or thin newsprint up to 45 grammage. Even indigenous mills with new machinery started
manufacturing newsprint with 45 gsm. This helped newspapers to get additional length or copies
in same weight. Which resulted to save newsprint cost up to 12%. That was the major advantage
for all the newspapers. b) Reduction in Width: Standard size width of a newspaper (for two
pages) use to be 33.5". Few newspapers like Times of India having double width machinery used
to have 67" width reels. As the newsprint prices started rising, Newspapers started demanding
the 32" width newsprint. Few new mills introduced 32" width paper (118) and even big
newspapers changed their policy to have 32" width. The reduction in width was continued, and
Times of India came with an advertisement of their product with words: "It is easy to handle."
And all the newspapers started using newsprint with 30" width. Whereas few newspapers started
using 28" width. This helped all the newspapers to save expenditure on newsprint up to 9.5%.
(3% for 1") c) Height of a newspaper Height of a newspaper depends on the cutoff size of a
printing press. It cannot be changed overnight. But looking at price rise in newsprint every
management started thinking of it. Whenever they get chance to change the machine which is
costly investment they thought of returns by saving newsprint cost which will be long term
saving. Whenever they change the machine, it went for 560 m.m. cutoff from traditional 579
cutoff which saved 3% of newsprint cost to pay back their investment. Whereas smaller
newspapers purchase machines with 545 m.m. cutoff which saved additional 4% of their
newsprint cost. Few newspaper purchased 508 m.m. cut of which helped them to reduce
newsprint cost up to 7%. In short journey from 579 to 508 m.m. saved the newsprint cost by
14%.

Q 2. Under these circumstances, what is the price elasticity of demand if the price of newsprint
equals Rs. 400 per metric ton?

Ans.
Consider Q = 400 – 4P
Þ 4P = 400 – Q Þ
P = 100 – 0.25Q
Multiplying by Q and taking the first derivative yields:
TR = P.Q
TR = (100 – .25Q)Q
TR = 100Q – 0.25Q2
MR = dTR/dQ
MR = 100 – 0.5Q
The total revenue and marginal revenue functions along with the demand curve, are plotted in
Figure 5.1. Notice that the slope of the marginal revenue function is twice the slope of the
demand function. You see in Figure 5.1 that demand is price elastic over the range of quantities
for which marginal revenue is positive. Because marginal revenue is the slope of total revenue
(remember that MR = dTR/dQ), you can tell that increasing sales by lowering price will cause
total revenue to rise over this interval. However, lowering price when demand is inelastic
(beyond Q = 200) will result in reduced revenues. As is evident from the above discussion, the
change in expenditure when price changes is related to the elasticity of demand. If elasticity is
less than unity (inelastic), the percentage change in price can exceed the percentage change in
quantity. The price change will then be the dominant one of the two changes and the revenue will
change in the same direction as the price change. If however, elasticity exceeds unity (elastic),
the percentage change in quantity will exceed the percentage change in price.

Q3. According to a study, the demand curve for newsprint in India is:
Q2 = 2672 -- 0-51 p
Where, Q2 is the number of metrix tons of newsprint demanded (in thousand). What is the price
elasticity of demand for newsprint in India if price equals Rs. 500 per metric ton?

Ans.

For instance, a ton of newspaper can be recycled using a technique that yields 0.85 tons of
newsprint pulp. Alternatively, a ton of newspaper can be recycled using a technique that yields
0.80 tons of packaging paper. Similarly, a ton of cardboard can be recycled to yield 0.80 tons of
newsprint or 0.70 tons of pack aging paper pulp. Note that newspaper and cardboard cannot be
converted to print stock pulp using the techniques available to the recycler. The cost of
processing each ton of raw material into the various types of pulp is summarized in the following
table along with the amount of each of the four raw materials that can be purchased and their
costs. The recycler wants to determine the least costly way of producing 500 tons of newsprint
pulp, 600 tons of packaging paper pulp, and 300 tons of print stock quality pulp.

Case Study

Venezuela's economy is in freefall. Hyperinflation, power EDL 102 Economics tor Managers Revisit cuts, and
food and medicine shortages are driving 12 Jun 2019 FSL Enter your response 2 1 of 2 @ All millions of
Venezuelans out of the country Many are blaming President Nicoläs Maduro and his government for the dire
state the nation is in. Hyperinflation is very high and typically accelerating inflation. It quickly erodes the real
value of the currency, as the prices of most or all goods increase. This causes people to minimize their holdings
in that currency as they usually switch to more stable foreign currencies. A socioeconomic and political crisis
has been taking place in Venezuela since 2010 under the presidency of Hugo Chavez and has continued into
the current presidency of Nicolas Maduro. The current situation is the worst economic crisis in Venezuela's
history and among one of the worst crises experienced in the Americas. The crisis was the result of populist
policies that began under the Chévez administration's Bolivarian Revolution. On 2 June 2010, President
Chévez declared an "economic war" due to the increasing shortages in Venezuela. The crisis intensified under
the Maduro government, growing more severe as a result of low oil prices in early 2015. The Venezuelan
government has declined that there is a crisis within the country.

Q1. ​What measures can be taken by the government of the country to cure inflation in the economy?

Ans: ​Inflation occurs when an economy grows due to increased spending. When this happens,
prices rise and the currency within the economy is worth less than it was before; the currency
essentially won’t buy as much as it would before. When a currency is worth less, its ​exchange
rate​ weakens when compared to other currencies.
There are many methods used to control inflation; some work well while others may have
damaging effects. For example, controlling inflation through wage and ​price controls can cause a
recession​ and cause job losses.

Inflation is generally controlled by the Central Bank and/or the government. The main policy
used is monetary policy – set by Central Banks. However, in theory, there are a variety of tools
to control inflation including:

Monetary policy – Setting interest rates. Higher interest rates reduce demand, leading to lower
economic growth and lower inflation. In a period of rapid economic growth, demand in the
economy could be growing faster than its capacity to meet it. This leads to inflationary pressures
as firms respond to shortages by putting up the price. We can term this ​demand-pull inflation​.
Therefore, reducing the growth of aggregate demand (AD) should reduce inflationary pressures.

Fiscal Policy

The government can increase taxes (such as income tax and VAT) and cut spending. This
improves the budget situation and helps to reduce demand in the economy.

Both these policies reduce inflation by reducing the growth of Aggregate Demand. If economic
growth is rapid, reducing the growth of AD can reduce inflationary pressures without causing a
recession.

If a country had high inflation and negative growth, then reducing aggregate demand would be
more unpalatable as reducing inflation would lead to lower output and higher unemployment.
They could still reduce inflation, but it would be much more damaging to the economy.

Wage Control
If inflation is caused by wage inflation (e.g. powerful unions bargaining for higher real wages),
then limiting wage growth can help to moderate inflation. Lower wage growth helps to reduce
cost-push inflation and helps to moderate demand-pull inflation.

However, as the UK discovered in the 1970s, it can be difficult to control inflation through
incomes policies, especially if the unions are powerful.

Monetarism

Monetarism seeks to control inflation by controlling the money supply. Monetarists believe there
is a strong link between the money supply and inflation. If you can control the growth of the
money supply, then you should be able to bring inflation under control. Monetarists would stress
policies such as:

● Higher interest rates (tightening monetary policy)


● Reducing budget deficit (deflationary fiscal policy)
● Control of money being created by the government

However, in practice, the link between ​money supply and inflation​ is less strong.

Supply Side Policies

Often inflation is caused by persistent uncompetitiveness and rising costs. Supply-side policies
may enable the economy to become more competitive and help to moderate inflationary
pressures. For example, more flexible labour markets may help reduce inflationary pressure.

However, ​supply-side policies can take a long time, and cannot deal with inflation caused by
rising demand.

Ways To Reduce Hyperinflation – change currency

In a period of hyperinflation, conventional policies may be unsuitable. Expectations of future


inflation may be hard to change. When people have lost confidence in a currency, it may be
necessary to introduce a new currency or use another like the dollar (e.g. ​Zimbabwe
hyperinflation​).

Ways to Reduce Cost-Push Inflation

Cost-push inflation (e.g. rising oil prices can lead to inflation and lower growth. This is the worst
of both worlds and is more difficult to control without leading to lower growth.

Case Study
Appalachia beverage company Inc is considering alternative proposals for expansion into midwest
,alternative construct a single plant in indianapolis .Indiana with a monthly production capacity of
300,000 cases a monthly fixed cost of 262500 and variable cost of $3.25 per case
Alternative 2
Construct three plants one each in mancie indiana .normal and dayton , Ohio with capacities of 120,000 ,
100,000 and 80,000 respectively and monthly fixed cost of rs 120,000$ , $110,000 and $95000 each
.variable cost would be only $3 percase because of lower distribution costs.to achieve these cost savings
sales from each smaller plant would be limited to demand within its homerate.the total estimated monthly
sales volume of 200,000 cases in these three midwestern states is distributed as follows 80,000 cases in
Indiana , 70000 cases in Illnis and 50000 cases in okhio

Q1. assuming a wholesale price of $5 per case, calculate break even output quantities of each
alternative?
Ans
The breakeven output quantity for the single plant alternative is:
Q = ​TFC
P-AVC
= ​$262,500
$ 5- $3.25
= 1, 50, 000 cases per month
The breakeven output quantities for the single plant alternative is:
Q muncic = ​$1,20,000
$5-$3
= 60,000 cases per month
Q normal = ​ $110,000
$5-$3
= 55,000 cases per month
Q dayton = ​ $95,000
$5-$3
= 47,500 cases per month
Thus, the firm-level breakeven quantity for the multiple plant alternative is:
Q = 60,000+55000+47,500
= 162,500 case per month
Provide that demand was distributed among the states in amounts equal to the breakeven quantities for
each individual plant.

Q2. at a wholesale price of $5 percase in all states and assuming sales at the projected levels which
alternative expansion scheme provides?
Ans
Single plant alternative
Cost analysis and estimation
π = TR- TC
= P*Q-TFC-AVC*Q
= $5(200,000)-$262,500-$3.25(200,000)
$87,500
Multiple plant alternative:
π = π = TR- TC
= ​PHQ​-​TFC​M​- TFC​N- ​TFC​D-​AVCHQ
= $5(200,000)-$120,000-$110,000-$95000-$3 (200,000)
$75,000
Management would prefer the plant alternative because of its greater profitability.

Case Study
Electron Control, Inc., sells voltage regulators to other manufacturers, who then customize and
distribute the products to quality assurance labs for their sensitive test equipment. The yearly
volume of output is 15,000 units. The selling price and cost per unit are shown below:

Selling price $200


Costs:
Direct material $35
Direct labour 50
Variable overhead 25
Variable selling expenses 25
Fixed selling expenses 15 150
Unit profit before tax $ 50

Management is evaluating the alternative of performing the necessary customizing to allow


Electron Control to sell its output directly to Q/A labs for $275 per unit. Although no added
investment is required in productive facilities, additional processing costs are estimated as:

Direct labour$25 per unit


Variable overhead$15 per unit
Variable selling expenses $10 per unit
Fixed selling expenses $100,000 per year

Question:
1. Calculate the incremental profit Electron Control would earn by customizing its
instruments and marketing directly to end users.
Answer:
No of units produced annually =15000
Profit on selling price of 200 is 50 per unit.
So total profit earned is 15000x50= 7, 50,000

If they sell it @ 275 to Q/A labs then:

Added variable Expense=50 per unit

Total variable expense = 150 + 50 = 200 per unit.


Total profit per unit = SP – CP
= 275 – 200
= 75 per unit
Added profit per unit= Total profit – profit on SP of 200
= 75 – 50
=25

Total Profit=75X15000=11, 25,000

Net profit = Total profit – Fixed selling expenses.


Net profit =11, 25,000 - 1, 00,000 =10, 25,000.
Incremental profit= 10, 25,000- 7, 50,000= 2, 75,000.

CASE STUDY
What this answer means is that XYZ corporation has to produce and sell 50000 widgets in order to cover
their total expense , fixed and variable . At this level of sales, they will make no profit but will just break
even.
A firm uses a number of inputs to produce its output . if the firm varies the quantity of only one input ,
keeping the other input quantities unchanged , then the quantity of its output obtained varies with quantity
of the variable input.

1.Give an example and prepare a schedule to explain the above case


Ans-: While fixed costs, such as rent or other overhead, generally remain level, ​variable costs will
correlate with the number of products manufactured. Because average variable costs differ widely among
industries, comparisons are generally most meaningful among companies operating within the same
industry.
When analyzing a company's income statement, it should be remembered that rising costs are not
necessarily a troubling sign. Whenever sales rise, more units must first be produced (excluding the impact
of stronger pricing), which in turn means that variable production costs must also increase. Thus, for
revenues to climb, expenses must also rise accordingly.

2. Explain the law of production underlying in above situation


Ans-:In this formula, fixed costs are stated as a total -- the total fixed costs for the firm. Basically, this
means the total overhead for the firm. Price and variable costs, however, are stated as per unit costs - the
price for each product sold and the variable cost for that unit of the product. The denominator of the
equation, price minus variable costs, is called the contribution margin. In other words, this is the
amount, per unit of product sold, that the firm can contribute to pay in gits fixed costs.
XYZ Corporation has to produce and sell 50,000 widgets in order to cover their total expenses,fixed and
variable. At this level of sales, they will make no profit but will just breakeven.

CASE STUDY

DEMAND ANALYSIS?DEFINITION AND MEANING?

Find out the definition and meaning for demand analysis to discover the customer requests for a
product or service in a particular market. Demand is the amount of goods that consumers or
buyers are willing and capable to buy for a specific price in a specific time period while
everything else remains the same. Demand is an economic principle that describes the
willingness and desire of consumers to purchase specific goods or services at a specific Ceteris
Paribus (all other things being unchanged or constant).

Law of demand:

The law of demand describes the inverse relationship of price and the quantity demanded. all else
remaining constant. If the price is high. the quantity demand decreases and if the price decreases.
then the quantity demand will increase.

Change in related goods price:

Price change in one good it can change the demand of other related goods For example, if the
price of one good increases, more consumers will buy other relative goods or if the price of the
first good decreases, more consumers will buy it.

Change in consumer expects

Any change in consumer expectations affects the demand analy a consumer behavior expects his
or her income to rise in the ful existing demand will increase and if the consumer expects his or
to reduce in the future. the existing demand will decrease according.

Q1. What are the factors determining Elasticity of Demand?

Ans.
Ans- Nature of commodity:
Elasticity of demand of a commodity is influenced by its nature. A commodity for a person may
be a necessity, a comfort or a luxury.

i. When a commodity is a necessity like food grains, vegetables, medicines, etc., its demand is
generally inelastic as it is required for human survival and its demand does not fluctuate much
with change in price.

ii. When a commodity is a comfort like fan, refrigerator, etc., its demand is generally elastic as
consumer can postpone its consumption.

iii. When a commodity is a luxury like AC, DVD player, etc., its demand is generally more
elastic as compared to demand for comforts.

iv. The term ‘luxury’ is a relative term as any item (like AC), may be a luxury for a poor person
but a necessity for a rich person.

2. Availability of substitutes:
Demand for a commodity with large number of substitutes will be more elastic. The reason is
that even a small rise in its prices will induce the buyers to go for its substitutes. For example, a
rise in the price of Pepsi encourages buyers to buy Coke and vice-versa.

Thus, availability of close substitutes makes the demand sensitive to change in the prices. On the
other hand, commodities with few or no substitutes like wheat and salt have less price elasticity
of demand.

3. Income Level:
Elasticity of demand for any commodity is generally less for higher income level groups in
comparison to people with low incomes. It happens because rich people are not influenced much
by changes in the price of goods. But, poor people are highly affected by increase or decrease in
the price of goods. As a result, demand for lower income group is highly elastic.

4. Level of price:
Level of price also affects the price elasticity of demand. Costly goods like laptop, Plasma TV,
etc. have highly elastic demand as their demand is very sensitive to changes in their prices.
However, demand for inexpensive goods like needle, match box, etc. is inelastic as change in
prices of such goods do not change their demand by a considerable amount.

Q2. Describe income and cross elasticity of demand.

Ans-The proportionate change in quantity demanded for a goods due to the proportionate change
in consumer's income is called income elasticity of demand. Income elasticity is usually
symbolized by 'Ey' and written as:
Ey = %ofchangeindemand%ofchangeinincome

Ey =ΔQQX100%ΔyyX100%

i.e. Ey=ΔQΔy X yQ
Where,

Ey = Income elasticity demand

Δ = Small Change

Q = Quantity

y = Income

DEGREE / TYPES OF INCOME ELASTICITY OF DEMAND

There are three types of income elasticity of demand. They are as follow:

1) Positive Income Elasticity (Ey>0)

If the quantity demand for a commodity increases with the increase in consumers income and
decreases with the decrease in income of the consumer is known as positive income elasticity.
Hence, there is a positive relation between income and demand. In that case, the value of
elasticity remains greater than zero. Under positive income elasticity of demand, there are three
types of demand. They are:

Income Elasticity of Demand is greater than unity (Ey>1)

If the percentage change in quantity demanded for a commodity is greater than percentage
change in income of the consumer, it is said to be income greater than unity. For example: When
the consumer’s income rises by 3% and the demand rises by 7%, it is the case of income
elasticity greater than unity.

In the given figure, quantity demanded and consumer’s income is measured along X-axis and
Y-axis respectively. The small rise in income from ​OY to ​OY​1 has caused greater rise in the
quantity demanded from ​OQ to ​OQ​1 and vice versa. Thus, the demand curve ​DD shows income
elasticity greater than unity.

● Income elasticity equal to unity (E​Y​ = 1)


If the percentage change in quantity demanded for a commodity is equal to percentage change in
income of the consumer, it is said to be income elasticity equal to unity. For example: When the
consumer’s income rises by 5% and the demand rises by 5%, it is the case of income elasticity
equal to unity.

In the given figure, quantity demanded and consumer’s income is measured along X-axis and
Y-axis respectively. The small rise in income from ​OY to ​OY​1 has caused equal rise in the
quantity demanded from ​OQ to ​OQ​1 and vice versa. Thus, the demand curve ​DD shows income
elasticity equal to unity.

● Income elasticity less than unity (E​Y​ < 1)

If the percentage change in quantity demanded for a commodity is less than percentage change in
income of the consumer, it is said to be income greater than unity. For example: When the
consumer’s income rises by 5% and the demand rises by 3%, it is the case of income elasticity
less than unity.

In the given figure, quantity demanded and consumer’s income is measured along X-axis and
Y-axis respectively. The small rise in income from ​OY to ​OY​1 has caused equal rise in the
quantity demanded from ​OQ to ​OQ​1 and vice versa. Thus, the demand curve ​DD shows income
elasticity equal to unity.

● Income elasticity less than unity (E​Y​ < 1)

If the percentage change in quantity demanded for a commodity is less than percentage change in
income of the consumer, it is said to be income greater than unity. For example: When the
consumer’s income rises by 5% and the demand rises by 3%, it is the case of income elasticity
less than unity.

In the given figure, quantity demanded and consumer’s income is measured along X-axis and
Y-axis respectively. When the consumer’s income rises from ​OY to ​OY​1 the
​ quantity demanded
of inferior goods falls from ​OQ to ​OQ​1 and
​ vice versa. Thus, the demand curve ​DD shows
negative income elasticity of demand.

3. Zero income elasticity of demand ( E​Y​=0)


If the quantity demanded for a commodity remains constant with any rise or fall in income of the
consumer and, it is said to be zero income elasticity of demand. For example: In case of basic
necessary goods such as salt, kerosene, electricity, etc. there is zero income elasticity of demand.

In the given figure, quantity demanded and consumer’s income is measured along X-axis and
Y-axis respectively. The consumer’s income may fall to ​OY​1 or rise to ​OY​2 from ​OY​, the
quantity demanded remains the same at ​OQ​. Thus, the demand curve ​DD​, which is vertical
straight line parallel to Y-axis shows zero income elasticity of demand.

Section-C
____________________________________________________
1. A long-run demand curve, as compare to a short-run demand curve for the same commodity,
is generally
Ans - (more elasticity)

2. Most operation produce a mixture…….is closest to producing pure services?


Ans -(restaurant)

3. The least cost combination of factors or producers….. Curves and iso costs
Ans- (ISO product)

4. Which one of the following are used in calculating opportunity costs?


Ans- (all of above )

5. Which of the following cannot be classed as a market structure?


Ans - (communism)

6. Demand curves are derived while holding constant


Ans - (income, tastes, and the price of the good )

7. The quantity of a good demanded rises from 1000 to 1500….product is approximately


Ans - (3)

8. Cross elasticity of demand is


Ans - (negative for complementary goods)

9. An explicit cost is
Ans- (none of the above)

10. The __ shape implies that the cost of production…….optimum scale (marginal cost= average
cost.)
Ans - (u)

11. The price elasticity demand for a good is .75, the demand for the good can be described as
Ans - (inelastic)

12.when the market operates without interference, price increase will distribute…this process is
known as
Ans- (price rotation)

13 when excess demand occurs in an unregulated market, there is tendency for


Ans - (price to rise)

14. A market without legal price is in equilibrium when


Ans (quantity demanded equal quantity supplied )

15 if goods are complements, definitely their


ANs -(income elasticities are negative)

16. When the decrease in the price of one good causes the demand for another good to decrease,
the goods are
Ans- (complements)

17. Isoquants that are downward- sloping straight lines imply that the inputs.
Ans - (are perfect substitutes)

18. If input prices increase, all else equal


Ans - (supply will decrease)

19. An increase in the number of firms selling pizza will cause, ceteris paribus,
Ans - (supply increase)

20. If the cross-price elasticity between two commodities is 1.5 the two good are
Ans - (normal)

21. Marginal costs is the change in total resulting from unit change in..
Ans - (output)

22. Costs that change between alternatives are called


Ans - (relevant)

23. Complementary goods have.


Ans -( negative cross price elasticity of demand with respect to each other )

24. If the principles of complement increases all else equal,


Ans- (demand will increase)

25. Which of the following is a characteristics of a perfectly competitive market?


Ans - (firms can exit and enter the market freely)

26. If a perfectly competitive firm currently produces where prices is greater than marginal costs
it.
Ans - (will increase its profits by producing more)

27. A perfectly competitive firm will maximize profit at the quantity at which the firm’s
marginal revenue equals
Ans - (marginal cost)

28. Which of the following is not included in the decision that every society must make?
Ans- (who will produce goods)

29. For a given normal supply curve, the amount of tax paid by the buyer will be larger
Ans -(the more inelastic the demand)

30. When the price of products is increased percent, the quantity demanded…………………
Ans - (elastic)

31. Decision making situation can be categorized along a scale which ranges from
Ans - (uncertainty to certainty to risk)

32.nif marginal benefits is greater than marginal costs, a rational choice involves.
Ans- (more or less depending on the benefits of other activities )

33.the quantity demand of pepsi has decreased. The best explanation for this is that.
Ans- (the price of pepsi increased)

34. The production functions incorporates the technically efficient method of..
Ans - (production)

35. A cost that does not affect a decision is called an


Ans- (irrelevant)

36. Economists typically assume that the owners of firms wish to


Ans- (all of the above)

37. In the long run, a profit-maximizing firm will choose to exit a market when
Ans- (fixed cost exceeds sunk cost)
38. A cost incurred in the past time cannot be changed by any future action is
Ans- (sunk)

39. Total revenue fails at the price of a good increases if price elasticity of demand is
Ans - (inelastic)

40. The type of market structure represented by the constant returns to scale (CRS) technology
includes
Ans- (perfect competition)

41. MRP stands for


Ans - (marginal revenue product )

42. In a free market system, the amount of goods and services that any one household gets
depend upon its
Ans - (wages and interest income)

43.cif the elasticity of demand for a commodity is estimated to be 1.5 then a decrease
in…………..
Ans - (0.56)

44. Two explanations for the law of demands are


Ans - (price and quantity effects.)

45. When firms have an incentive to exit a competitive market. Their exit will
Ans - (all the above are correct)

46. With a perfectly demand and a normal supply (upward-sloping)


Ans- (consumers will bear the entire tax burden.)

47. Is economics the central problem is


Ans- (allocation)

48. A price- and quantity -fixing agreement is known as


Ans - (collusion)

49. Setting a price below that of the competition is called


Ans- ( competitive pricing)

50. Aggregates Supplies Is The Total Amount


Ans - (Of Goods And Services Produced In An Economy)
51. Network Economics Are.
Ans.- Confined To Sectors Such As…..

52. The Bertrand Paradox…...


Ans.C- The Fact That If Firm Charge…..

53. Taxes On Activities That Cause…..


Ans.- Both A And B.

54. The Statement ‘if You Don't Pay….


Ans.- Threat When Spoken By Someone Significant Other.

55. Applying The No Cash On The Table…..


Ans.- Was Already Contained In The …..

56. In Which Of The Following Short Run Scenarios…..


Ans.- Price Is Less Than Average Fixed Cost.

57. Which Of The Following Is Not Characteristic Of A …..


Ans.- There Are High Barriers To Entry.

58. A Firm That Emerges As The Only…..


Ans.- Monopoly.

59. The Assumption That Individuals Act…..


Ans.- A Reasonable First…..

60.Cost Plus Regulation.


Ans.- Hold Down Prices Because Firms Are Not….

61. In A Cournot Duopoly A Nash …….


Ans.- When Each Player’s Price Is The Best …..

62. Suppose A Perfectly Competitive…..


Ans.- Price Equals Marginal….

63. Tacit Conclusion Excludes…..


Ans.- Threats To Undercut A Rival…..

64. If A Firm Triples All Its Inputs And …..


Ans.B- Has Economies To Scale.

65. Barriers To Entry.


Ans.B- Are Forces That Limit New…..
66. After A Price Floor Is Imposed Above….
Ans.- Rise.

67. The Rationing And Allocative …..


Ans.- Work Together To Guide Resources…..

68. The Amount By Which Price Received …...


Ans.- Producer Surplus.

69. When The Marginal Return To…...


Ans.- Parallel To The Vertical Axis.

70. Which Of The Following Is Not A Determinant…..


Ans.- Consumer Income.

71. E-commerce And An Internet Presence Are….


Ans.- Allowing The Few Employees With The Skill….

72. The Residual Demand Curve For An …...


Ans.- The Demand For The Oligopolist’s Outputs Given That….

73. If The Monopolist’s Demand Curve Is….


Ans.- -7

74. The Total Reduction In Total Economic Surplus….


Ans.- Deadweight Loss.

75. Price Subsidies To Sellers.


Ans.- Must Be Above Equilibrium…..

76. Consider A Market For Wheat Which Has An…..


Ans.- 25000

77. The Common Objective That In Economics…...


Ans.- Maximize Profits.

78. Which Of The Following Statement Expresses The…..


Ans.- Efficiency Maximizes Total….

79. A Public Utility Should Set Price Equal….


Ans.- Marginal Cost.

80. Which Of The Following Factors Of Production Is….


Ans.- The Location Of The Firm.
81. If A Single Firm,Belong To A Perfectly……
Ans.- Their Firm Will Lower Price To Drive…..

82. The Profit Maximizing Rule Mr=Mc…...


Ans.- Monopolist Only.

83. Price Discrimination


Ans.- Increases Economic Efficiency Because It Reduces …..

84. The Correct Sequence Of Market …..


Ans.- Perfect Competition,Monopolistic…..

85. The Prisoner’s Dilemma Refers…..


Ans.- Where The Playing Of Dominant…..

86. When The Supply Curve For A Good…...


Ans.- Inelastic Buyer.

87. Which Of The Following Is Not A Requirement Of A Game?


Ans.- Payoffs.

88. A Price Taker Confronts A …….


Ans.- Downward Sloping.

89.- Suppose A Competitive Firm And A …..


Ans.- The Competitive Firm Is Charging…...

90. Which Of The Following Would Be An…...


Ans.- Consumer Deciding That At The…...

91. The More Inelastic Is Supply The…….


Ans.- Larger;Producers And …….

92. Which Of The Following Problems Would Not …..


Ans.- Negotiating The Price Of A ……...

93. Which Of The Following Is Not Guaranteed By The…..


Ans.- Rich And Poor Will Have Adequate….

94. The Key Feature That ……..


Ans.- Profit Maximization.

95. The More Elastic Demand Is The……...


Ans.- Smaller,Products.
96. If All Firms In A Perfectly …...
Ans.- Exist The Industry,Sloping When Economic…..

97. In Regulated Markets,The…..


Ans.- Will Guide Resources On Some Basis …..

98. Given The Total Cost Function, Tc…..


Ans.- Fall At First And Then Rise…..

99. The Present Value Of……..


Ans.- 8667

100. Suppose The Government Grants Grain…...


Ans.- As The Profit Of Farming Increase…..

101. A long-run demand curve, as compared to a short-term demand….


Ans.- Can't say

102. A market without legal prices is in equilibrium when:


Ans.- The demand curve remains….

103.when firm have an incentive to exit a competitive market their exit will
Ans. - All of above are corrected

104. The production function incorporating the technically efficient method of -----
Ans.- All of the above

105.A perfect competitive firm will maximize profit at the qty at which the firm’s marginal
revenue equals
Ans.- Total cost.

106. If the price of a complement increases, all else equal,


Ans.- Quantity Demanded will increase.

107. If input price increase, all else equal


Ans.- Supply will decrease.

108. An increase in the number of firm selling pizza will cause,ceteris paribus
Ans.-SUPPLY FALLS

109. Two explanations for the law of demand are...


Ans.- Substitution and income effects.

110. Demand curves while holding constant:


Ans.- Income tastes, and the price of the…..
111. An explicit cost is
Ans.- None of the above

112. The quantity of a good demanded rises from 1000 to 1500 units……..
Ans.- 1

113. With a perfectly elastic demand and a normal supply(upward -sloping)


Ans.- consumers and product…...

114.isoquants that are downward- sloping straight lines imply that the inputs
Ans .-ARE PERFECT SUBSTITUTE

115. If the cross price elasticity between two commodities is 1.5 the two goods are
Ans.- SUBSTITUTE

116. When excess demand occurs in an unregulated market,there is tendency for


Ans.- PRICE TO RISE

117. Which of the following is not included in the decision that every society must make
Ans.- WHO WILL CONSUME THE GOODS……….

118.If a perfectly competitive firm currently produces where price is greater than marginal cost.
Ans.- WILL INCREASE ITS PROFIT BY

119. Setting a price below that of the competition is called…...


Ans.- SKIMMING

120. If the price elasticity of demand for a goods is .75 the demand for the goods can be
described as.
Ans.- NORMAL
121. A cost incurred in the past that cannot be changed by any future action is
Ans.-LOST

122. The ……….shape implies that the cost of the production continues to be low till the firm
reaches the optimum scale (MC=AC)
Ans.- Q

123. When the market operates without interference……….


Ans.- QUANTITY ADJUSTMENT

124. The least cost combination of factor or produced equilibrium is now explained with the
help of above_________ curves and lso coasts
Ans.- BOTH OF THE ABOVE
125. In a long run ,a profit - maximizing firm will choose to exit a market when
Ans.- REVENUE FROM PRODUCTION IS LESS…..

126. The type of market structure represented by the constant…….


Ans. DUOPOLY

127. Economist typically assume …….wish to


Ans.- ALL OF THE ABOVE

128. For a given normal supply curve , the amount if a tax paid by the buyer will ……….
Ans.- WHEN THE PRICE IS HIGH

129. Marginal cost is change in total cost…….


Ans.- BOTH

130. Cost that change between alternatives are called


Ans.- OPPORTUNITY

131. If goods are complements, definitely their


Ans.-CROSS ELASTICITY ARE NEGATIVE

132. A price quantity fixing agreement is known as


Ans.- Price leadership

133. In a tree market system , the amount of goods and services……….depends upon its
Ans.- Income

134. Total revenue falls as the price of a good increases if price ela….
Ans.- Inelastic

135. If Marginal Benefits Is Greater Than Marginal Cost,......Involves


Ans.- No More Of The Activity.

136. Cross Elasticity Of Demand Is


Ans.- Negative For Complementary Goods

137. Complementary Goods Have


Ans.- Positive Income Elasticity Of Demand...

138. Which Of The Following Cannot Be Classes As A Market Structure?


Ans.- Communication

139. Mrp Stand For


Ans.- None Of Those.
140. Which Of The Following Are Used In Calculating Opportunity Costs?
Ans.- Monetary Cost.

141. When The Decrease In The Price Of One Good Causes The Demand….
Ans.- Complements

142. Most Operations Product A Mixture Of Both Product And…...


Ans.- Counceller

143. Which Of The Following Is A Characteristic Of A Perfectly Competitive...


Ans.- Firms Can Exit And Enter The….

144.Decision Making Situation Can Be Categorized Along A Scale Which Range From
Ans. -certainty To The Risk To Uncertain To …….

145. Aggregate Supplies Is The Total Amount


Ans.- Of Goods And Services Produced...

146. In Economics The Central Problem Is:


Ans.- Scarcity

147.When The Price Of A Product Is Increased 10 Percent……..Demand For This Product Is.
Ans.-inelastic

148.If The Elasticity Of Demand For A Commodity Is Estimated To Be1.5 Then Decrease In
Price From $2.10 To $1.90………..
Ans.-0.15

149. The Qty Demanded Of Pepsi Has Decreased .The Best Explanation For This Is That…...
Ans D- The Price Of Coca Cola Has …..

150. A Cost That Does Not Affect A Decision Is Called An


Ans.D-irrelevant

151. Uncertainty And Risk Arise Mainly Due To

A. Uncertainly Behaviour Of The Market Forces,


B. Changing Business Environment,
C. Emergence Of Complexity Of The Modern Business World And Social And
Political, External Influence On The Domestic Market And Political Changes In
The Country,
D. All Above

Ans. D
152. Source Resources Are To Be Used With Utmost Efficiency To Get

A. The Optimal Results,


B. The Sub-optimal Results,
C. The Normal Results,
D. All the above

Ans. D

153. The root cause of all economic problems is

A. Over-population,
B. Scarcity,
C. Capital
D. Poor management

Ans. B

154. Discounting principle refers to

A. Time value of interest,


B. Time value of money,
C. Time value of investment,
D. Time value of capital

Ans. B

155. Which​ ​of the following statements is true?

a. When the supply increases, both the price and the quantity will increase,
b. When the supply increases, the supply curve shifts towards the left,
c. A shift in the supply curve towards the right results in a fall in the price,
d. A decreases in the quantity supplied results in shifting of the supply curve towards the
left.

Ans. c

156. Which of the following statements is false?

a. An increase in tax will affect the customers more than the producers if the supply
schedule in inelastic,
b. An increase in tax will affect the customers more than the producers if the demand
schedule is inelastic,
c. Both (a) and (d) above,
d. An increase in tax will affect the customer's less than the producers if the demand
schedule in inelastic.

Ans. a
157. For complementary goods, the cross elasticity of demand will be

a. Zero,
b. Infinity
c. Positive but less than one,
d. Negative.

Ans. c

158. When the income elasticity of demand for a good is negative, the good is

a. Normal good,
b. Luxury good,
c. Inferior good,
d. Giffen good

Ans. c

159. If change in all inputs leads to a proportional change in the output, it is a case of

a. Increasing returns to scale,


b. Increasing returns to scale,
c. Diminishing returns to scale,
d. Variable returns to scale

Ans. d

160. Isoquants are

a. Equal cost lines,


b. Equal product lines,
c. Equal revenue lines,
d. Equal total utility lines

Ans. a

161. When average product is highest

a. Total product is maximum,


b. Total product is maximum,
c. Marginal product is zero,
d. Marginal product is equal to average product

Ans. d

162. If marginal product is negative, it means that the

a. Total product is maximum,


b. Average product is at maximum,
c. Average product is falling,
d. Total product is increasing

Ans. c

163. Which of the following curves is called envelope curve?

a. Long run total cost curve,


b. Long run average total cost curve,
c. Long run total cost curve,
d. Long run average variable cost curve

Ans. d

164. In perfect competition, a firm maximizing its profits will set its output at that level where

a. Average variable cost=price


b. Marginal cost=price
c. Fixed cost=price
d. Average fixed cost=price

Ans. b

165. Which of the following curves resembles supply curve under perfect competition in the
short run?

a. Average cost curve above breakeven point


b. Marginal cost curve above shut down point
c. Marginal utility curve
d. Average utility curve

Ans. a

166. Which of the following is not a feature of perfect competition?

a. Low price
b. No one is large enough to influence the market pice
c. Homogeneous products
d. Horizontal demand curve

Ans. a

167. In the long run, a perfectly competitive firm earns only normal profits because of

a. Large number of seller and buyer in the industry


b. Large number of seller and buyer in the industry
c. Free entry and exit of industry
d. Both (a) and (b) above
Ans. c

168. The horizontal demand curve for a firm is one of the characteristic features of

a. Oligopoly
b. Monopoly
c. Monopolistic competition
d. Perfect competition

Ans. d

169. Capital budgeting analysis uses

a. Cash inflows,
b. Cash outflows,
c. Cash inflows and cash outflows,
d. None of the above

Ans. c

170. Cost of capital is the rate at which the company could obtain

a. Capital (funds) from its creditors and investors,


b. Capital (Funds) From its bankers,
c. Capital (funds) from its employees,
d. All the above

Ans. a

171. Decision trees are useful tools for helping you to choose between several courses of

a. Process,
b. Action
c. Plan
d. Management

Ans. b

172. Behaviour economics is concerned with the ways in which the actual decision-making
process influences

a. Decision that are made in practice; combines psychology and mathematical,


b. Decisions that are made in unpracticed; combines psychology and economics,
c. The decisions that are made in practice,
d. The decisions that are made in practice; combines psychology and economics.

Ans. d
173. The demand-for-money curve illustrates the..........................Relationship between the
quantity demanded of money and...................

a. Inverse; the interest rate


b. Direct; GDP.
c. Direct; the interest rate
d. Inverse; GDP

Ans. a

174. If the interest rate increases, the opportunity cost of holding money........................, and the
quantity demanded of money.....................

a. Does not change; does not change


b. Increases; also increases
c. Decreases; increases
d. Increases; decreases
e. Decreases; also decreases

Ans. e

175. as the interest rate falls, the quantity

a. Increases; increases; more


b. Increases; decreases; more
c. Increases; decreases; less
d. Decreases; increases; more
e. Decreases; decreases; more

Ans. e

176. As the interest rate falls, the quantity

a. Demanded of money falls


b. Demanded of money rises
c. Supplied of money rises
d. Supplied of money falls

Ans. a

177. Compared to the Keynesian transmission mechanism, the monetarist transmission


mechanism is

a. Direct
b. Indirect
c. Inverse
d. Elliptical
e. Nine of the above
Ans. c

178. The Keynesian transmission mechanism might get blocked if

a. Investment is insensitive to changes in interest rates


b. The goods market is not in equilibrium
c. The goods market supply rises too quickly
d. Interest rates are too high before they fall.

Ans. a

179. If market interest rates increase, the prices of exiting bonds will

a. Decreases
b. Not change
c. Increase
d. Decrease if Real GDP decreases and increase if real GDP increases.

Ans. c

180. Assuming you want to earn profits, it is best to ....................bonds when you expect interest
rate are going to

a. Buy; rise
b. Buy; fall sell; rise
c. A and c
d. B and c

Ans. b
181. Monetarists believe that changes in the supply of money
a. Do not affect aggregate demand
b. Affect aggregate demand through the loanable funds market only
c. Affect only the investment component of aggregate demand
d. Affect aggregate demand directly
Ans. b

182. Compared to the monetarist transmission mechanism, the Keynesian transmission


mechanism is

a. Indirect and long


b. Direct and long
c. Direct and short
d. Indirect and short

Ans. d
1. Econometrics is
a)​ ​A modern name for economics
b) A specialized branch of economics which applies the tools of statistics to the
economic problems
c)​ ​A branch of economics which combines macroeconomic principles with welfare economics
d)​ ​A branch of economics which combines microeconomic principles with international trade
e)​ ​A specialized branch of economics which describes neo-classical microeconomics

2. Which of the following comes under the broad definition for factors of production?
a)​ ​Technology
b)​ ​Obsolete machinery
c)​ ​Innovations
d) Capital
e)​ ​Patent rights

3. Which of the following statements is true?

a) When the supply increases, both the price and the quantity will increase.
b)​ ​When the supply increases, the supply curve shifts towards the left
c)​ ​A shift in the supply curve towards the right results in a fall in the price
d)​ ​A decrease in the quantity supplied results in shifting of the supply curve towards the left.
e) ​An increase in the quantity supplied leads to a fall in the price resulting in the shifting of the
supply curve towards the left.

4. Which of the following statement(s) is/are false?


a)​ ​If the demand falls, the price will fall.
b)​ ​As the price rises the quantity demanded will fall.
c) If demand rises, the demand schedule shifts to the left.
d)​ ​Both (a) and (b) above.
e)​ ​Both (a) and (c) above.

5. Which of the following statements is false?


a) An increase in tax will affect the customers more than the producers if the supply
schedule is inelastic.
b) ​An increase in tax will affect the customers more than the producers if the demand schedule
is inelastic.
c) ​An increase in tax will affect the customers less than the producers if the supply schedule is
inelastic.
d) ​An increase in tax will affect the customers less than the producers if the demand schedule is
inelastic.
e)​ ​Both (a) and (d) above.

6. Which of the following statements is true?


a)​ ​Elasticity of demand is constant throughout the demand curve.
b) Elasticity of demand increases as one goes down the demand curve.
c)​ E​ lasticity of demand decreases as one goes down the demand curve.

d)​ ​The slope of the demand curve equals its elasticity.


e)​ ​The price and total revenue move in the same direction when demand is elastic.

7. For complementary goods, the cross elasticity of demand will be


a)​ ​Zero.
b)​ ​Infinity.
c)​ ​Positive, but less than infinity.
d) Negative.
e)​ ​None of the above.

8. When the income elasticity of demand for a good is negative, the good is
a)​ ​Normal good.
b)​ ​Luxury good.
c) Inferior good.
d)​ ​Giffen good.
e)​ ​Necessity.

9. If both income and substitution-effects are strong, this region of the demand curve must be
a) Relatively price elastic.
b)​ ​Relatively price inelastic.
c)​ ​Unit-elastic.
d)​ ​Perfectly inelastic.
e)​ ​Perfectly elastic.

10. If a good has close substitutes,


a) Its demand curve will be relatively elastic.
b)​ ​Its demand curve will be relatively inelastic.
c)​ ​Its demand curve could be unit-elastic.
d)​ ​Either (a) or (c).
e)​ ​Either (b) or (c).

11. The demand for most products varies directly with the change in consumer income. Such
products are known as
a) Normal goods.
b)​ ​Prestigious goods.
c)​ ​Complementary goods.
d)​ ​Inferior goods.
e)​ ​Substitute goods.

12. Which of the following statements is true with regard to price elasticity of demand?
a)​ ​Elasticity remains constant throughout the demand curve.
b)​ ​Elasticity increases with increase in quantity demanded.
c) Elasticity increases as the price decreases.
d)​ ​Elasticity is equal to the slope of the demand curve.
e)​ ​Higher the elasticity, more responsive the demand is for a given change in price.

13. Which of the following goods can be considered substitutes?


a)​ ​Pen and Paper.
b)​ ​Car and Petrol.
c)​ ​Bread and Butter.
d) Tea and Coffee.
e)​ ​Keyboard and Monitor.

14. Which of the following statements concerning indifference curves is true?


a) ​An indifference curve is the locus of points describing proportional price levels of the two
goods.
b)​ ​Indifference curves pre-suppose the measurement of total utility and marginal utility.
c) An indifference curve is the locus of points representing various combinations of
two goods about which the consumer is indifferent.
d)​ ​Indifference curve pre-suppose the validity of “the law of diminishing returns”
e)​ ​None of the above

15. If a change in all inputs leads to a proportional change in the output, it is a case of
a)​ ​Increasing returns to scale
b) Constant returns to scale
c)​ ​Diminishing returns to scale
d)​ ​Variable returns to scale
e)​ ​Inefficient returns to scale

16. Isoquants are


a)​ ​Equal cost lines
b) Equal product lines
c)​ ​Equal revenue lines
d)​ ​Equal total utility lines
e)​ ​Equal marginal utility lines

17. When average product is highest


a)​ ​Total product is maximum
b) Marginal product is maximum
c)​ ​Marginal product is zero
d)​ ​Marginal product is negative
e)​ ​Marginal product is equal to average product

18. If marginal product is negative, it means that the


a)​ ​Total product is at maximum
b)​ ​Average product is at maximum
c) Average product is falling
d)​ ​Total product is increasing
e)​ ​Average product is negative

19. Which of the following curves is called envelope curve?


a)​ ​Long run total cost curve
b) Long run average total cost curve
c)​ ​Long run marginal cost curve
d)​ ​Long run average variable cost curve
e)​ ​Long run average fixed cost curve

20. Which of the following costs remain constant as the output increases?
a)​ ​Marginal cost
b)​ ​Average variable cost
c) Average fixed cost
d)​ ​Total variable cost
e)​ ​None of the above

21. In perfect competition, a firm maximizing its profits will set its output at that level where
a)​ ​Average variable cost = price
b) Marginal cost = price
c)​ ​Fixed cost = price
d)​ ​Average fixed cost = price
e)​ ​Total cost = price

22. Which of the following curves resembles supply curve under perfect competition in the short
run?
a)​ ​Average cost curve above breakeven point
b) Marginal cost curve above shut down point
c)​ ​Marginal utility curve
d)​ ​Average utility curve
e)​ ​Average variable cost curve

23. Which of the following is not a feature of perfect competition?


a)​ ​Large number of seller and buyers
b)​ ​No one is large enough to influence the market price
c)​ ​Homogeneous products
d) A horizontal demand curve
e)​ ​Low price

24. In the long run, a perfectly competitive firm earns only normal profits because of
a)​ ​Product homogeneity
b)​ ​Large number of seller and buyer in the industry’
c) Free entry and exit of industry
d)​ ​Both (a) and (b) above
e)​ B
​ oth (b) and (c) above

25. The horizontal demand curve for a firm is one of the characteristic features of
a)​ ​Oligopoly
b)​ ​Monopoly
c)​ ​Monopolistic competition
d) Perfect competition
e)​ ​Duopoly

26. A perfectly competitive firm can increase its sales by


a)​ ​Reducing the price
b)​ ​Increasing the price
c) Increasing the production
d)​ ​Increasing the expenditure of advertisement
e)​ ​Increasing the sales force

27. Which of the following is not a source of market imperfection?


a) Technology
b)​ ​Size of the firm
c)​ ​Product differentiation
d)​ ​Availability of resources
e)​ ​Forces of supply and demand

28. The maximum profit condition for a monopoly firm is


a)​ ​Total cost should be minimum
b)​ ​Total revenue should be maximum
c) Marginal revenue is equal to marginal cost
d)​ ​Quantity should be maximum
e)​ ​Price should be maximum

29. “Four-firm concentration” refers to


a)​ ​The number of firms in an industry
b)​ ​The four largest firm in four different and important industries in an economy
c)​ ​The number of industries in an economy which have only four firms
d) The percent of the total industry output that is accounted for by the largest four
firms
e)​ ​The percent of the total industry output that is accounted for by the largest four firms

30. Market inefficiencies can come from


a)​ ​Externalities
b)​ ​Monopolies
c) Imperfect information
d)​ ​Entry barriers
e)​ ​All of the above
31. A monopolist who faces a negatively sloped demand curve operates in the region where the
elasticity of demand is
a)​ ​Less than 1
b)​ ​Equal to 1
c)​ ​Greater than 1
d) between 0 and 1
e)​ ​0

32. In which of the following market structures the entry is least difficult?
a)​ ​Monopoly
b) oligopoly
c)​ ​duopoly
d)​ ​regulated monopoly
e)​ ​monopolistic competition

33. Which of the follow is false in a monopolistic competition?


a)​ ​Many buyers and sellers
b) Identical products
c)​ ​Easy entry and exit
d)​ ​Price of the competitor is the benchmark price
e)​ ​Each firm could be market leader in its product segment

34. The term “differentiated product” denotes


a)​ ​Different products in similar packets
b)​ ​Different products
c) Same product used in different applications
d)​ ​Different products used by a differentiated set of people
e)​ ​Products whose important characteristics vary

35. Which of the following is common feature in both a monopolistic competitive market and
oligopoly market?
a) Product differentiation
b)​ ​Interdependence among member firms
c)​ ​Kinked demand curve
d)​ ​Limited number of sellers
e)​ ​Entries blocked

36. The term “collusion” refers to


a)​ ​A situation in which government sets prices with the market leader in oligopoly
b) ​A situation in which government jointly sets prices with the small players in an industry in
the larger interest of the society
c) A situation in which all firms in an industry decide the price and output
d) ​A situation in which two powerful groups in an industry hand with government to rule the
industry
e) ​A situation in which central and state government jointly decide price and output for an

industry

37. Which of the following does not likely to lead to the failure collusive oligopoly?
a)​ ​Secret price cutting
b)​ ​More number of firms
c)​ ​Undifferentiated products
d) Rapidly changing technology
e)​ ​Competition from foreign firms

38. Price leadership refers to


a)​ ​Pre-emptive pricing made possible by the learning curve
b) A form of price collusion
c)​ ​The maintains of a monopolistic price
d)​ ​Cut throat competition
e)​ ​None of the above

39. A cartel is
a) A group firms which get together and make joint price and output decisions to
maximize joint profits
b)​ ​Form of tacit collusion
c)​ ​Type of oligopoly in which curve is kinked
d)​ ​Duopoly
e)​ ​None of the above

40. A zero-sum game is one in which


a)​ ​The gain of one player equals the loss of another player
b)​ ​The gain of one player will not equal the loss of another player
c)​ ​The maximin equals the minimax
d)​ ​The maximin does not equal the minimax
e)​ ​The equilibrium and the dominant equilibrium are the same

41. A change in quantity demanded refers to


a) Contraction along a demand curve
b)​ ​Shift of the demand curve
c)​ ​Movement along a demand curve
d)​ ​Expansion along a demand curve

42. A change in demand refers to


a)​ ​Contraction along a demand curve
b) Shift of the demand curve
c)​ ​Movement along a demand curve
d)​ ​Expansion along a demand curve

43. If two goods are substitutes, the price elasticity of demand is


a)​ ​Negative
b) Positive
c)​ ​Zero
d)​ ​Not defined

44 If two goods are complementary, the price elasticity of demand is


a) Negative
b)​ ​Positive
c)​ ​Zero
d)​ ​Not defined

45. Price elasticity of demand is defined as


a)​ ​Absolute change in quantity demanded due to absolute change in price
b)​ ​Percentage change in quantity demanded due to percentage change in price
c) Relative change in quantity demanded due to change in price
d)​ ​Marginal change in quantity demanded due to marginal change in price

46. Total revenue will increase if


a) Demand is elastic
b)​ ​Demand is inelastic
c)​ ​Demand is unitary elastic
d)​ ​None of the above

47. An isoquant shows


a)​ ​All combinations of labor and capital
b)​ ​All combinations of good X and good Y
c) All combinations of labor and capital
d)​ ​All combinations of labor and capital

48. Changes in income are shown by


a)​ ​Parallel shift of isoquant
b)​ ​Movement along the budget line
c)​ ​Parallel shift of budget line
d) Both (b) & (c)

49. The relationship between elasticity and total revenue


a)​ ​MR p 1 1efff f g
b)​ ​MR p 1 D 1efff f g
c)​ ​MR p 1@1efff f g
d) None of the above

50. In case of inferior goods the income elasticity is


a)​ ​Positive
b) Negative
c)​ ​Zero
d)​ N
​ one of the above.

51. An indifference curve is the locus of


a)​ ​All the combinations of good X and Y giving different level of satisfaction
b) All the combinations of capital and labor giving same level of output
c)​ ​All the combinations of good X and Y giving same level of satisfaction
d)​ ​All the combinations of capital and labor giving different level of output

52. Short run in production function refers to


a)​ ​When all factors of production become variable
b)​ ​One factor of production varies keeping all other constant
c)​ ​When all factors of production become variable
d) None of the above

53. Opportunity cost refers to


a)​ ​The expected return from the use of the resource
b) The expected return from the second best alternative use of the resource
c)​ ​Accounting cost less of unilateral transfers
a)​ ​None of the above

54 Long run average cost curve is also known as


a) Envelope curve
b)​ ​Angel curve
c)​ ​Laffer curve
d)​ ​None of the above

55. Internal economies of scale determine


a)​ ​The position of long run average cost curve
b) The shape of long run average cost curve
c)​ ​The shape of short run average cost curve
d)​ ​The position of short run average cost curve

56. The nature and shape of AFC is


a)​ ​A rectangular Hyperbola
b)​ ​A horizontal Line
c) It is “U” shaped
d)​ ​A vertical Line

57. Which one of the statement is correct


a) All costs are variable costs in the long run except LMC
b)​ ​TFC is inverse “S“Shaped reflecting Laws of Returns.
c)​ ​Over a very long range of Operation, AFC is Zero.
d)​ ​None of the above is correct.

58. Explicit cost is also known as –


a)​ ​Imputed Cost
b) Implied Cost
c)​ ​Accounting Cost
d)​ ​Opportunity Cost

59. The use of highly structured meeting agenda and restricted discussion or interpersonal
communication during the decision making process is known as –
a)​ ​Nominal Group Technique,
b)​ ​Brainstorming,
c) Delphi Group Technique,
d)​ ​Both (b) & (c)

60. Variation in Data occurring due to regularly recurring fluctuations in economic activity
during each year is
a)​ ​Cyclical fluctuations
b) Seasonal Variations
c)​ ​Random Variation
d)​ ​Irregular Variation

61. In the short run the supply curve of a firm in perfectly competitive market is
a)​ ​Average cost curve
b)​ ​Total cost curve
c) Marginal cost curve
d)​ ​None of the above

62. A firm is price taker in perfect competition market structure because


a)​ ​Single firm supplies significant part of total market supply
b)​ ​Single firm supplies insignificant part of total market supply
c)​ ​Single firm supplies Homogeneous product
d) Both (b) and (c)

63. Highest degree of allocative inefficiency is the feature of


a)​ ​Perfect competition
b) Monopoly
c)​ ​Oligopoly
d)​ ​Not defined

64 Cartels and collusion are


a)​ ​Illegal activities
b)​ ​Legal framework
c)​ ​Authorized framework
d) Not defined
65. As more labour is added to a fixed amount of input, the rate at which output goes up begins
to decrease. This is called
a)​ ​Diminishing marginal utility.
b) Diminishing marginal productivity.
c)​ ​Diminishing marginal costs.
d)​ ​Diminishing marginal profit.

66. If the cost of sugar rises and sugar is a major ingredient in jelly beans, then the jelly bean
a)​ ​Demand curve shifts to the left.
b)​ ​Supply curve shifts to the left.
c)​ ​Supply curve shifts to the right.
d) Demand and supply curves both shift to the right.

67. Which one of the following is not the characteristic of Perfect Competition?
a)​ ​All firms sell an identical product.
b)​ ​All firms are price takers.
c)​ ​All firms have a relatively small market share.
d) Buyers do not know the nature of the product being sold and the prices charged by
each firm.

68. Which one of the following is not the characteristic of demand?


a)​ ​There should be the willingness to purchase
b)​ ​There should be the capacity to purchase
c) Specific time frame
d)​ ​Real market place is required

69. For a normal good:


a)​ ​The price elasticity of demand is negative; the income elasticity of demand is negative
b)​ ​The price elasticity of demand is positive; the income elasticity of demand is negative
c)​ ​The price elasticity of demand is negative; the income elasticity of demand is positive
d) The price elasticity of demand is positive; the income elasticity of demand is

70. which of the following is true?


a)​ ​If the marginal cost is greater than the average cost the average cost falls
b)​ ​If the marginal cost is greater than the average cost the average cost increases
c)​ ​If the marginal cost is positive total costs are maximised
d) If the marginal cost is negative total costs increase at a decreasing rate if output
increases

71. If marginal cost is positive and falling:


a)​ ​Total cost is falling
b) Total cost is increasing at a falling rate
c)​ ​Total cost is falling at a falling rate
d)​ ​Total cost is increasing at an increasing rate
72. To maximise sales revenue a firm should produce where:
a)​ ​Marginal cost is zero
b) Marginal revenue is maximised
c)​ ​Marginal revenue is zero
d)​ ​Marginal revenue equals marginal cost

73. Normal profit occurs when:


a) Average revenue equals average variable cost
b)​ ​Marginal revenue equals marginal cost
c)​ ​Average revenue equals marginal cost
d)​ ​Average revenue equals average cost

74 Barriers to entry:
a)​ ​Do not exist in monopoly
b) Cannot exist in oligopoly
c)​ ​Do not exist in monopolistic competition
d)​ ​Do exist in perfect competition

75. Which best describes price discrimination?


a)​ ​Charging different prices for different products
b)​ ​Charging the same prices for different products
c)​ ​Charging the same prices for the same products
d) Charging different prices for the same products

76. In perfect price discrimination:


a) Consumer surplus is maximized
b)​ ​Consumer surplus is zero
c)​ ​Producer surplus is zero
d)​ ​Community surplus is maximised

77. In perfect price discrimination:


a) The demand curve is the marginal cost curve
b)​ ​The average revenue equals the average cost
c)​ ​The marginal cost is the average cost curve
d)​ ​The demand curve is the marginal revenue

78. If a few firms dominate an industry the market is known as:


a) Oligopoly
b)​ ​Competitively monopolistic
c)​ ​Duopoly
d)​ ​Monopolistic competition

79. In case certain goods are not sold within a reasonable time, the retailer pulls the price down,
it is known as
a) Adjustment pricing
b)​ ​Administered pricing
c)​ ​Mark-down pricing
d)​ ​Mark-up pricing

80. This pricing strategy acts as a barrier to entry to new firms


a) Limit Pricing
b)​ ​Administered Pricing
c)​ ​Peak –Load Pricing
d)​ ​Skimming Pricing

81. Select the group that best represents the basic factors of production.
a) land, labor, capital, entrepreneurship
b)​ ​land, labor, money, management skills
c)​ ​land, natural resources, labor, capital
d)​ ​land, labor, capital, technology

82. Scarcity is a condition that exists when


a)​ ​There is a fixed supply of resources.
b)​ ​There is a large demand for a product.
c) Resources are not able to meet the entire demand for a product.
d)​ ​All of the above.

83. Managerial economics is best defined as


a)​ ​The study of economics by managers.
b)​ ​The study of the aggregate economic activity.
c) The study of how managers make decisions about the use of scarce resources.
d)​ ​All of the above are good definitions.

84. A critical element of entrepreneurship (as opposed to managerial skills) is


a)​ ​Leadership skills.
b) Risk taking.
c)​ ​Technology.
d)​ ​Political skills.

85. A company will strive to minimize


a)​ ​Transaction costs.
b)​ ​Costs of internal operations.
c) Total costs of transactions and internal operations combined.
d)​ ​Variable costs.

86. Company goals that are concerned with creating employee and customer satisfaction and
maintaining a high degree of social responsibility are called ___________ objectives.
a)​ ​social
b) noneconomic
c)​ ​welfare
d)​ p​ ublic relations

87. ______________ risk involves variation in returns due to the ups and downs of the economy,
the industry and the firm.
a)​ ​structural
b)​ ​fluctuation
c) business
d)​ ​financial

88. _____________ risk concerns the variation in returns that is induced by leverage.
a)​ ​business risk
b)​ ​premium
c)​ ​business
d) financial

89. A new taco-making machine that is similar in size and cost to hot dog carts has encouraged
more street vendors to begin selling tacos. What short-run impact do you think this might have
on the market for hot dogs?
a) decrease in the demand for hot dogs
b)​ ​increase in the demand for hot dogs
c)​ ​decrease in the supply of hot dogs
d)​ ​increase in the supply of hot dogs

90. Which of the following is not a non-price determinant of demand?


a)​ ​tastes and preferences
b)​ ​income
c) technology
d)​ ​future expectations

91. Which of the following is not a nonprice determinant of supply?


a)​ ​costs
b)​ ​technology
c) income
d)​ ​future expectations

92. Which of the following statements is not true?


a)​ ​An increase in demand causes equilibrium price and quantity to rise.
b)​ ​A decrease in demand causes equilibrium price and quantity to fall.
c)​ ​An increase in supply causes equilibrium price to fall and quantity to rise.
d) A decrease in supply causes equilibrium price to rise and quantity to rise.

93. Which of the following would cause a decrease in the demand for fish?
a)​ ​The price of red meat increases.
b)​ ​The price of fish increases.
c) The price of chicken decreases.
d)​ T
​ he number of fishing boats decreases.

94. Which of the following will not cause a short run shift in the supply curve?
a)​ ​a change in the number of sellers
b)​ ​a change in the cost of resources
c) a change in the price of the product
d)​ ​a change in future expectations

95. In the short run, a change in the equilibrium price will


a)​ ​Always lead to inflation.
b)​ ​Cause a shift in the demand curve.
c)​ ​Cause a shift in the supply curve.
d) Cause a change in the quantity demanded or supplied.

96. The switch to the use of HFCS from sugar in soft drinks was prompted in large part by its
relatively lower price. Assuming a competitive market, what effect would this change have on
the equilibrium price and output for soft drinks?
a)​ ​price rises, output falls
b) price falls, output rises
c)​ ​price rises, output rises
d)​ ​price falls, output falls

97. Which of the following would indicate that price is temporarily below its market
equilibrium?
a)​ ​There are a number of producers who are left with unwanted inventories.
b) There are a number of customers who must be placed on waiting lists for the
product.
c)​ ​Firms decide to leave the market.
d)​ ​The government must step in and subsidize the product.

98. Suppose demand is expressed as QD = 300 - 50P. If we want to make this equation
consistent with the typical supply and demand diagram, this equation must be stated as:
a)​ ​P = 300 - 50Q
b) P = 6 - .02Q
c)​ ​P = 50 - 300Q
d)​ ​Q = 6 - .02P

99. Which of the following refers to a shift in the demand curve?


a) "This new advertising campaign should really increase our demand."
b)​ ​"Let's drop our price to increase our demand."
c)​ ​"We dare not raise our price because our demand will drop."
d)​ ​"If new sellers enter the market, the demand for the product is bound to increase."

100. A decrease in the price of personal computers can result from


a)​ ​A decrease in the price of chips.
b)​ ​Improvements in methods of assembling computers.
c)​ ​An increase in the gross national product.
d) Both a. and b.

101. Which of the following can result in an increase in the supply of residential housing in the
short run?
a) a decrease in the price of lumber
b)​ ​a decrease in real household incomes
c)​ ​an increase in the wages of electricians
d)​ ​none of the above

102. Which of the following is a key determinant of both supply and demand?
a)​ ​income
b) future expectations
c)​ ​tastes and preferences
d)​ ​sales tax

103. A market is in equilibrium when


a)​ ​Supply is equal to demand.
b)​ ​The price is adjusting upward.
c) The quantity supplied is equal to the quantity demanded.
d)​ ​Tastes and preference remain constant.

104. Which of the following indicates that there is a shortage in the market?
a)​ ​demand is rising
b)​ ​demand is falling
c) price is rising
d)​ ​price is falling

105. The sensitivity of the change in quantity demanded to a change in price is called
a)​ ​Income elasticity.
b)​ ​Cross-elasticity.
c) Price elasticity of demand.
d)​ ​Coefficient of elasticity.

106. The sensitivity of the change in quantity consumed of one product to a change in the price
of a related product is called
a) Cross-elasticity.
b)​ ​Substitute elasticity.
c)​ ​Complementary elasticity.
d)​ ​Price elasticity of demand.

107. A product that is similar to another, and can be consumed in place of it, is called
a)​ ​A normal good.
b)​ ​An inferior good.
c)​ ​A complementary good.
d) A substitute good.

108. Two goods are _____________ if the quantity consumed of one increases when the price of
the other decreases.
a)​ ​normal
b)​ ​superior
c) complementary
d)​ ​substitute

109. The government unit that wants to achieve "revenue enhancement" will find it considerably
more favorable to enact an excise tax on products whose demand is
a)​ ​Highly elastic.
b)​ ​Relatively elastic.
c) Highly inelastic.
d)​ ​Unitary elastic.

110. A product consumed in conjunction with another is called a(n)


a)​ ​Inferior good.
b) Complementary good.
c)​ ​Normal good.
d)​ ​Substitute good.

111. Managerial economics deals with the problem of


a) An individual firm.
b)​ ​An industry.
c)​ ​An economy.
d)​ ​Global economy.

112. Managerial Economics as a specialized branch of Economics


a)​ ​Provide ready-made solutions to business problems.
b) Provide logic and methodology to find solutions to business problems.
c)​ ​Provide theoretical background to analyze business problems.
d)​ ​Provide alternative answers to specific business problems.

113. Managerial economics as a new branch of Economics


a)​ ​Highlights on analyzing business problems.
b)​ ​Acts totally independent of other subjects.
c)​ ​Uses new techniques to identify business and management problems.
d) Applies economic theories and concepts to solve business and management
problems.

114. Demand for a product refers to


a)​ ​Various quantities that are demanded by consumers.
b)​ ​Various amounts desired by consumers.
c)​ ​Total quantity of a product demanded during a given period of time.
d) Total quantity of a product demanded at a particular price in the market during a
given period of time.

115. The relationship between price and demand is


a)​ ​Direct.
b) Inverse.
c)​ ​Proportionate.
d)​ ​Positive.

116. Which of the assumptions on which the demand is based are


a)​ ​Production Costs.
b)​ ​Technology.
c)​ ​Prices of Inputs.
d) Prices of other related goods and tastes and preferences.

117. In case of increase in demand, the demand curve


a)​ ​Shifts backwards.
b) Shifts forward.
c)​ ​Will have upward slope.
d)​ ​Will be horizontal.

118. The Law of Demand assuming other things to remain constant, establishes the relationship
between
a)​ ​Income of the consumer and the quantity of a good demanded by him.
b) Price of a good and the quantity demanded.
c)​ ​Price of a good and the demand for its substitute.
d)​ ​Quantity demanded of a good and the relative prices of its complimentary goods.

119. Demand forecasting is made - for the


a)​ ​For the existing products only.
b)​ ​New products only.
c) For both the existing products & for the new products.
d)​ ​For the substitutes only.

120. A firm operating under conditions of perfect competitions can


a)​ ​Determine the price of its product.
b) Determine only the size of its output.
c)​ ​Promote the sales through effective advertisement.
d)​ ​Capture the market by cutting down the price.

121. Historical data is used in estimating future demand under


a)​ ​Survey method.
b)​ ​Expert opinion method.
c) Statistical method.
d)​ C
​ omplete Enumeration method.

122. An increase in supply demand remaining constant will change the equilibrium
a) Causing a fall in price.
b)​ ​Causing a backward shift in supply curve.
c)​ ​Causing no change in price.
d)​ ​Causing a rise in price.

123. If the proportionate change in supply is exactly equal and proportionate to the change in
price than elasticity of supply is
a)​ ​Equal to zero.
b)​ ​Greater than one.
c)​ ​Less than one.
d) Equal to one.

124. Which of the following is not an economic activity?


a)​ ​A chartered accountant doing his own activity.
b)​ ​A teacher teaching in a college.
c) A son looking after his ailing mother.
d)​ ​A manager managing his organization.

125. Production function explains


a) ​The relationship between Qty of inputs employed and the corresponding total production

cost.
b)​ ​The relationship between the firm’s total revenue and total production cost.
c) The relationship between Qty of inputs used and the corresponding output
obtained.
d)​ ​The relationship between market price charged and quantity supplied.

126. In case of short run production function Qty of fixed input remains constant and
a) Qty of either one or two variable inputs change.
b)​ ​Qty of one or two variable inputs are kept constant as Qty of fixed inputs change.
c)​ ​The Qty of both fixed as well as variable inputs remains constant.
d)​ ​The Qty of both variable and fixed input change.

127. In case of long run production


a) The Qty of both fixed and variable inputs are changed.
b)​ ​The Qty of both fixed and variable inputs are kept constant.
c)​ ​The Qty of both fixed and variable inputs are changed in the same proportions.
d)​ ​The Qty of both fixed and variable inputs are changed in different proportions.

128. Williamson's model is an example for


a)​ ​Profit maximization model.
b)​ ​Non- Profit maximization model.
c) Managerial utility model.
d)​ B
​ ehavioral model.

129. Marris model is an example for


a)​ ​Profit- maximizing model.
b)​ ​Non-profit maximizing model.
c)​ ​Behavioral model.
d) Growth maximizing model.
.
130. Average revenue curve of the firm is the same as the demand curve of the consumer except
in the context of
a)​ ​Perfect competition.
b)​ ​Monopolistic competition.
c)​ ​Monopoly.
d) Discriminatory monopoly.

131. What is MR?


a) MR=TRn-TRn-1.
b)​ ​MR=TR/TQ.
c)​ ​MR=TR/TC.
d)​ ​MR=TR/AC.

132. Total revenue will increase with the reduction in price when the price elasticity of demand
for the product is
a) Relatively elastic.
b)​ ​Relatively inelastic.
c)​ ​Unitary elastic.
d)​ ​Perfectly elastic.

133. A market situation where there are only a few large buyers for the product is called
a) Oligopoly.
b)​ ​Pure competition.
c)​ ​Oligopsony.
d)​ ​Duopsony.

134. Demand pull inflation is the result of


a)​ ​Increase in production.
b)​ ​Increase in the supply of goods.
c) Increase in money supply.
d)​ ​Increase in the cost of production.

135. Cash-reserve-ratio refers to


a) ​A ratio between the liquid cash to be maintained by a commercial bank with that of total

deposits of the bank.


b)​ ​A ratio between the total deposits with that of liquid cash maintained by a bank.
c) The % of total deposits to the % of cash that must be invested by a bank as per
instructions of the central bank of the country.
d)​ ​The % of liquid cash to be maintained by a bank with that of its investment.

136. When national income of a country is calculated in terms of constant prices, it is called as
a)​ ​Nominal GNP.
b)​ ​GNP at current prices.
c) GNP at constant prices.
d)​ ​GDP at constant prices.

137. The II phase of the law of variable proportion is described as the most economic region
because of the following reasons except
a)​ ​The producer is employing the most ideal factor combinations.
b)​ ​Total output is highest when marginal product zero.
c) Total output is highest even before the marginal product is zero.
d)​ ​Represents the range of rational production decision.

138. The prices statutorily determined by the government for certain important goods like steel,
cement, etc., is called
a)​ ​Mark-up pricing.
b)​ ​Customary pricing.
c)​ ​Full cost pricing.
d) Administered prices.

139. Suppose a firm is selling 5 units of the output at the price of Rs.15 per unit. Now if it wants
to sell 6 units instead of 5 units and there by the price of the product falls to Rs.14 then the
marginal revenue will be
a) Rs.14.
b)​ ​Rs.10.
c)​ ​Rs.15.
d)​ ​dRs.9.

140. If a production function is homogenous of degree one it implies that


a)​ ​There is decreasing return to scale.
b)​ ​There is increasing return to scale.
c) There is constant return to scale.
d)​ ​None of these.
d)
141. In a monopolistically competitive market,
a) A firm earns economic profits in the long run because it has market power.
b) A firm has market power because it produces a differentiated product.
c) There are a large number of firms.
d) Both a and b
e)​ ​Both b and c
142. A monopolist…
a)​ Always charges a price that is higher than marginal revenue.

b)​ ​Can earn a greater than normal rate of return in the long run.

c)​ ​Can raise its price without losing any sales because it is the only supplier in the market.

d) Both a and b
e)​ ​Both b and c

143. In a monopolistically competitive market,


a)​ ​No firm has any market power.
b)​ ​Firms are small relative to the total market.
c)​ ​There is easy entry and exit in the market.
d)​ ​a and b
e) b and c

144. Which of the following would indicate a relatively large amount of market power?
a)​ ​High demand elasticity
b)​ ​Low Lerner index
c) Low cross-price elasticity with other products
d)​ ​All of the above
e)​ ​None of the above

145. A monopolistic competitor is similar to a monopolist in that:


a)​ ​Both earn positive economic profit in the long run.
b) Both have market power.
c)​ ​Both produce the output at which long-run average cost is at a minimum.
d)​ ​A and b
e)​ ​All of the above

146. Which of the following is a characteristic of a perfectly competitive market?


a)​ ​Firms are price setters.
b)​ ​There are few sellers in the market.
c) Firms can exit and enter the market freely.
d)​ ​All of these

147. If a perfectly competitive firm currently produces where price is greater than marginal cost
it
a) Will increase its profits by producing more.
b)​ ​Will increase its profits by producing less.
c)​ ​Is making positive economic profits.
d)​ ​Is making negative economic profits.

148. When a perfectly competitive firm makes a decision to shut down, it is most likely that
a) Price is below the minimum of average variable cost.
b)​ ​Fixed costs exceed variable costs.
c)​ ​Average fixed costs are rising.
d)​ ​Marginal cost is above average variable cost.
149. In the long run, a profit-maximizing firm will choose to exit a market when
a)​ ​Fixed costs exceed sunk costs.
b)​ ​Average fixed cost is rising.
c) Revenue from production is less than total costs.
d)​ ​Marginal cost exceeds marginal revenue at the current level of production.

150. When firms have an incentive to exit a competitive market, their exit will
a)​ ​Drive down market prices.
b)​ ​Drive down profits of existing firms in the market.
c) Decrease the quantity of goods supplied in the market.
d)​ ​All of the above are correct.

151. In a perfectly competitive market, the process of entry or exit ends when
a)​ ​Firms are operating with excess capacity.
b) Firms are making zero economic profit.
c)​ ​Firms experience decreasing marginal revenue.
d)​ ​Price is equal to marginal cost.

152. Equilibrium quantities in markets characterized by oligopoly is


a)​ ​Lower than in monopoly markets and higher than in perfectly competitive markets.
b)​ ​Lower than in monopoly markets and lower than in perfectly competitive markets.
c)​ ​Higher than in monopoly markets and higher than in perfectly competitive markets.
d) Higher than in monopoly markets and lower than in perfectly competitive markets.
153. In economics the central problem is:
a)​ ​Allocation.
b)​ ​Consumption.
c) Scarcity.
d)​ ​Money.
e)​ ​Production.

154. Indicate below what is NOT a factor of production.


a)​ ​Land.
b) A bank loan.
c)​ ​Labor.
d)​ ​Capital.

155. Macroeconomics deals with:


a)​ ​The behavior of firms.
b) Economic aggregates.
c)​ ​The activities of individual units.
d)​ ​The behavior of the electronics industry.

156. Microeconomics is not concerned with the behavior of:


a) Aggregate demand.
b)​ C​ onsumers.

c)​ ​Industries.
d)​ ​Firms.

157. The study of inflation is part of:


a)​ ​Normative economics.
b) Macroeconomics.
c)​ ​Microeconomics.
d)​ ​Descriptive economics.

158. Aggregate supplies is the total amount:


a)​ ​Produced by the government.
b)​ ​Of products produced by a given industry.
c)​ ​Of labor supplied by all households.
d) Of goods and services produced in an economy.

159. The total demand for goods and services in an economy is known as:
a)​ ​National demand.
b)​ ​Economy-wide demand.
c)​ ​Gross national product.
d) Aggregate demand.

160. If marginal benefit is greater than marginal cost, a rational choice involves:
a) More of the activity.
b)​ ​Less of the activity.
c)​ ​No more of the activity.
d)​ ​More or less, depending on the benefits of other activities.

161. A student chooses to study because the marginal benefit is greater than the ________ cost.
a)​ ​average
b)​ ​total
c) marginal
d)​ ​expected

162. Periods of less than full employment correspond to:


a)​ ​Points outside the production possibility curve.
b) Points inside the production possibility curve.
c)​ ​Points on the production possibility curve.
d)​ ​Either points inside or outside the production possibility curve.

163. The circular flow of goods and incomes shows the relationship between:
a)​ ​Income and money.
b)​ ​Wages and salaries.
c)​ ​Goods and services.
d) Firms and households.
164. In a free market system, the amount of goods and services that any one household gets
depends upon its:
a)​ ​Income.
b)​ ​Wage and interest income.
c)​ ​Wealth.
d) Income and wealth.

165. In a planned or command economy, all the economic decisions are taken by the:
a)​ ​Consumers.
b)​ ​Workers.
c) Government.
d)​ ​Voters.
d)
166. The word that comes from the Greek word for “one who manages a household is
a)​ ​Market.
b)​ ​Consumer.
c)​ ​Producer.
d) Economy.

167. Economics deals primarily with the concept of


a) Scarcity.
b)​ ​Poverty.
c)​ ​Change.
d)​ ​Power.

168. Which of the following is NOT included in the decisions that every society must make?
a)​ ​what goods will be produced
b)​ ​who will produce goods
c) what determines consumer preferences
d)​ ​who will consume the goods
d)
169. In a market economy, characterized by Capitalism, there is full interference by the State in
the economic activities of consumers and producers.
a)​ ​True
b) False

170. In a market system of economy, there is no harmony between individual interests and
interests of the community.
a)​ ​True
b) False

171. Efficiency is the relation between returns and cost.


a) True
b)​ F
​ alse

172. The subject matter of macro –economic includes the theory of income and employment at
an individual level.
a)​ ​True
b) False

173. Rational decision results in…………for the employees.


a)​ ​Division of work.
b)​ ​Centralization.
c)​ ​Discipline.
d) Motivation

174. A Theory Y approach is more suitable where a job offers:


a)​ ​A high degree of intrinsic satisfaction.
b)​ ​The ability to exercise initiative.
c)​ ​An element of problem solving.
d) All of the above.

175. Breech identifies four main elements of management. They are planning, control,
coordination and:
a)​ ​The division of work.
b)​ ​Centralization.
c)​ ​Discipline.
d) Motivation.

176. Many well-known business economists participate in public debates.


a) True
b)​ ​False

177. Marginal Utility is the utility derived from the additional unit of a commodity consumed.
a) True
b)​ ​False

178. Compared to the static model, the fishing effort in a dynamic model is likely to be
a) Larger.
b)​ ​Smaller.
c)​ ​Similar.
d)​ ​Larger or smaller.

179. Land, labor, and money are the three categories of economic resources.
a)​ ​True
b) False
180. Which of the following is not an interest rate derivative used for interest rate management?
Interest rate guarantee
a)​ ​Floor
b)​ ​Swap
c)​ ​Cap
d) All of the above are interest rate derivatives

181. An agreement which guarantees an investor a minimum return on a principal amount is


called a:
a)​ ​Cap
b)​ ​Executive stock option
c)​ ​Stock option
d) Floor

182. Which of the factors listed below is not a reason for decision making in organizations being
a complex process?
a) Modern information systems enable people to evaluate a range of possible outcomes
b)​ ​Factors in the current context of the organization affect the decision
c)​ ​Several stakeholders will have an interest in the decision
d)​ ​People have to make decisions in a historical context

183. The overall process of decision making in, for example, staff selection includes which of
these stages?
a)​ ​Deciding which candidate to appoint
b)​ ​Identifying the need for a new member of staff
c)​ ​Agreeing the job specification
d) All of the above

184. A manager who is helping a customer return some shoes they purchased last week is
dealing with what type of decision?
a)​ ​Uncertainty
b)​ ​Non-programmed decision
c)​ ​Bounded rationality
d) Programmed decision

185. Decision making situations can be categorized along a scale which ranges from:
a)​ ​Uncertainty to certainty to risk
b)​ ​Certainty to uncertainty to risk
c)​ ​Certainty to risk to uncertainty
d) Certainty to risk to uncertainty to ambiguity

186. The quantity demanded of Pepsi has decreased. The best explanation for this is that:
a) The price of Pepsi increased.
b)​ P​ epsi consumers had an increase in income.

c)​ ​Pepsi's advertising is not as effective as in the past.


d)​ ​The price of Coca Cola has increased.

187. Demand curves are derived while holding constant:


a)​ ​Income, tastes, and the price of other goods.
b)​ ​Tastes and the price of other goods.
c)​ ​Income and tastes.
d) Income, tastes, and the price of the good.

188. When the decrease in the price of one good causes the demand for another good to decrease,
the goods are:
a)​ ​Normal
b)​ ​Inferior
c) Substitutes
d)​ ​Complements

189. If the demand for coffee decreases as income decreases, coffee is:
a)​ ​An inferior good.
b) A normal good.
c)​ ​A complementary good.
d)​ ​A substitute good.

190. Which of the following will NOT cause a shift in the demand curve for compact discs?
a)​ ​A change in the price of pre-recorded cassette tapes.
b)​ ​A change in income.
c) A change in the price of compact discs.
d)​ ​A change in wealth.
d)
191. The demand for a product or a service depends on a host of factors.
a) True
b)​ ​False

192. Demand curves may also be shifted by changes in expectations.


a) True
b)​ ​False

193. Quantity demand is a specific quantity that buyers are willing and able to buy at a specific
demand price.
a) True
b)​ ​False

194. If the price of a complement increases, all else equal,


a)​ ​Quantity demanded will increase.
b)​ ​Quantity supplied will increase.
c)​ ​Demand will increase.
d) Demand will decrease.

195. Which of the following would lead to an INCREASE in the demand for golf balls?
a)​ ​An decrease in the price of golf balls.
b)​ ​An increase in the price of golf clubs.
c)​ ​A decrease in the cost of producing golf balls.
d) An increase in average household income when golf balls are a normal good.
d)
196. If the price elasticity of demand for a good is .75, the demand for the good can be described
as:
a)​ ​Normal
b)​ ​Elastic
c)​ ​Inferior
d) Inelastic.
197. If it is observed that, in a particular market, price has risen and quantity exchanged has
increased, it is likely that:
a)​ ​Supply has increased.
b)​ ​Supply has decreased.
c) Demand has increased.
d)​ ​Demand has decreased

198. For a given normal supply curve, the amount of a tax paid by the buyer will be larger
a)​ ​The more elastic the demand.
b) The more inelastic the demand.
c)​ ​The income elasticity is equal to zero
d)​ ​When the price is high.
d)
199. Most operations produce a mixture of both products and services. Which of the following
businesses is closest to producing “pure” services?
a)​ ​IT company
b)​ ​A Restaurant
c) Counsellor/therapist
d)​ ​Steel company

200. Economists typically assume that the owners of firms wish to


a)​ ​Produce efficiently.
b)​ ​Maximize sales revenues.
c)​ ​Maximize profits.
All of these.

Q1: opec is an example of


A producers cooperative
B trust
C marketing board
D cartel
Ans. cartel

Q2: if the price is statutorily fixed and equal to IMC ,monopoly profits will be
A at same level
B decreased
C eliminated
D increased
Ans At same level

Q4 if a demand curve exhibits unit elasticity for all prices the MR curve
A is identical with it
B lies below demand curve
C is identical with X axis
D. is identical with Y axis
Ans: lies below the demand curve

Q5 which is not a fixed cost?


A monthly rent of $1000 contractually specified in one yr lease
B an insurance premium of $50 per year paid last month
C an attorney’s retainer of $50000 per year
D a workers wage of $15 per hour
Ans: a workers wage of $15 per hour

Q7 the total utility is maximum when


A MU IS ZERO
B MU IS HIGHEST
C MU=AU
D AU IS HIGHEST
Ans: MU IS ZERO

Q8 when a firm doubles its inputs and finds that its output has more than doubled this is known as
A economics of scale
B constant returns to scale
C diseconomies of scale
D a violation of law of diminishing returns
Ans: Economies of scale.

Q9 variable costs are

A sunk costs
B multiplied by fixed costs
C defined as change in total cost resulting from production of an additional unit of output
D costs that change with level of production
Ans : D costs that change with the level of production

Q10-: if a firm’s revenues just cover all its opportunity costs then
Total revenues equal to its implicit costs
Economic profit is zero
Normal profit is zero
Total revenues equal to its explicit costs
Ans: Economic profit is zero

Q11.the law of diminishing returns only applies in cases where

A there is increasing scarcity of factors of production


B the price of extra units of factor is increasing
C there is at least one fixed factor of production
D capital is a variable input
Ans: there is at least one fixed factor of production

Q12 price control is one of the monopoly regulations which is most advantageous for

The government
B the producer
C the consumer
D no one
Ans: the producer

Q13 an indifference curve is always

A a vertical straight line


B convex to the origin
C concave to the origin
D a horizontal straight line
Ans. convex to the origin

Q14 on an indifference map the income consumption curve spices downwards to right it shows that
A both X and Y are superior goods
B Y is an interior good
C X is an interior good
D both X and Y are interior goods
Ans: X is an inferior good

Q15. A perfectly competitive industry becomes a monopoly with same cost conditions it will now sell
A larger output to a higher price
A larger output to the old price
An unchanged output to higher price
A reduced output at higher price
Ans. A reduced output at higher price

Q16 in the circular flow of income model injections


A are assumed to be exogenous
B are assumed to be a function of national income
C decrease aggregate demand
D decrease the investment into an economy
Ans. ​Are assumed to be exogenous

Q17-: Implicit costs are


A equal to total fixed costs
B comprised entirely of variable cost
C payments for self employed resources
D always greater in the short run than in long run
Ans. payments for self employed resources

Q18:demand pull inflAtion may be caused by


A increase in cost
B reduction in interest rates
C reduction in govt. spending
D outward shift in aggregate supply
Ans. reduction in interest rates

Q20 if a more efficient technology was discovered by a firm there would be


A. an upward shift in AFC curve
B. a downward shift in MC curve
C. an upward shift in AVC curve
D. a downward shift in AFC curve
Ans. a downward shift in MC curve

Question 1 what does price elasticity of demand measure?


A. Change in price caused by changes
B. The rate of change of sales
C. The responsiveness of demand to price
D. Value of sales at given price
Ans. A

Question 2 which is not a central problem of an economy?


A. What to produce
B. How to produce
C. How to maximize power profits
D. Whom to produce
Ans. B

Question 3 which of the following describes the law of supply?


A. Supply curve
B. Supply schedule
C. Supply equation
D. All of above
Ans. A

Question 4 A desire backed by ability to pay and willingness to pay for a commodity is called?
A. Want
B. Purchasing power
C. Perfect desire
D. demand
Ans. D

Question 5 income elasticity of demand for normal good is always:


A. 1
B. More than 1
C. Negative
D. positive
Ans. C

Question 6 during a particular year farmers experienced a dry weather , if all other factors remain constant
, farmers supply curve for wheat will shift to
A. Rightward
B. Leftward
C. Upward
D. downward
Ans. B

Question 7 An increase in demand would cause supply curve to?


A. Shift to left
B. Shift to right
C. Change in slope of supply curve
D. No effect on supply
Ans. B

Question 8 which one is increasing function of price


A. Demand
B. Utility
C. Supply
D. consumption
Ans. C

Question 9 Implicit demand is also known as?


A. Derived demand
B. Direct demand
C. Composite demand
D. Joint demand
Ans. B

Question 10 Total utility is maximum when -:


A. Marginal utility is zero
B. Marginal utility is at its highest point
C. Marginal utility is equal to average
D. Average utility is maximum
Ans A

Question 11 according to Joseph Schumpeter profit is reward for?


A. Innovation
B. Uncertainty bearing
C. Risk taking
D. management
Ans. B

Question 12 the term “ of equilibrium of the firm mal profit “ as used in the analysis of equilibrium of
firm under perfect competition refers to:
A. Earnings for management
B. Reward for enterprise
C. Reward for innovation
D. Residential income for business
Ans. A

Question 13 which is not an essential condition for an economic problem to arise?


A. Unlimited wants
B. Use of money
C. Scarcity of resources
D. Alternative uses of scarce resources
Ans. D

Question 14 if price changes by 1% and supply changes by 2%


A. Elasti
B. Inelastic
C. Indeterminate
D. static
Ans. A

Question 15 if elasticity of supply is greater than one .Supply curve will be


A. Horizontal
B. Vertical
C. Passing through origin
D. Touching Y-axis
Ans. C

Question 16 the demand for salt is?


A. Elastic
B. Inelastic
C. Zero
D. maximum
Ans. B

Question 17 if price and total revenue move in the same direction then demand is
A. Inelastic
B. Elastic
C. Unrelated
D. Perfectly elastic
Ans. B
Question 18 Who argued that pure profit can arise only in a dynamic economy?
A. f.H.knight
B. J.b clark
C. Bohm Bawerk
D. Alfred Marshall
Ans. B

Question 19 Indirect demand is also known as?


A. Direct
B. Composite
C. Joint
D. derived
Ans. D

Question 20 In which of the following cases the consumer’s surplus would be the most ?
A. Giffen goods
B. Necessities
C. Luxuries
D. Prestige goods
Ans. B

1. CONSIDER THE FOLLOWING

1. Large no. of buyers and sellers

Firms produce differential products

3. Free entry and exit of firms

4.. Perfect knowledge about tech.

Which is characteristics of monopolistic competition?

A. 1 AND 3
B. 2 AND 3
C. 2,3 AND 4
D. 1,2,3,4

ANS-C

2. THE DIFFERENCE BETWEEN MONOPOLY EQUILIBRIUM


A. THE MC SHOULD RISE AT THE POINT OF EQUILIBRIUM UNDER
B. UNDER PERFECT COMPETITION
C. THERE IS NO DIFFERENCE AT ALL
D. NONE

ANS-B UNDER PERFECT COMPETITION

3. The value of sigma for cobb douglas production function is

A. Infinity
B. 0
C. 1
D. -1

ANS-D - 1

4.the equilibrium level of output for pure monopolist is where

A. MR=MC
B. P<AC
C. MR<MC
D. MR>MC

Ans-a MR=MC

5. THE imposition of ceiling on monopolists price will affect his

A. Equilibrium output only


B. Average revenue in short
C. Profits only
D. Equilibrium output and profits

Ans-C profits only

6. A monopolist charging high price operates on

A. The constant elastic part of demand curve


B. The inelastic part of demand curve
C. The elastic part of demand curve
D. Ignores elasticity of demand altogether

ANS-C

7. Monopoly exploitation is reduce by regulation that

A. Restricts output
B. Enhances product market competition
C. Increases bargaining power of workers
D. Increases bargaining power of employees

ANS-C

8. Given that cost conditions are same in monopolistic competition price is ...and output is …...as
perfect competition

A. Higher,same
B. Same,lower
C. Ans. Higher,lower
D. Lower,higher

Ans c

9. Which is true at equilibrium in monopolistic competition?

A. Price is greater than marginal cost


B. Average revenue is equal to marginal revenue
C. Both a and b
D. Avg revenue equals to marginal revenue

Ans d

10.bilateral monopoly means

A. A monopoly seller buying his input


B. Two rival buyers only
C. Two rival sellers only
D. A monopolist facing monopolist
Ans b

11.the market period supply curve for perishable commodities is

A. Perfectly elastic
B. Perfectly inelastic
C. Relatively elastic
D. Relatively inelastic

Ans b

12.the degree of monopoly power can be measured by formulae

A. (P-MC)/P
B. MR/(AR-MR)
C. AR/(AR-MR)
D. (AR-MR)/MR

Ans d

13. On an indifference map higher indiff curves show

A) Levels of satisfaction among which


B) The optimum level of satisfaction
C) The higher level of utility
D) D)Same lower level of satisfaction

Ans b

14. A monopolist will fix the equilibrium output of his product where the elasticity of his AR
curve is

A. Equal to less than one


B. Greater than or equal to one
C. Less than one but more than zero
D. Zero

Ans-c
15. Which of following is assumption of model of perfect competition?

A. Product homogeneity
B. No. govt. Regulation
C. Perfect mobility of factors of production
D. All of above

Ans-a

16.when a monopolist is in

A. Long run equilibrium, the mayor may not be in short run


B. Long run equilibrium he will also be in short run equilibrium
C. Short run equilibrium he will also be in long run equilibrium
D. none

Ans-d

17. The law of diminishing returns depends on assumption that

A. Land is factor kept constant


B. The state of technical knowledge is unchanged
C. Total output is constant.
D. Avg. output declines faster than marginal output

Ans ​a

18.under monopoly and imperfect competition MC IS

A. Equal to price
B. Less than price
C. more than price
D. Any of above

Ans a

19. The law of diminishing returns is applied to all fields of production was stated by

A. AC pick
B. Walras
C. Alfred marshall
D. David ricardo

Ans d

20.monopoly equilibrium can be reached when

A. Marginal cost is rising


B. Marginal cost is remaining constant
C. Marginal cost is falling
D. All of above

Ans b

1. What is Economics?

Ans: Both A and B

2. What is the scope of managerial economics?

Ans: All

1. What is Scarcity?

Ans: Unlimited want and limited resources

2. What is Efficiency?

Ans: Allocating scare resources to its highest satisfaction

1. What is the opportunity cost?


Ans: NEXT BEST>>>>

2. What is the Production possibility curve?

Ans: Both A and B

1. What is the difference b/w managerial economics and economics?

Ans: All

2. What is the nature of managerial economics?

Ans: All

Question 1

Strategy A has an expected value of 10 and a standard deviation of 3. Strategy B has an expected
value of 10 and a standard deviation of 5. Strategy C has an expected value of 15 and a standard
deviation of 10. Which one of the following statements is true?

a)​ "​ A risk averse decision maker will always prefer A to B, but may prefer C to A"

b)​ A
​ risk neutral decision maker will always prefer C to A or B

c)​ A
​ risk seeking decision maker will always prefer C to A or B

d) All of the above are correct


Question 2

Benefits of the opportunity cost approach to individual decision making:

a) ​focuses on the economic ramifications of choices and expresses what is given up (a

net benefit) by making a choice.

b)​ "​ Opportunity costs are subjective, existing in the minds of decision makers."

c) ​"Economic profit would be total revenue minus all costs, including both explicit costs
of purchased inputs and implicit (opportunity) costs of owner provided resources. "

d) All of the above

Question 3

"If a person's utility doubles when their income doubles, then that person is risk"

a)​ a​ verse

b)​ n​ eutral

c)​ s​ eeking

d)​ N
​ one of the above
Question 4

A situation in which a decision maker must choose between strategies that have more than one
possible outcome when the probability of each outcome is unknown is referred to as

a)​ d
​ iversification

b)​ c​ ertainty

c)​ r​ isk

d) Uncertainty

Question 5

Which one of the following does measure risk?

a)​ C
​ oefficient of variation

b)​ S
​ tandard Deviation

c)​ E
​ xpected value

d)​ A
​ ll of the above

Question 6

"If a decision maker is risk averse, then the best strategy to select is the one that yields the"
a)​ h
​ ighest expected payoff

b)​ l​ owest coefficient of variation

c) highest expected utility

d)​ l​ owest standard deviation

Question 7

Circumstances that influence the profitability of a decision are referred to as

a)​ s​ trategies

b)​ p​ ayoff matrix

c)​ s​ tates of nature

d)​ m
​ arginal utility of money

Question 8

"If the production possibilities frontier is not bowed out but is a line, indicates there is"

a)​ S
​ carcity

b)​ C
​ onstant opportunity cost

c)​ U
​ nemployment

d)​ I​ ncreasing Opportunity Cost


Question 9

What is the least amount of money you would have to value seeing Kwan in order for you to
choose the holiday on Ice Show?

a)​ $
​ 0

b)​ $​ 7

c)​ $
​ 38

d)​ $
​ 48,olk

Question 10

Basic concept used under this study is:

a)​ M
​ arginalism

b)​ O
​ pportunity Cost

c)​ I​ ncremental Cost

d)​ N
​ one of the above

1. Which one of the following industries comes closest to perfect competition?

Ans: Wheat farming


2. Economic profits are generally.

Ans: b

1. Consider a monopolistically competitive firm. From the point of view of remaining firms,
as firm leave the industry we can think of this as a:

Ans: Shift back in each individuals firm’s demand curve

2. In monopolistically competitive industry

Ans: c

1. A defining characteristics of a natural monopoly is that

Ans: c

2. An Unregulated monopoly will:

Ans: a

1. Which of the following is not a characteristics of an oligopoly market?

Ans: Entry is relatively easy

2. Where the industry strategy is set by small firm recognized as having good market
intelligence?
Ans: Barometric leadership

1. Two firm compete in a stackelberg fashion and firm two is the leader, then

Ans: firm one view the output of firm two given

2. When firm one act as a Stackelberg leader:

Ans: None

1. Student railcards are an example of:

Ans: Third degree price determination

2. __________________is the best strategy to extract consumer surplus:

Ans: First degree price discrimination

1. Which of the following is a characteristic of a perfectly competitive market?

Ans: c

2. It is perfectly competitive firm currently produces where price is greater than marginal
cost it?

Ans: Will increase its profits by producing more


1. When a perfectly competitive firm makes a decision to shut down, it is most likely that

Ans: Price is below the minimum of average variable cost

2. In the long run, a profit-maximizing firm will choose to exit a market when

Ans: c

1. When firms have an incentive to exit a competitive market, their exit will

Ans: c

2. In a perfectly competitive market, the process of entry or exit ends when

Ans: b

1. When a markets are imperfect and exhibit externalities:

Ans: There is an inefficient allocation and use of society’s scarce resources.

2. The social cost of monopoly is:

Ans: b

1. In A competitive labour market, the demand curve for labour is given by which one of
the following?
Ans: Marginal revenue product

2. A monopsony type of market failure is said to be occur for which one of the following?

Ans: Buyers of labour have market power

1. Which two condition are most likely to read to a rise in union bargaining power?

Ans: Rise in the percentage of the workforce in the union

2. A European directive which helps establish employer consultation procedures for


multinational companies in the EU is which one of the following?

Ans: The European Works Council Directive

1. Which one of the following describe a state of the labour market which is often linked to
a lower wage for those employed?

Ans: Monopsony

2. Where the return to the factor is greater than is needed for it to supply itself?

Ans: economic rent

1. Firms in perfect competition faces a:

Ans: Perfect elastic demand curve

2. In perfect competition:
Ans: The price charged by a firm equals the marginal revenue

Perfect Competition -2

1. A profit maximizing firm in perfect competition produces where?

Ans: c

2. In perfect competition?

Ans: The product firm offer very similar

1. In the long run in perfect competition:

Ans: c

2. In perfect competition?

Ans: b

1. In perfect competition?

Ans: There are many buyers and sellers

2. In the short run firms in perfect competition will still produce provided:

Ans: The price covers average variable cost


1. In the long run equilibrium in perfect competition:

Ans: Price=average cost=marginal cost

2. For a perfectly competitive firm:

Ans: Total revenue is straight line

1. Over the last 15 or 20 years widespread computerization has generally led to

Ans: more competition and lower price

2. The perfect competitor’s demand curve is _________a horizontal line.

Ans: Always

Perfect Competition -7

1. The perfect competitor’s price is set by

Ans: market supply and demand

2. In the short run the perfect competitor/

Ans: may make a profit or lose

1. When the perfect competitor is taking a loss, over the long run some firms will
Ans: c

2. The perfect competitor always attains peak efficiency in the

Ans: long run

1. Advertising can represent a _______cost.

Ans: sunk

2. Strategic interaction only occurs in __________.

Ans: oligo

1. In a prisoner’s dilemma with prisoners A and B, if they both confess, A gets 5 years and
B gets 8 years. If both remain silent, A gets 2 years and B goes free. If one confesses and the
other does not, the one who confesses gets 1 year and the other get 15 years. Which statement is
true of this case?

Ans: C

2. In baseball game if a pitcher appears to have thrown at an opposing batter it is generally


understood that someone one that pitcher’s team will be thrown at happen relatively infrequently
over a season. Once each team has made its point the incident is usually forgetting.

Ans: tit for tat

1. Profit is maximum when:

Ans: distance bw tr and tc is maximum


2. Profit is maximum when:

Ans: slope of tc and tr are same

1. The profits of the leader in a stackelberg duopoly?

Ans: Are greater than those of the followers

2. Economist use game theory to predict the behaviour of oligopolists. Which of the
following is crucial for the success of the analysis?

Ans: All of the above

1. Suppose two type of consumers buy suits. Consumers of type A will pay $100 for a coat,
and $50 for pants. Consumers of type B will pay $75 for a coat, and $75 for pants. The firm
Selling suits faces no competition and has a marginal cost of zero. If the firm sells coats and
pants for $25 each, but offers a bundle containing both a coat and paints for $1.50, how many
bundles will the firm sell?

Ans: 0

2. Suppose two type of consumers buy suits. Consumers of type A will pay $100 for a coat,
and $50 for pants. Consumers of type B will pay $75 for a coat, and $75 for pants. The firm
Selling suits faces no competition and has a marginal cost of zero. Strategy is?

Ans: charge $150 for a suit

1. A Broadway theater sells weekday shows tickets at a lower price than for a weekend
show.

This is an example of?


Ans: All

2. Which of the following statements about a price matching strategy is incorrect?

Ans: b

1. A stable equilibrium in the prisoner’s Dilemma is known as:

Ans: Nash

2. An example of a two person, noncooperative, zero sum game is:

Ans: Beach kiosk

1. If firm A cheats on the cartel and firm B complies with the agreement, Firm A's profit is”

a.​ $ 3 million

b.​ $​ 2 million

c.​ Zero

d.​ (​ -) $ 1 million

Question 2

"If firm A cheats on the cartel and firm B complies with the agreement, firm B's profit is”
a.​ $ 3 million

b.​ $​ 2 million

c.​ Zero

d.​ (​ -) $ 1 million

Question 3

"If this game is played only once,”

a.​ both firms A and B will cheat


b.​ F
​ irm A will cheat and Firm B will not cheat

c.​ Firm A will not cheat and Firm B will cheat


d.​ n​ either Firm A nor Firm B will cheat

Question 4

The equilibrium in the previous question is called

a.​ Credible strategy equilibrium


b.​ N
​ ash Equilibrium
c.​ Duopoly Equilibrium

d.​ C
​ ooperative equilibrium

Question 5

If this game is played repeatedly and both firms adopt trigger strategies so that the cooperative
equilibrium emerges

a.​ both firms A and B will cheat


b.​ F
​ irm A will cheat and Firm B will not cheat

c.​ Firm A will not cheat and Firm B will cheat


d.​ n​ either Firm A nor Firm B will cheat

Question 6

Which one of the following is a part of every game theory model?

a.​ Players

b.​ P
​ ayoffs

c.​ Probabilities

d.​ S
​ trategies

Question 7

"In game theory, a choice that is optimal for a firm no matter what its competitors do is referred
to as"

a.​ dominant strategy


b.​ g​ ame-winning choice

c.​ Super optimal


d.​ a​ gonzo selection

Question 8

A prisoners' dilemma is a game with all of the following characteristics except one. Which one is
present in a prisoners' dilemma?

a.​ Players cooperate in arriving at their strategies.


b.​ B
​ oth players have a dominant strategy.

c.​ Both players would be better off if neither chose their dominant strategy.

d.​ T
​ he payoff from a strategy depends on the choice made by the other player.
Question 9

"Which of the following legal restrictions, if enforced effectively, would be likely to solve a
prisoners' dilemma type of problem for the firms involved?"

a.​ A law that prevents a cartel from enforcing rules against cheating.

b.​ A
​ law that makes it illegal for oligopolists to engage in collusion.

c.​ A law that prohibits firms in an industry from advertising their services.

d.​ A
​ ll of the above would be likely to solve a prisoners' dilemma for the firms.

Question 10

"Until recently, medical doctors and lawyers have been prohibited from engaging in competitive
advertising. If the prisoners' dilemma applies to this situation, then the presence of this restriction
would be likely to"

a.​ increase profits earned by individuals in these professions.


b.​ r​ educe profits earned by individuals in these professions.

c.​ have no effect on the profits earned by individuals in these professions.


d. increase the profits of some and reduce the profits of other individuals in

these professions.

2. under perfect competition a firm can produce with


An optimum plant
Identical products at low cost
Maximum profit
An optimum output
Ans: Maximum profit

4. match the following:

A.increasing cost industry 1.horizontal long run supply curve


b. decreasing cost industry 2.positively sloped long run supply curve
C. constant cost industry 3.negatively sloped long run supply curve
ABC
321
Abc
123
ABC
231
ABC
213

Ans: A B C
231

6. other things being equal an increase in supply can be caused by


A rise in income of consumer
An improvement in techniques of production
A rise in price of commodity
An increase in income of seller
Ans: rise in price of commodity

7. consider following
1.in a capitalist economy there is private ownership of means of production
2. in a communist nation the means of production are owned bt the state
3.in a free market economy there is minimum role of govt.
Which of these are true?
Onlu 1 and 3
Only 2 and 3
Only 3
All
Ans: All

8. FROM THE resource allocation point of view point perfect competition is preferable because
There is no more restrictions on entry and exit of firms
There is whole variety of output produced
The firms operate at excess capacity levels
There is no idle capacity
Ans: There is no more restrictions on entry and exit of firms

9. For equilibrium in an open four sector economy


Actual injections=actual withdrawls
Planned injections=planned withdrawls
savings=investment
Govt spending = tax revenue
Ans:Planned injections=planned withdrawals

10. which of the following is correct

1. The slope of isoquants represent MRTS


2. The mtrs of inputs x and y =MPx/MP y
3. The elasticity of substitution between two inputs x and y is proportional change in ratio of two
inputs divided by proportionale change in MRTS
4. If degree of homogeneity is greater than one the production function is increasing returns to fixed
factor
● 2 3 and 4
● 1,3,4
● 1,2,4
● 1,2,3
Ans: 1,2,3

11. When tr curve and Tc curve are parallel and TR exceeds TC


Total profit is minimized
Normal profit is minimized
Total profit is maximized
Normal profit is maximized
Ans. Total profits is maximized

12. In monopoly the relationship between average revenue and marginal revenue curves is as follows
Average revenue curve lies above the MR curve
AR curve lies below the MR curve
All curve coincides with MR curve
All curve is parallel to Mr curve
follows

Ans: Average revenue curve lies above the MR curve

13. the revealed performance theory is based on


Introspection
Utility and demand
The assumption of indifference
Observed consumer behaviour

Ans: The assumption of indifference

14. if marginal cost is above average variable at a time when output is rising then
Average variable cost is failing
Average variable cost is rising
Average total cost is failing
Average total revenue is failing
Ans: Average variable cost is rising

15. The records of exports and imports in goods and services and transfer payments is known as

Current account
Budget surplus
Economic leakage
Degree of openness
Ans: Current account

16. macroeconomies is a study of economics the deals with which 4 major factors
Households,firms , govt and demand supply
Households,firms, govt and external sector
Firms, govt , free market and regulations
None of the above
Ans: Households,firms, govt and external sector

17. direct regulation of business has the potential to yield economic benefits to society when

Diseconomies of scale exist


Barriers to entry are exit
There are not good substitutes for a product
Many firms serve a given market
Ans. There are not good substitutes for a product

20. inflation
Always reduces the cost of thing
Always reduces the standard of living
Reduces the price of products
Reduces the purchasing power of money
Ans. Reduces the purchasing power of money

1. The opportunity cost of machine which can produce only one product is:

Ans: infinite

2. The PPF shows:

Ans: b
Consumer Behavior: Demand Function

1. What is law of Demand?

Ans: Inverse relationship b/w price and demand of the commodity

2. Why demand curve Slopes downwards?

Ans: Both A and B

1. Elasticity of demand is determined by all the following factor except:

Ans: Govt. policies

2. Income Elasticity of demand for diamond would be?

Ans: >1

1. What is cross Elasticity of Demand?

Ans: Percentage change in demand for one good to the percentage change in the price of some
other related good.

2. Demand of electricity in elastic because

Ans: none
1. What is Econometric method?

Ans: Combination of economic tools with statistical tools too estimate economic variable

2. What is Barometric method of forecasting?

Ans: a

1. Consider two goods X and Y. There was no change in price of X, but its demand was
seen to fall from 6000 units to 5500 units.

Ans: -0.75

2. Consider two goods X and Y. There was no change in price of X, but its demand was
seen to fall from 6000 units to 5500 units.

Ans: 0.82

1. What is elasticity of supply?

Ans: Percentage change in quantity supplied due to one percent change in price

2. A Vertical Supply curve means that elasticity of supply?

Ans: zero

1. A typical indifference curve:

Ans: b
2. A utility function shows the relation b/w

Ans: The amount of good consumed and consumers

1. As the consumption of coke increase, its marginal utility to the drinker will:

Ans: Decrease

2. In the world of just two goods, where all income is spent on the two goods, both of them
cannot be inferior?

Ans: True

1. Consumer is an equilibrium when marginal utilities are the consumption of coke increase,
its marginal utility to the drinker will:

Ans: Equal

2. In economics, one or more persons sharing common consumers budget is called?

Ans: house hold

1. Ordinal utilities Analysis was developed by

Ans: JR hick and RJ

2. “Utility or satisfaction is a subjective concept; therefore it could only be ranked”. The


Statement support?

Ans: b
1. Substitutions effects takes place when price of the commodity becomes;

Ans: both a and b

2. Different quantity purchased at different possible of a commodity is called:

Ans: Demands schedule

1. Diagrammatic presentation of demand schedule of an individual buyer of a commodity in


the market yield?

Ans: individual demand curve

2. Goods are undemanded because these possess:

Ans: utility

1. When price is below equilibrium level there will be:

Ans: Shortage of commodity in market

2. Price of product is determined in a free market;

Ans: by both demand and supply

1. Diagrammatic presentation of demand schedule of an individual buyer of a commodity in


the market yields
Ans: individual curve

2. Goods are undemand because these possess

Ans: utility

1. Which best describes consumer surplus?

Ans: d

2. Which of the following Statement is NOT true?

In the free market change in the price of a product?

Ans: d

1. Community Surplus equals:

Ans: Consumer Surplus plus producer surplus

2. Monopoly power in a market is likely to?

Ans: c

1. If the cross elasticity of demand is -2:

Ans: c
2-The cross price elasticity b/w two products is found to be -1/2. From this you know that the
two products are:

Ans: complem

1. If the cross price elasticity between goods B and A is -2 and the price of good B increase
by 5%, the quantity demanded of good A will:

Ans: decreased by 10%

2. If a coffee and tea are substitutes, what do we know for certain about the cross elasticity
of demand for coffee with respect to the price of tea?

Ans: c

1. Which of the following is likely to have the smallest price elasticity of demand?

Ans: an automobile

2. A 10 % decrease in the price of a Pepsi decreases the demand for a Coca-Cola by 50 %.


The cross elasticity of demand b/w a Pepsi and Coca-Cola is?

Ans: 5

1. A rise in the price of a product lowers the total revenue from the product if the

Ans: demand for the product is elastic

2. If a 4% rise in the price of peanut butter lowers the total revenue received by the
producers of peanut butter by 4%, the demand for peanut butter?
Ans: b

1. If the demand for a good is inelastic, an increase in its price will cause the total
expenditure of the consumers of the good to:

Ans: increase

2. The horizontal demand curve parallel to x-axis implies that the elasticity of demand is:

Ans: b

1. When the unit price of product X decrease from $6 to $5, its price elasticity of supply is:

Ans: inelasticity

2. The feedback of ‘Anti-Smoking campaign for the youth’ is very successful under huge
promotions

Ans: to supply of cigrete is inelastic

1. The price elasticity of demand is:

Ans: a

2. If demand is price elastic, then.

Ans: b
1. The demand for movies is unit elastic if

Ans: c

2. Unit elastic demand

Ans: c

1. Moving up along a linear demand curve, the price elasticity of demand

Ans: increase

2. If the price elasticity of demand for a product equals 1, as its price rises the

Ans: c

1. Suppose that the quantity of root beer demanded declines from 103,000 gallons per week
to 97,000 gallons per week as a consequence of a 10 percent increase in the price of root beer.
The price elasticity of demand is

Ans: 0.60

2. The price elasticity of demand is 5.0 if a 10 % increase in the price results in a


_______decrease in the quantity demanded.

Ans: 50%

1. Income elasticity of demand is defined as the responsiveness of:

Ans: Quantity demand to change a income


2. If the income elasticity of a demand for a good is negative, then the good is:

Ans: an inferior good

1. The income elasticity of demand is high

Ans: Luxuries

2. A Product is likely to have a price elasticity of demand that exceeds 1 when

Ans: it has close substitute

1. The income elasticity of demand is the percentage change in

Ans: b

2. Demand is income elastic if

Ans: b

1. If the price elasticity of demand for some good is estimated to be 4. Then a 1% increase
in price will lead to a:

Ans: 4 % decrease in quantity demand

2. If the price elasticity of demand is unit then a fall in price:

Ans: Reduce costs


1. The price elasticity of demand is a negative number this means:

Ans: c

2. Price increase from 10 to 12 pence and the price elasticity demanded was 500 units. What
will it be now?

Ans: c

1. If demand is price inelastic?

Ans: b or c

2. For an inferior good with a downward sloping demand curve:

Ans: b

1. If an average curve has a negative slope, then the corresponding

Ans: b

2. If a firm total cost curve is defined by a straight line that has a positive intercept that is
equal to fixed cost, then

Ans: c

1. Equilibrium:
Ans: a

2. A rise in supply and demand in equal proportion will result in:

Ans: c

Question 1

Market - clearing price and qunatity using simultaneous equations will be:

a)​ "​ P = 4, Q = 6"

b)​ "​ P = 3, Q = 2"

c)​ "​ P = 6, Q= 4"

d) "P = 5, Q = 16"

Question 2

Suppose P = 7. Calculate the ecess. Is it a shortage or surplus?

a)​ "​ 8, Surplus"

b)​ "​ 10, Surplus"

c)​ "​ 8, shortage"

d)​ "​ 10, shortage"


Question 3

Suppose P = 2. Calculate the ecess. Is it a shortage or surplus.

a)​ "​ 6, surplus"

b)​ "​ 15, shortage"

c)​ "​ 15, surplus"

d)​ "​ 6, shortage"

Question 4

"If price falls from $250 to $200, what is the elasticity of demand over this range?"

• -0.08

• -1.5

• -1

• -0.67

Question 5

"As output increases from 2,100 to 2,700 what is marginal revenue?"

a)​ (​ -) $ 25

b)​ 2​ 5
c)​ 5
​ 0

d)​ (​ -) $300

Question 6

"If price falls from $250 to $200,"

a) ​an arrow representing the price effect points down and is longer than an arrow for the
quantity effect

b)​ a​ rrows representing the price and quantity effects both point up

c)​ a​ rrows representing the price and quantity effects both point down

d) ​an arrow representing the price effect points down and is shorter than an arrow for the
quantity effect

Question 7

If price falls from $200 to $150

a) ​an arrow representing the price effect points down and is shorter than an arrow for
the quantity effect

b) ​total revenue moves in the same direction as the arrow representing the quantity

effect

c)​ t​ otal revenue moves in the same direction as the arrow representing the price effect

d)​ a​ rrows representing the price and quantity effects both point down
Question 8

"If the own-price elasticity of demand for a good is -0.6 and quantity demanded decreases by
30%, price must have"

a)​ d
​ ecreased by 0.6%

b)​ d​ ecreased by 18%

c)​ i​ ncreased by 50%

d)​ i​ ncreased by 20%

Question 9

Which of the following would tend to DECREASE the elasticity of demand for good X?

a)​ C
​ onsumers begin spending a smaller percentage of their income on X

b)​ N
​ ew substitutes for X become available from other firms

c)​ T
​ he cost of producing X declines

d)​ b
​ oth b and c

Question 10

Which of the following would tend to INCREASE the elasticity of demand for good X?
a)​ a​ new discovery allows firms to produce X at a much lower cost.

b)​ b​ oth b and c

c)​ "​ a new product, Y, which can be used in place of X, is introduced"

d)​ t​ he percentage of a consumer's income spent on good X increases.

1. Which of the following is considered production in economics?

Ans: Tiling of soil

2. Which of the following is not a characteristic of land?

Ans: it is produced by our forefathers

1. A production function measures the relationship b/w

Ans: c

2. Is the marginal product is decreasing, then marginal product.

Ans: Must be greater than AP

1. L-shaped isoquants imply that production requires that the inputs are perfect substitutes.

Ans: C

2. Isoquants that are downward sloping straight line exhibit?


Ans: A constant marginal rate of technical substitution

1. Internal economies of scale occur when?

Ans: b

2. Diseconomies of scale are a sign that

Ans: c

1. Economies of scale exist when

Ans: c

2. Economies of scope exist when

Ans: c

1. Which of the following statement is true regarding the difference between economic and
accounting cost?

Ans: d

2. Which of the following costs always decline as output increases?

Ans: Average fixed cost


1. In Maris’s model the goals of managers and shareholders are seen to be more compatible
than in other managerial theories because?

Ans: b

2. What is the difference b/w Debt ratio and Liquidity Ratio (LR)?

Ans: DR operates with value of debts whereas LR operates with Liquid assets

1. What is Behavioural Model of Cryert and March?

Ans: c

2. Is the marginal Product is decreasing, then marginal product:

Ans: Must be greater than AP

1. Which of the following is a positive statement?

Ans: b

2. The opportunity cost of a particular activity e. measure the direct benefits of that activity

Ans: b

1. When the marginal product of labour is greater than the average product of labour?

Ans: c
2. The Marginal cost curve intersect the:

Ans: b

1. A firm encountering economies of scale over some range of output will have

Ans: b

2. The long run is a period of time in which

Ans: b

1. When marginal revenue is equal to 0:

Ans: c

2. If the marginal revenue is $8, the firm should _______.

Ans: Raise output

1. The costs that depend on output in the short run are:

Ans: both total variable and total cost

2. Which of the following is NOT a reason for increasing prices?

Ans: Price comparison


1. If we have a Cobb-Dougal’s aggregate production function, the term of “constant returns
to scale” refers to the fact that?

Ans: c

2. If we assume a cobb-Douglas production function where the share of labor is ¾ and the
share of capital is ¼, then the marginal product of capital can be calculated as:

Ans: y/4k

1. The marginal product of labor curve shows the change in total product resulting from a:

Ans: b

2. When the total product curve is falling, the:

Ans: b

1. When marginal product reaches its maximum, what can be said of total product?

Ans: c

2. Variable costs are:

Ans: c

1. Which is not a fixed cost?

Ans: a worker’s wage of $15 per hour


2. If you know that with 8 units of output, average fixed cost is $12.50 and average variable
cost is $81.25, then total cost at this output level is:

Ans: 750

1. With fixed costs of $400, a firm has average total costs of $3 and average variable costs
of $2.50. its output is:

Ans: 800

2. The reason the marginal cost curve eventually increases as output increases for the
typically firm is because?

Ans: of the law of diminishing returns

1. Suppose that a firm produces 200,000 units a year and sells them all for $10 each. The
explicit costs of production are $1,500,000 and the implicit costs of production are $300,000.

The firm has an accounting profit of?

Ans: $500,000 and an economic profit of $200,000

2. The short run is a time period in which:

Ans: some resources

1. Implicit costs are:

Ans: c
2. Which would be an implicit cost of firm? The cost:

Ans: d

1. If a firm’s revenue just cover all its opportunity costs, then:

Ans: b

2. Suppose a firm sells its product at a price lower than the opportunity cost of the inputs
used to produce it. Which is true?

Ans: d

1. For a rational consumer who has to choose between two goods in the context of budget
constraints, the price change of one of the goods, caeteries, paribus, will determine:

Ans: b

2. The price of the product A was reduced from 100 to 90 lei and, as a result, the quantity
demanded has increased from 70 to 75 units. The demand is:

Ans: inelastic:

1. Which one of the following is a factor of production?

Ans: enterprise

2. Opportunity cost is:

Ans: measure in term of what foraging


1. If the demand curve for product A moves to the right, and the price of product B
decrease, it can be concluded that:

Ans: A and B are complementary goods

2. Suppose the price of a good decreases by 10% and the quantity demanded for a certain
period of time increase by 15%. In these conditions?

Ans: b

1. The indifference curve means:

Ans: equal utilities from the consumption of two combination of goods

2. The point located at the intersection of the budget line with the coordinate axes means?

Ans: b

1. Economic growth can be measured by:

Ans: GDP

2. In a boom:

Ans: unemployment is likely to fall

1. In a recession, GDP:
Ans: Grows negatively

2. To boost growth the government is most likely to?

Ans: c

1. A Govt. is most likely to use a reflationary policy:

Ans: In a recession

2. Potential growth measures:

Ans: The rate of growth that could be achieved if resources were fully employed

1. An increase in the wage rate:

Ans: b

2. A decrease in the supply of labor is likely to lead to:

Ans: A higher equilibrium wage and lower quantity of labour employed

1. Which of the following is a property of all linear programming problems?

Ans: Alternate courses of action to choose from

2. A point that satisfies all of a problem’s constraints simultaneously is a(n)?

Ans: None
1. If the short-run average variable costs of production for a firm are rising, then this
indicates that:

Ans: marginal cost

2. If a more efficient technology was discovered by a firm, there would be:

Ans: d

1. The law of diminishing returns states that:

Ans: c

2. The law of diminishing returns only applies in cases where?

Ans: c

1. Suppose the supply for product A is perfectly elastic. If the demand for this product
increase?

Ans: c

2. If the coefficient of income elasticity of demand is higher than 1 and the revenue
increases, the share of expenditure for commodity X in total expenditure:

Ans: Will increase

1. If the demand for agricultural products in inelastic:


Ans: b

2. For a rational consumers who has to choose between two goods in the context of budget
constraints, the price change of one of the goods, caeteris paribus, will determine?

Ans: b

1. A production function measures the relation b/w

Ans: the quantity of input and output

2. A short—run production function assumes that……

Ans: the usage of at least one input is fixed

1. If average product is decreasing, then marginal product..

Ans: b

2. Which of the following statement is true?

Ans: c

1. Suppose you operate a sandwich shop and currently have two employees. If you hire a
third employees, your output of sandwiches per day rises from 75 to 90. If you hire a fourth
employees would causes output to rises to 120 and 125 per day, respectively.

Choose the correct statement:

Ans: c
2. The marginal product of labour?

Ans: both a and c

1. Diminishing marginal productivity:

Ans: both a and b

2. Diminishing returns refers to the decrease in:

Ans: marginal product that results from increases in the variable input.

1. If a firm is producing a given level of output in a technically-efficient manner, then it


must be the case that:

Ans: both a and b

2. If a firm is producing a given level of output in an economically-efficient manner, then it


must be the case that..

Ans: both a and b

1. A short-run cost function assumes that...

Ans: c

2. Average total cost:

Ans: increases if marginal cost is greater than average total cost


1. Which of the following is a variable cost?

Ans: b

2. Which of the following is implicit cost?

Ans: b

1. If the output levels at which short-run marginal and average cost curves reach a minimum
are listed in order from smallest to greatest, then the order would be?

Ans: c

2. Learning curves represent the relation between

Ans: b

1. If an input is owned and used by a firm, then it’s?

Ans: Explicit cost of zero

2. Short-run marginal cost is equal to?

Ans: All

1. Short-run average variable cost is equal to:


Ans: All the above

2. Which of the following short-run cost curves declines continuously?

Ans: Average fixed cost

1. The law of diminishing returns begins at the level of output where?

Ans: Marginal cost is at minimum

2. The long-run average cost curve is at a minimum at a level of output where?

Ans: all the above occur

1. If a firm has a downward slopping long-run average cost curve, then

Ans: c

2. One reason that a firm may experience increasing returns to scale is that greater levels of
output make it possible for the firm to?

Ans: All

1. One reason that a firm may experience decreasing returns to scale is that greater levels of
output can result in

Ans: b

2. Economies of scope refers to the decrease in average total cost that can occur when a firm
Ans: Produce more than one product

1. Breakeven analysis identifies the

Ans: level of output where economic profit is equal to zero

2. Which of the following is* an assumption of linear breakeven analysis?

Ans: c

QUESTION 1

Calculate marginal product of each workers.

a)​ "​ 6,5,4,3,2,1"

b)​ "​ 5,4,3,2,1"

c)​ "​ 0,1,2,3,4,5"

d)​ "​ 7,6,5,4,3,2"

QUESTION 2
Calculate the marginal cost of each level of output

a)​ "​ 10, 13.5 , 17.67, 30, 55"

b)​ "​ 48, 13, 12, 10.9, 3, 1"

c)​ "​ 10, 15.5, 20.25, 40, 60"

d)​ "​ 10, 12.5, 16.67, 25, 50"

QUESTION 3

"If all resources used in the production of a product are increased by 20 percent and output
increases by 20 percent, then there must be:”

a)​ e​ conomies of scale

b)​ d​ iseconomies of scale

c)​ c​ onstant returns to scale​.

d)​ i​ ncreasing average total costs


QUESTION 4

"If the output levels at which short-run marginal and average cost curves reach a minimum are
listed in order from smallest to greatest, then the order would be"

a)​ "​ AVC, MC, ATC "

b)​ "​ ATC, AVC, MC"

c)​ "​ MC, AVC, ATC"

d)​ "​ AVC, ATC, MC"

QUESTION 5

"If an input is owned and used by a firm, then its"

a)​ e​ xplicit cost is zero

b)​ i​ mplicit cost is zero

c)​ o
​ pportunity cost is zero

d)​ e​ conomic cost is zero


QUESTION 6

Short - run marginal cost is:

a)​ t​ he change in total cost divided by the change in output

b)​ t​ he change in total variable cost divided by the change in output

c) the cost per unit of the variable input divided by the marginal product of the variable

input

d)​ a​ ll of the above

QUESTION 7

One reason that a firm may experience increasing returns to scale is that greater levels of output
make it possible for the firm to

a)​ e​ mploy more specialized machinery

b)​ o​ btain bulk purchase discounts

c)​ e​ mploy a greater division of labor

d)​ A
​ ll of the above are correct
QUESTION 8

Breakeven analysis identifies the

a)​ p
​ rofit-maximizing level of output

b)​ l​ evel of output where economic profit is equal to zero

c)​ l​ evel of output where marginal revenue is equal to marginal cost

d)​ A
​ ll of the above are correct

QUESTION 9

The responsiveness or sensitivity of a firm's profits to changes in output is measured by a firm's

a)​ o
​ perating leverage

b)​ c​ ontribution margin per unit

c)​ d
​ egree of operating leverage

d)​ r​ eturns to scale

QUESTION 10

"If a linear short-run variable cost function is estimated using cross-sectional data, then the
corresponding marginal cost function will be"
a)​ U
​ -shaped

b)​ U
​ pward sloping

c)​ D
​ ownward Sloping

d) Horizontal

1. In the context of decision making, which of the following best describe a heuristics?

Ans: c

2. In our model of decision making under different conditions, what is the difference
between risk and uncertainty?

Ans: b

+​1. The material wealth of a society is equal to the sum of_____

Ans: All real assets

2. Financial intermediaries exists because small investors can’t efficiently _________.

Ans: All the above

1. Cost of product that can be added to other large purchase of product is classified as:

Ans: Integrate loss


2. When customers evaluate more than one positive dimensions separately is classified.

Ans: Segrate gains

1. When the general level of prices is rising, we call that

Ans: Inflation

2. When prices rise slowly and predictably, we call that:

Ans: low inflation

1. Way in which choices are seen and presented by a decision maker is classified as:

Ans: decision framing

2. Customers who loss averse and tend to underweight the

Ans: c

1. Which of the following is GDP at current prices?

Ans: Nominal GDP

2. Which of the following is the term used to describe an addition to a nation’s capital
stock?

Ans: investment
1. On the circular flow diagram of the economy, the arrow from the producer sector to the
overseas sector usually representation?

Ans: import payment

2. The circular flow diagram is a model showing

Ans: c

1. NDPMP will be equal to:

Ans: NDPFC + Net Indirect Taxes

2. Which of the following is not the form of Tax Revenue?

Ans: License fee

1. The net value of GDP after deducting depreciation from GDP is

Ans: Net domestic

2. When depreciation is deducted from GNP, the net value is:

Ans: Net national product

1. In the value of NNP at consumer point as:


Ans: NNP at market price

2. The value of NNP at production point is called:

Ans: b

1. Fiscal policy refers to:

Ans: b

2. To help fight a recession, the govt. could

Ans: b

1. An increase in the marginal propensity to consume will:

Ans: a

2. As national income increase:

Ans: a

1. The ________refers to the orderly relationship between spot and forwarded currency
exchange rates and the rates of interest between countries?

Ans: b

2. The _____ is especially well suited to offer hedging protection against transactions risk
exposure
Ans: Forward market

1. For regulation of insurance trade in the country has formed:

Ans: IrDA

2. In the context of an ’Indian-Economy’, open market operation refer to?

Ans: c

1. The key issues of macroeconomics are:

Ans: All the above

2. In the economy when a steel producers sells steel to car producers, it is regarded as__

Ans: intermediate goods

1. In our model of decision making under different conditions, what is the difference
between risk and uncertainty?

Ans: d

2. In the context of decision making, which of the following best describes a heuristic?

Ans: d
1) A Situation in which a decision maker knows all of the possible outcomes of a decision
and also known the probability associated with each outcome is referred to as

Ans: risk

2) Which of the following methods of selecting a strategy is consistent with risk averting
behaviour?

Ans: a

1. To increase a given future value, the discount rate should be adjusted____.

Ans: upward

2. Interest paid on only the original principal borrowed is often referred as_______.

Ans: Simple interest

1. Time value of money indicates that?

Ans: a

2. Time value of money supports the comparison of cash flows recorded at different time
period by?

Ans: Using either a or b

1. A cause of inflation

a)​ I​ ncrease in money supply


b) increase in money supply and a fall in production

c)​ F
​ all in production

d)​ d
​ ecrease in money supply and fall in production

QUESTION 2

Inflation brings more benefit to which one of the following?

• Government pensioners

• Creditors

• Savings Bank Account Holders

• Debtors

QUESTION 3

Inflation is mostly harmful to which one of the following?

• Debtors

• Creditors

• Business class

• Holder of real assets


QUESTION 4

Increasing unemployment and inflation is a situation of:

• Hyperinflation

• Galloping inflation

• Stagflation

• Reflation

QUESTION 5

Who among the following are not protected against inflation?

• Salaried class

• Industrial workers

• Pensioners

• Agricultural farmers

QUESTION 6

"The period of high inflation, low economic growth and high unemployment rate is termed as:”

• Stagnation

• Take - off stage in economy


• Stagflation

• None of the above

QUESTION 7

"Deficit financing aims to put more money into the economy by creating additional paper
currency to fill the gap between expenditure and revenue. The device aims at economic
development but if it fails, it generates"

• Inflation

• Deflation

• Devaluation

• Demonitisation

QUESTION 8

A steady increase in the general level of prices as a result the aggregate demand is increasing in
unsustainable rate as compared to aggregate supply is termed as

• Demand - pull inflation

• Cost push inflation

• Stagflation

• Structural Inflation
QUESTION 9

Which is correct in respect to inflation?

• rise in budget deficits

• rise in money supply

• rise in general price index

• rise in price of consumer goods

QUESTION 10

Which of the following is not a reason for inflation?

• Increase in administered prices

• Increase in cost of capital

• More dependence on indirect taxes for revenue

• None of these

Question 1

The revenue earned at Downtown Bakery is equal to

area A+ B

a)​ a​ rea B
b)​ a​ rea A

c)​ a​ rea C

Question 2

Area A is bigger than area C. This means that

a)​ T
​ he quantity demanded of chocolate cake exceeds the quantity supplied

b)​ D
​ emand for chocolate cake is price inelastic

c)​ D
​ emand for chocolate cake is highly price elastic

d)​ T
​ he quantity supplied of chocolate cake exceeds the quantity demanded.

Question 3

Which of the following statements is true?

a)​ E
​ lasticity is identical to the slope of the demand curve.

b) ​"A single, straight-line demand curve can be elastic in one region and inelastic in

another"

c)​ P
​ erfectly inelastic demand can be represented by a horizontal line

d)​ "​ When demand is unit elastic, revenue is strongly affected by price changes."
Question 4

Suppose a 50% increase in the price of a drug results in no change in the quantity demanded.
What is the price elasticity of the drug?

a)​ 0

b)​ 0​ .05

c)​ 0
​ .5

d)​ 1

Question 5

Which one of the following goods is most likely to have a perfectly elastic demand?

a)​ S
​ hoes

b)​ R
​ ice in a developing country

c)​ a​ particular brand of butter

d)​ C
​ igrattes

Question 6

"A population subsists largely on potatoes, plus small amounts of dairy products and vegetables.
The price of potatoes rises, driving many poor families deeper into poverty. As a result, these
families are forced to eliminate dairy products and vegetables from their daily diet and start
eating even more potatoes than they did before. In this example potatoes are"
a)​ i​ nferior goods

b)​ n​ ormal goods

c)​ b
​ oth b and c

d)​ G
​ iffen goods

Question 7

Calculate private income

a)​ 3
​ 25 crores

b)​ 3​ 50 crores

c)​ 3
​ 90 crores

d)​ 3
​ 55 crores

Question 8

Calculate national income

a)​ 4
​ 60 crores

b)​ 2​ 80 crores

c)​ 2
​ 45 crores

d)​ 6
​ 60 crores
Question 9

Calculate personal disposable income

a)​ 2
​ 25 crores

b)​ 3​ 15 crores

c)​ 3
​ 20 crores

d)​ 3
​ 05 crores

Question 10

The financial year in India is

a)​ A
​ pril 1 to March 31

b)​ M
​ arch 16 to March 15

c)​ J​ anuary 1 to December 31

d)​ M
​ arch 1 to April 31

1. What is Scarcity?

a)​ R
​ elationship between limited resources and unlimited wants

b)​ R
​ elationship between limited wants and limited resources

c)​ R
​ elationship between unlimited resources and limited wants
d)​ N
​ one of the above

Question 2

Investment problems lead to:

a)​ H
​ ow much to expand a firm

b)​ H
​ ow much to invest

c)​ I​ n which to invest

d) All of the above

Question 3

Indifference is

a)​ C
​ oncave to the origin

b) Convex to the origin

c)​ H
​ orizontal parallel to x-axis

d)​ N
​ one of the above

Question 4

MRTS is equal to ratio of commodity prices which known as

a)​ C
​ onsumer equilibrium

b)​ I​ ndifference curve

c)​ C
​ onsumer surplus
d)​ D
​ emand Elasticity

Question 5

Budget Constraint refers to

a)​ C
​ ombination of goods a consumer can purchase with income as a constraint

b)​ C
​ ombination of only two goods a consumer can purchase with income as a constraint

c) ​Combination of goods and services a consumer can purchase with income as a


constraint

d)​ N
​ one of the above

Question 6

Total fixed costs remain ________ as output increases

a)​ C
​ onstant

b)​ u​ pward sloping

c)​ d
​ ownward sloping

d)​ N
​ one of the above

Question 7

"In second stage of production, TP increases but _____ than proportionate to increase in labor"

a)​ M
​ ore

b)​ L
​ ess
c)​ E
​ qual

d)​ N
​ one of the above

Question 8

Price discrimination in which seller charges different prices for different classes for buyers is
classified as

a)​ f​ ourth-degree discrimination

b)​ s​ econd-degree price discrimination

c)​ f​ irst-degree price discrimination

d) third-degree discrimination

Question 9

"Image pricing, location pricing, channel pricing and time pricing are all types of price
discrimination of"

a)​ F
​ irst degree

b)​ S
​ econd degree

c)​ T
​ hird degree

d)​ F
​ ourth degree

Question 10

"Price discrimination in which seller charges less to customer's, who buy in large volumes is
classified as"
a)​ F
​ irst degree

b)​ S
​ econd degree

c)​ T
​ hird degree

d)​ F
​ ourth degree

Question 11

Scarcity implies that the allocation scheme chosen by society can

a)​ N
​ ot make more of any one good

b)​ T
​ ypically make more of a good but at the expense of making less of another

c)​ A
​ lways make more of all goods simultaneously

d)​ N
​ one of the above

Question 12

The short run is a time period in which:

a)​ a​ ll resources are fixed

b)​ t​ he level of output is fixed

c)​ t​ he size of the production plant is variable.

d) some resources are fixed and others are variable.

Question 13

Variable costs are:


a)​ s​ unk costs.

b)​ m
​ ultiplied by fixed costs.

c) costs that change with the level of production

d) d​ efined as the change in total cost resulting from the production of an additional unit
of output

Question 14

When marginal product reaches its maximum, what can be said of total product?

a)​ t​ otal product must be at its maximum

b)​ t​ otal product starts to decline even if marginal product is positive

c)​ t​ otal product is increasing if marginal product is still positive

d)​ t​ otal product levels off

Question 15

The marginal product of labor curve shows the change in total product resulting from a:

a) ​one-unit increase in the quantity of a particular resource used, letting other resources

vary.

b) one-unit increase in the quantity of a particular resource used, holding constant other

resources.

c)​ c​ hange in the cost of a variable resource.

d)​ c​ hange in the cost of a fixed resource.

Question 1. Economics is the study of


o​ p​ roduction technology

o​ c​ onsumption decisions

o​ "​ how society decides what, how, and for whom to produce"

o​ t​ he best way to run society

Question 2. The opportunity cost of a good is

a. the time lost in finding it

b.​ t​ he quantity of other goods sacrificed to get another unit of that good

c. the expenditure on the good

d. the loss of interest in using savings

Question 3. A market can accurately be described as

a. a place to buy things

b. a place to sell things

c.​ the process by which prices adjust to reconcile the allocation of resources

d. a place where buyers and sellers meet


Question 4. In a free market __________ ___________

a. governments intervene

b. governments plan production

c. governments interfere

d. prices adjust to reconcile scarcity and desires

Question 5. In the mixed economy

a.​ economic problems are solved by the government and market


b. economic decisions are made by the private sector and free market

c. economic allocation is achieved by the invisible hand

d. economic questions are solved by government departments

Question 6. Normative economics forms ___________ based on _____________

a. "positive statements, facts"

b.​ "​ opinions, personal values"

c. "positive statements, values"

d. "opinions, facts"
Question 7. Microeconomics is concerned with

a. the economy as a whole

b. the electronics industry

c.​ the study of individual economic behaviour


d. the interactions within the entire economy

Question 8. Macroeconomics is the study of ___________________

a. individual building blocks in the economy

b. the relationship between different sectors of the economy

c. household purchase decisions

d.​ t​ he economy as a whole

Question 9. Data are important in economics because __________ and __________

a)​ "​ they suggest relationships for explanation, allow testing of hypotheses"

b) " they can be used for tables, they can be graphed"

c) "they can be used in computers, governments use them"


d) "they provide interesting information, can be summarised"

Question 10. Time series data show information

a. about the same point in time over different places

b.​ a​ bout different points in time over the same variable

c. about different variables over different places

d. about different points in time over different places

Question No. 11

The retail price index is used to ______________

Options

1.​ construct price lists


2.​ compare shop prices


3.​ measure changes in the cost of living


4.​ none of the above



Question No. 12 Marks - 10

A real value can be derived from a nominal value by

Options

1.​ adjusting for changes over time


2.​ adjusting for data collection errors


3.​ adjusting for population changes


4.​ adjusting for changes in prices


Question No. 13 Marks - 10

If your income during one year is £10,000 and the following year it is £12,000, then it has grown
by

Options

1.​ 20%

2.​ 2%

3.​ 12%

4.​ 16%

Question No. 14 Marks - 10

A straight-line diagram can be drawn knowing the ______ and _________

Options

1.​ vertical axis and horizontal axis


2.​ intercept and slope


3.​ scale and slope


4.​ intercept and scale


Question No. 15 Marks - 10

On a graph, a positive linear relationship

Options

1.​ moves down to the right


2.​ moves up to the left


3.​ moves up to the right


4.​ moves down to the left



Question No. 16 Marks - 10

If the diagram of a line shows that lower values on the vertical scale are associated with higher
values on the horizontal scale, this is an example of _____________

Options

1.​ a nonlinear relationship


2.​ a positive linear relationship


3.​ a scatter diagram


4.​ a negative linear relationship


Question No. 17 Marks - 10

When we know the quantity of a product that buyers wish to purchase at each possible price, we
know

Options

1.​ Demand

2.​ Supply

3.​ Excess demand



4.​ Excess supply

Question No. 18 Marks - 10

The equilibrium price clears the market; it is the price at which ________ _________

Options

1.​ Everything is sold


2.​ Quantity demanded equals quantity supplied


3.​ Excess demand is zero


4.​ b&c

Question No. 19 Marks - 10

When a market is in equilibrium

Options

1.​ Quantity demanded equals quantity supplied


2.​ Excess demand and excess supply are zero


3.​ The market is cleared by the equilibrium price



4.​ All of the above

Question No. 20 Marks - 10

________ and ________ do not directly affect the demand curve

Options

a)​ t​ he price of related goods, consumer incomes

b)​ c​ onsumer incomes, tastes

c)​ t​ he costs of production, bank opening hours

d) the price of related goods, preferences

Question No. 21 Marks - 10

A demand curve can shift because of changing


Options

1.​ incomes

2.​ prices of related goods


3.​ tastes

4.​ all of the above


Question No. 22 Marks - 10

A supply curve is directly affected by

Options

1.​ technology

2.​ input costs


3.​ government regulation


4.​ all of the above


Question No. 23 Marks - 10

If a price increase of good A increases the quantity demanded of good B, then good B is a
Options

a)​ s​ ubstitute good

b)​ c​ omplementary good

c)​ b
​ argain

d) inferior good

Question No. 24 Marks - 10

An increase in consumer income will increase demand for a _______ but decrease demand for a
_________

Options

a)​ s​ ubstitute good, inferior good

b)​ n​ ormal good, inferior good

c)​ i​ nferior good, normal good

d) normal good, complementary good

Question No. 25 Marks - 10

The price elasticity of demand measures ________________


Options

a)​ t​ he responsiveness of quantity demanded to a change in price

b)​ h​ ow far a demand curve shifts

c)​ a​ change in price

d) a change in quantity demanded

Question No. 26 Marks - 10

If demand is ___________ then price cuts will __________ spending

Options

a)​ i​ nelastic, increase

b)​ e​ lastic, increase

c)​ e​ lastic, decrease

d) none of the above

Question No. 27 Marks - 10


Positive cross-elasticities suggest that goods are _________ and negative cross-elasticities that
goods are __________

Options

a)​ s​ ubstitutes, inferior

b)​ n​ ormal, complements

c)​ s​ ubstitutes, complements

d) normal, inferior

Question No. 28 Marks - 10

A measurement showing how quantity demanded varies with income is the

Options

a)​ p
​ rice elasticity of demand

b)​ c​ ross-price elasticity of demand

c)​ b
​ udget elasticity of demand

d)​ i​ ncome elasticity of demand


Question No. 29 Marks - 10

Inferior goods have ___________ and luxury goods have ____________

Options

a)​ n
​ egative income elasticities, income elasticities greater than 1

b)​ i​ ncome elasticities greater than 1, negative income elasticities

c)​ p
​ ositive income elasticities, negative income elasticities

d) none of the above

Question No. 30 Marks - 10

If your income doubles and the prices of the goods you buy double, then your demand for these
goods will likely ________

Options

a)​ i​ ncrease

b)​ n​ ot change

c)​ d
​ ecrease

d) shift
Question No. 31 Marks - 10

The income effect of a price increase of a normal good is to __________ of that good and the
substitution effect is to _______ of that good

Options

a)​ i​ ncrease quantity demanded, reduce quantity demanded

b)​ i​ ncrease quantity demanded, increase quantity demanded

c)​ r​ educe quantity demanded, reduce quantity demanded

d) reduce quantity demanded, increase quantity demanded

Question No. 32 Marks - 10

The opportunity cost of a student is

Options

a)​ C
​ ourse fees and rent

b)​ A
​ loan from the bank

c)​ W
​ hat the student could have earned in the best job available by not studying

d) What the student will earn after graduation


Question No. 33 Marks - 10

Economics assumes that people consume goods and services to achieve

Options

a)​ S
​ tatus

b)​ P
​ restige

c)​ U
​ tility

d) Self-esteem

Question No. 34 Marks - 10

The extra utility from consuming one more unit of a good is called

Options

a)​ M
​ arginal utility

b)​ A
​ dditional utility

c)​ S
​ urplus utility

d) Bonus utility
Question No. 35 Marks - 10

Adding up the quantities demanded of a good by different people facing the same price gives us
the

Options

a)​ S
​ upply curve

b)​ M
​ arket demand curve

c)​ D
​ emand curve

d) Market supply curve

Question No. 36 Marks - 10

Firms are assumed to _________ costs and to _________ profits

Options

a)​ i​ ncur, desire

b)​ p​ ay, make

c)​ c​ harge, earn


d)​ m
​ inimize, maximize

Question No. 37 Marks - 10

The increase in total cost when one more unit is produced is known as

Options

a)​ m
​ arginal cost

b)​ o​ pportunity cost

c)​ l​ imited cost

d) average cost

Question No. 38 Marks - 10

Marginal revenue is the _________ when output is ____________

Options

a)​ c​ hange in average revenue, increased

b)​ c​ hange in total revenue, increased by one unit

c)​ c​ hange in average revenue, increased by one unit


d) change in total revenue, increased

Question No. 39 Marks - 10

Profits are maximized when _________________

Options

a)​ c​ osts are minimized

b)​ r​ evenue is maximized

c)​ a​ verage cost is less than average revenue

d)​ m
​ arginal cost equals marginal revenue

Question No. 40 Marks - 10

If a firm's wage costs increase this will cause __________ and ________

Options

a)​ m
​ arginal cost to increase, output to fall

b)​ m
​ arginal revenue to increase, output to fall

c)​ o
​ pportunity cost to increase, the firm will close
d) average cost will rise, output will increase

QUESTION 1

Demand is determined by

a)​ P
​ rice of the product

b)​ R
​ elative prices of other goods

c)​ T
​ astes and habits

d)​ A
​ ll of the above

ANSWER: - All of the above

QUESTION 2

Managerial economics generally refers to the integration of economic theory with business

a.​ Ethics

b.​ M
​ anagement

c.​ Practice

d.​ A
​ ll of the above

ANSWER: - Practice

QUESTION 3

Which of the following statements is true?

a.​ "When the supply increases, both the price and the quantity will increase."

b. An increase in the quantity supplied leads to a fall in the price resulting in the

shifting of the supply curve towards the left.

c.​ A shift in the supply curve towards the right results in a fall in the price

d. ​A decrease in the quantity supplied results in shifting of the supply curve

towards the left.

ANSWER: - An increase in the quantity supplied leads to a fall in the price resulting in the
shifting of the supply curve towards the left.

QUESTION 4

The problems in business decision-making and forward planning can be grouped into
___________ Categories

a.​ 4

b.​ 3​

c.​ 2

d.​ 5​

ANSWER:-3

QUESTION 5

The root cause of all economic problems is__________

a.​ Demand

b.​ S
​ carcity

c.​ Supply

d.​ C
​ ost

ANSWER: - Scarcity

QUESTION 6

"The cost of the particular best choice is the benefit of the next best alternative foregone, known
as __________."

a.​ Discounting Time Perspective


b.​ M
​ arginalism

c.​ Opportunity Cost


d.​ D
​ emand

ANSWER: - Opportunity Cost

QUESTION 7

The degree of uncertainty and risk can be greatly reduced if__________ are predicted with a
high degree of reality.

a.​ Opportunity Costs


b.​ R
​ esources allocation

c.​ Demand & Supply


d.​ M
​ arket Conditions

ANSWER: - Demand & Supply


QUESTION 8

Which of these topics are covered under Managerial economics

a.​ Profit Analysis.


b.​ C
​ ost Analysis

c.​ Marginal analysis


d.​ A
​ ll of these

ANSWER: - All of these

QUESTION 9

The term differentiated product denotes

a.​ Different products in similar packets


b.​ D
​ ifferent products

c.​ Same product used in different applications


d.​ D
​ ifferent products used by a differentiated set of people

ANSWER: - Different products used by a differentiated set of people

QUESTION 10

"In the simple formula for discounting, P denotes to:”

a.​ Present value of fund


b.​ P
​ eriod
c.​ Rate of discount

d.​ P
​ olicy

ANSWER: - Rate of discount

ECONOMICS FOR MANAGER

QUESTION 1

The obligatory condition for the maximization of output given the factor prices:

a.​ Isoquant line must be tangent to one of the isocosts.


b. Isoquant line must be tangent to all the isocosts.

c.​ Isoquant line need not be tangent to isocost.


d.​ I​ soquant line needs to be parallel to isocost.

QUESTION 2

"Q = f (L, K, I, R, E)..... iss considered as"

a.​ Production Function


b. Production Function with one variable inputs

c.​ Production Function with two variable inputs


d.​ P
​ roduction Function with various inputs
QUESTION 3

"The 'law of diminishing returns to scale' refers to the general tendency for ___to eventually
diminish as more of the variable input is employed, given the quantity of fixed inputs."

a. marginal product

b.​ a​ verage total cost

c.​ marginal cost


d.​ a​ verage product

QUESTION 4

Marginal cost is computed as

a. Prime cost + All Variable overheads

b.​ D
​ irect material + Direct labor + Direct Expenses + All variable overheads

c.​ Total costs All fixed overheads


d.​ A
​ ll of the above

QUESTION 5

A monopolistically competitive firm in long-run equilibrium

a.​ will make negative profit


b. will make zero profit

c.​ will make positive profit



d.​ A
​ ny of the above are possible

QUESTION 6

A monopolistically competitive firm in short-run equilibrium

a.​ will make negative proit (lose money).


b. will make zero profit (break-even).

c.​ will make positive profit.


d.​ A
​ ny of the above are possible

QUESTION 7

Perfect competition occurs in a market where there are many firms each selling:

a. an identical product

b.​ a​ similar product

c.​ a unique product


d.​ a​ competitive product.

QUESTION 8

"In perfect competition, a firm's marginal revenue equals its:"

a.​ average revenue


b. price

c.​ total revenue



d.​ b​ oth a and b

QUESTION 9

"In a Sweezy Oligopoly, if a firm decreases its prices, a competitor will?"

a.​ Increase prices


b. Decrease prices

c.​ Keep their prices constant


d.​ E
​ ither B or C

QUESTION 10

A cooperative buying group is an example of...

a.​ A factor leading to internal economies of scale


b. A factor leading to external economies of scale

c.​ Both A and B


d.​ N
​ one of the above

QUESTION 11

"The marginal principle asserts that, in general, when net benefit is maximized"

a.​ total benefit will be equal to total cost.


b.​ a​ verage benefit will be equal to average cost.

c. marginal benefit will be equal to marginal cost.


d.​ a​ verage cost will be above total cost but below average benefit.

QUESTION 12

A stable equilibrium in the Prisoner's Dilemma game is known as a:

a.​ Baumol equilibrium.


b.​ P
​ orter equilibrium.

c. Nash equilibrium.

d.​ D
​ ouglas equilibrium.

QUESTION 13

"If both average cost (AC) and marginal cost (MC) are U shaped, then"

a. ​AC will reach a minimum at a level of output that is less than that at which

MC reaches a minimum.

b.​ t​ he total cost curve will be a straight line.

c. ​AC will reach a minimum at a level of output that is greater than that at which

MC reaches a minimum.

d. both AC and MC will reach a minimum at the same level of output

QUESTION 14

"If firms compete in a Cournot fashion, then"

a. each firm views the output of the rival as given.

b.​ e​ ach firm views the prices of rivals as given.


c.​ each firm views the profits of rivals as given.

d.​ a​ ll of the above

QUESTION 15

Marginal cost (MC) curve intersects the

a.​ ATC and AFC curves at their minimum points.


b.​ "​ ATC, AVC and AFC curves at their minimum points"

c.​ AVC and AFC curves at their minimum points


d. ATC and AVC curves at their minimum points

QUESTION 16

Game theory is concerned with:

a.​ How production can be managed at least cost.


b. How individuals make decisions taking into account the actions of others.

c.​ Managing a financial portfolio to minimize risk.


d.​ N
​ one of the above.

QUESTION 17

"In the long run, the typical firm:"

a.​ has all inputs fixed except one.


b. Has only variable inputs


c.​ May change some but not of all its inputs

d.​ I​ s concerned with its variable cost of production

QUESTION 18

"If a firm's revenues just cover all its opportunity costs, then:"

a.​ normal profit is zero


b. economic profit is zero

c.​ total revenues equal its explicit costs


d.​ t​ otal revenues equal its implicit costs

QUESTION 19

"For a firm to maximize it s profit, the level of output should reach at following two marginals:"

a. MC=MR; Slope of MR<="" label="" style="font-family: "Open Sans",


sans-serif; text-shadow: none !important; letter-spacing: normal !important;">

b. MC>MR; Slope of MR= Slope of MC

c.​ MC=MR; Slope of MR= Slope of MC


d.​ M
​ C Slope of MC

QUESTION 20

A long-run is also expressed as a series of short-runs.

a. TRUE
b.​ F
​ ALSE

c.​ Can t say


d.​ N
​ one of the above

ECONOMICS FOR MANAGERS

QUESTION 1

Tastes & first choices are determinants of

supply

demand

demand curve

elasticity

QUESTION 2

Increase in demand is shown by:

Upward movement on the demand curve

Downward movement on the demand curve

Rightward shift of the demand curve

Leftward shift of the demand curve


QUESTION 3

An indifference curve shows combinations of two goods that:

could be available to the consumer in a given time period

would provide the consumer with the same level of satisfaction

could provide the consumer with similar levels of satisfaction

a consumer could buy with their given income.

QUESTION 4

"Customers will be ready to purchase a specified quantity of a product, at a specified price, if


marginal utility of further spending is equivalent to the"

Cost

opportunity cost

revenue

product cost

QUESTION 5

Law of demand shows relation between:

Income and price of commodity

Price and quantity of a commodity

Income and quantity demand


Quantity demanded and quantity supplied

QUESTION 6

Economics is

the study of the markets for stocks and bonds

the study of choice under conditions of scarcity

exclusively the study of business firms

fundamentally the same as sociology e. applicable only when scarcity is not a problem

QUESTION 7

"Demand Function, X = f (P) where X = demand of a commodity; P = Price of the commodity"

Demand is directly related to price

Price is inversely related to demand

Price is directly related to demand

None of the above

QUESTION 8

Which of the following statements is NOT TRUE of indifference curves?

They exhibit higher levels of utility as you move from the origin

They could intersect

They are downward sloping


They are convex to the origin

QUESTION 9

Coefficient of elasticity of demand is negative. It means:

Consumers sometimes buy negative units of commodity

Price and quantity demanded move in same capital

Law of demand holds

The two goods are complimentary to each other

QUESTION 10

"In our model of decision making under different conditions, what is the difference between risk
and uncertainty?"

"Under risk, there is a well defined problem; under uncertainty, the definition is unclear"

"Under risk, information is reliable; under uncertainty, it is not"

"Under risk, probabilities can be measured; under uncertainty, they cannot"

"Under risk, choices are clear and the chances of different outcomes can be measured;
under uncertainty, neither applies"

QUESTION 11

Ostentation means goods purchased not for _______ but for snob appeal.

Utility

Direct satisfaction
Demand

None of these

QUESTION 12

This is an assumption of law of demand:

Price of the commodity should not change

Quantity should not change

Supply should not change

Income of consumer should not change

QUESTION 13

Expansion of demand is shown by:

Upward movement on the demand curve

Downward movement on the demand curve

Rightward shift of the demand curve

Leftward shift of the demand curve

QUESTION 14

An indifference curve between two commodities where one is a bad and the other a good would:

be vertical to the axis measuring units consumed of the good

remain downward sloping


be vertical to the axis measuring units consumed of the bad

be upward sloping

QUESTION 15

"In the case of a Giffen good, the demand curve will be:"

Horizontal

Downward-sloping to the right.

Vertical

Upward-sloping to the right

QUESTION 16

People and organizations have to make choices about how to allocate time and money because of

government rules and regulations

corporate control of our lives

scarcity of time and money

religious values

QUESTION 17

Contraction of demand is shown by:

Upward movement on the demand curve

Downward movement on the demand curve


Rightward shift of the demand curve

Leftward shift of the demand curve

QUESTION 18

The situation in which limited resources are being used most effectively is called:

efficient

economic

abundant

scarce

QUESTION 19

The opportunity cost of a particular activity

is the same for everyone pursuing this activity

may include both monetary costs and forgone income

always decreases as more of that activity is pursued

usually is known with certainty e. measures the direct benefits of that activity

QUESTION 20

The three fundamental questions of economic organization are:

"when, for whom, and how"

"how, what, and for whom"


"who, how, and when"

"what, who, and why"

You might also like